SlideShare a Scribd company logo
1 of 115
Download to read offline
1
Ph n 1:
C C TR TRONG I S :
M t s d ng toán thư ng g p:
▼ D ng 1: ưa v d ng bình phương
I. Phương pháp gi :
ưa v d ng
A2
≥0, ho c A2
+ c≥ c (v I c là h ng s ) d u b ng x y ra khi A=0
II. M t s bài t p ví d :
Ví d 1:
Tìm giá tr l n nh t c a P ( )1x x= −
L i gi i:
( )
2
1 1 1
1
2 4 4
P x x x x x
 
= − = − + = − − + ≤ 
 
ng th c x y ra khi
1
2
x = và
1
4
x =
Do ó giá tr l n nh t c a P là
1
4
t khi
1
4
x =
Ví d 2:
Tìm giá tr c a x bi u th c 2
1
2 2 5x x− +
có giá tr l n nh t
L i gi i:
Ta có:
( )
2
2
2
2 2 5 2 3 3
1 1
32 2 5
x x x
x x
− + = − + ≥
⇒ ≤
− +
Do ó, khi 2x = thì b êu th c 2
1
2 2 5x x− +
có giá tr l n nh t là
1
3
V í d 3:
V I x,y không âm; tìm giá tr nh nh t c a bi u th c:
2 3 2 2004,5P x xy y x= − + − +
L i gi i:
t ,x a y b= = v I , 0a b ≥ ta có:
2
( )
( ) ( )
( )
( )
2 2
2 2
22 2
2 2
2
2
2 3 2 2004,5
2 1 3 2004,5
2 1 1 2 2 2003,5
1 1
1 2 2003,5
4 2
1
1 2 2003 2003
2
P a ab b a
a b a b
a b a b b b
a b b b
a b b
= − + − +
= − + + +
= − + + + + − +
 
= − − + − + + − 
 
 
= − − + − + ≥ 
 
Vì ( )
2
1 0a b− − ≥ và
2
1
0 ,
2
b a b
 
− ≥ ∀ 
 
1a b= +
3
2
a =
2003P = ⇔ ⇔
1
2
b =
1
2
b =
V y P t giá tr nh nh t là 2003 khi
3
2
x = và
1
2
y = hay
9
4
x = và
1
4
y =
III. Bài t p t gi i:
1) Tìm giá tr l n nh t c a bi u th c: 2 2
2 5 4 2P x y xy x= − − − +
2) Tìm giá tr nh nh t c a ( ) 2 2
, 2 6 12 45f x y x xy y x= − + − +
3) Cho hai s x,y tho mãn ng th c: 2 2
2
1
8 4
4
x y
x
+ + =
Xác nh x,y tích xy t giá tr nh nh t
4) Cho a là s c nh, còn x, y là nh ng s bi n thiên. Hãy tìm giá tr nh
nh t c a bi u th c: A = (x– 2y + 1)2
+ (2x + ay +5)2
Hư ng d n gi I và áp s :
1)Max P = 3 khi (x,y) = (1, -2)
2) ( ) ( )
2 2
, 6 5 9 9f x y x y y= − − + + ≥
3) Thêm 2
4 4xy x+ vào 2 v
K t qu : xy t GTNN là
1
2
− khi
1
2
x = ± 1y = ±
4) 0A ≥ khi a ≠ -4,
9
5
A = khi a = -4
3
▼ D ng 2: s d ng mi n giá tr c a hàm s
I. Phương pháp gi :
Cho y = f(x) xác nh trên D
( )0y f D∈ ⇔ phương trình ( )0y f x= có nghi m 0a y b⇔ ≤ ≤
Khi ó min y = a, max y = b
II. M t s bài t p ví d :
Ví d 1:
Tìm Max và Min c a: 2
1
x
y
x
=
+
L i gi i:
T p xác nh D = R ⇒ 0y là m t giá tr c a hàm s
⇔ phương trình 0 2
1
x
y
x
=
+
có 1 nghi m x∈R
⇔ phương trình 2
0 0x y y x+ = có nghi m x∈R
⇔ phương trình 2
0 0 0x y x y− + = có nghi m x∈R
⇔ 0∆ ≥
⇔ 2
1 4 0y− ≥
⇔ 2
4y ≤
⇔
1 1
2 2
y− ≤ ≤
V y Min y =
1
2
− , Max y =
1
2
Ví d 2:
Xác inh các tham s a, b sao cho hàm s 2
ax
1
b
y
x
+
=
+
t giá tr l n nh t b ng
4, giá tr nh nh t b ng –1
L i gi i:
T p xác nh D = R
0y là m t giá tr c a hàm s ⇔ phương trình 0 2
ax+b
1
y
x
=
+
có nghi m x∈R
⇔ phương trình 2
0 0ax 0y x y b− + − = có nghi m x∈R (1)
• N u 0 0y = thì (1) ⇔ ax = -b có nghi m
a = b = 0
⇔
a ≠ 0
• N u 0 0y ≠ thì (1) có nghi m ⇔ 0∆ ≥
⇔ 2
0 04( ) 0a y b y− − ≥
4
⇔ 2 2
0 04 4 0y by a− + + ≥
Theo 0y t giá tr l n nh t là 4, giá tr nh nh t là –1 nên phương
trình
2 2
0 04 4y by a− + + ph I có nghi m là –1 và 4 (do -1.4 = -4 < 0)
2
4
4
a−
= − 4a = ±
Theo nh lý Viet ta có : ⇔
3b = 3b =
V y v I a = 4, b = 3 ho c a = -4, b = 3 thì min y = -1, max y = 4
Ví d 3:
Tìm giá tr l n nh t c a hàm s :
3
4
2
12 ( )
36
x x a
y
x
− 
=  + 
L i gi i: Hàm s ã cho xác nh khi ( ) 0x x a− ≥
t 2
12 ( )
36
x x a
z
x
− 
=  + 
(1) thì 4 3
y z= , 0z ≥
0z là m t giá tr c a hàm s (1) ⇔ phương trình 0 2
12 ( )
36
x x a
z
x
−
=
+
có nghi m
hay phương trình 2
0 0(12 ) 12ax 36 0z x z− − − = có
nghi m (2)
• 0z =12 : (2) ⇔ ax = -36 có nghi m khi 0a ≠
• 0 12z ≠ : (2) có nghi m ⇔ 2
0 036 36 (12 ) 0a z z∆ = + − ≥
2 2
0 0
2 2
0 0
2 2
0
12 0
12 0
6 36 6 36
a z z
z z a
a z a
⇔ + − ≥
⇔ − − ≤
⇔ − + ≤ ≤ + +
Vì 0 0z ≥ nên 2
00 6 36z a≤ ≤ + +
V y max 2
6 36z a= + + ; max 2 34
(6 36)y a= + +
III. Bài t p t gi i:
1) Tìm giá tr l n nh t, nh nh t c a bi u th c:
2
2
2 2
2 2
x x
y
x x
− +
=
+ +
2) Tìm giá tr l n nh t, nh nh t c a bi u th c:
3 3 4 1 1
4 3 3 1 1
x x
y
x x
+ + − +
=
+ + − +
3) Tìm giá tr nh nh t c a hàm s : 2 1
( )f x x x
x
= + + , x > 0
Hư ng d n gi I và áp s :
5
1) Max 3 2 2y = + , Min 3 2 2y = −
2) k: 3 1x− ≤ ≤
t 2
2
3 2.
1
t
x
t
+ =
+
;
2
2
1
1 2.
1
t
x
t
−
+ =
+
v I t = tg [ ]0;1
2
ϕ
∈
Ta có
2
2
7 12 9
5 16 7
t t
y
t
+ +
= −
− + +
Max y
9
7
y = khi x = -3; min
7
9
y = khi x = 1
0 < x ≤ 0y (1)
2
0
1
y x x
x
= + + ⇔
x > 0 2 2
0 02 1 0y x y x− + =
(2)
i u ki n (2) có nghi m là 0 2y ≥
Áp d ng Vi-et ta ch ng minh ư c 1 2 0x x y< <
V y min f(x) = 2 v I x >0
▼ Dang 3: S d ng m t s b t ng th c quen thu c
► B t ng th c Cauchy
I. Ki n th c c n n m:
• Cho hai s a, b ≥ 0, ta coù:
ab
ba
≥
+
2
D u “ =” x y ra khi ⇔ a = b
• Cho n s a1, a2, … , an ≥ 0, ta có:
n
n
n
aaa
n
aaa
...
...
21
21
≥
+++
D u “=” x y ra ⇔ a1 = a2 = … = an
II. M t s bài t p ví d :
◦ Bi n pháp 1: Áp d ng b t ng th c tr c ti p.
Ví d 1:
Cho x > 0 ; y > 0 tho mãn i u ki n
2
111
=+
yx
. Tìm giá tr nh nh t c a bi u
th c A = yx +
L i gi i:
3)Tìm nghi m c a h
6
Vì x > 0 ; y > 0 nên
x
1
> 0 ;
y
1
> 0 ; 0;0 >> yx , theo b t Cauchy có:






+≤
yxyx
11
2
11
.
1
=> 4
4
11
≥=>≤ xy
xy
V n d ng b t Cauchy v i hai s dương x và y ta ư c
A = yx + ≥ 42.2 ≥yx = 4 ( D u “=” x y ra ⇔ x = y = 4)
V y min A = 4 ( khi và ch khi x = y = 4).
Nh n xét: không ph i lúc nào ta cũng có th dùng tr c ti p b t Cauchy i v i
các s trong bài. Dư i ây ta s nghiên c u m t s bi n pháp bi n i m t bi u
th c có th v n d ng b t Cauchy r i tìm c c tr c a nó.
Bi n pháp 1 : tìm c c tr c a m t bi u th c ta tìm c c tr c a bình phương bi u
th c ó.
Ví d 2:
Tìm giá tr l n nh t c a bi u th c : A = .3753 xx −+−
L i gi i:
KX : .
3
7
3
5
≤≤ x
A2
= (3x – 5) + (7- 3x) + )37).(53(2 xx −−
A2
≤ 2 + ( 3x – 5 + 7 – 3x) = 4 ( d u “=” x y ra ⇔ 3x – 5 = 7 – 3x ⇔ x = 2).
V y max A2
= 4 => max A = 2 ( khi và ch khi x = 2).
Nh n xét: Bi u th c A ư c cho dư i d ng t ng c a hai căn th c. Hai bi u th c
l y căn có t ng không i (b ng 2). Vì v y, n u ta bình phương bi u th c A thì s
xu t hi n h ng t là hai l n tích c a căn th c. n ây có th v n d ng b t ng
th c Cauchy.
◦ Bi n pháp 2: Nhân và chia bi u th c v i cùng m t s khác 0.
Ví d 3:
Tìm giá tr l n nh t c a bi u th c A =
x
x
5
9−
L i gi i:
KX : x ≥ 9
7
A =
x
x
5
9−
=
30
1
10
3
99
5
3
3
9
2
1
5
3.
3
9
=
+−
=






+
−
≤
−
x
x
x
x
x
x
(d u “ =” x y ra khi và ch khi 183
3
9
=⇔=
−
x
x
).
V y max A =
30
1
( khi và ch khi x = 18).
Nh n xét: Trong cách gi i trên, x – 9 ư c bi u di n thành 3.
3
9−x
và khi vân
d ng b t Cauchy, tích 3.
3
9−x
ư c làm tr i tr thành t ng x
x
3
1
3
3
9
=+
−
có
d ng kx có th rút g n cho x m u, k t qu là m t h ng s . Con s 3 tìm ư c
b ng cách l y căn b c hai c a 9, s 9có trong bài.
Bi n pháp 3: Bi n i bi u th c ã cho thành t ng c a các bi u th c sao cho tích
c a chúng là m t h ng s .
1. Tách m t h ng t thành t ng c a nhi u h ng t b ng nhau.
Ví d 4 :
Cho x > 0, tìm giá tr nh nh t c a bi u th c : A = .
163
3
4
x
x +
L i gi i:
A = 3x + 4
333
16
....4
1616
x
xxx
x
xxx
x
≥+++=
A ≥ 4.2 = 8 ( d u “ =” x y ra khi và ch khi 2
16
3
=⇔= x
x
x
V y min A = 8 ( khi và ch khi x = 2).
Nh n xét: Hai s dương 3x và
x3
16
có tích không ph i là m t h ng s .Mu n kh
ư c x3
thì ph i có x3
= x.x.x do ó ta ph i bi u di n 3x = x + x + x r i dùng b t
Cauchy v i 4 s dương.
2. Tách m t h ng t ch a bi n thành t ng c a m t h ng s v i m t h ng t
ch a bi n sao cho h ng t này là ngh ch o c a h ng t khác có trong
bi u th c ã cho ( có th sai khác m t h ng s ).
Ví d 5:
Cho 0 < x < 2, tìm giá tr nh nh t c a bi u th c A = .
2
2
9
xx
x
+
−
8
L i gi i:
A = 1
2
2
9
+
−
+
− x
x
x
x
A 71921
2
.
2
9
.2 =+=+
−
−
≥
x
x
x
x
( d u “=” x y ra
2
12
2
9
=⇔
−
=
−
⇔ x
x
x
x
x
).
V y min A = 7 ( khi và ch khi
2
1
=x ).
◦ Bi n pháp 4: Thêm m t h ng t vào bi u th c ã cho.
Ví d 6:
Cho ba s dương x, y, z tho mãn i u ki n x + y + z = 2. Tìm giá tr nh nh t c a
bi u th c :
P = .
222
yx
z
xz
y
zy
x
+
+
+
+
+
L i gi i:
Áp d ng b t Cauchy i v i hai s dương
zy
x
+
2
và
4
zy +
ta ư c:
x
xzy
zy
xzy
zy
x
==
+
+
≥
+
+
+ 2
.2
4
..2
4
22
Tương t :
z
yx
yx
z
y
xz
xz
y
≥
+
+
+
≥
+
+
+
4
4
2
2
V y zyx
zyx
yx
z
xz
y
zy
x
++≥
++
+





+
+
+
+
+ 2
222
P ( ) 1
2
=
++
−++≥
zyx
zyx (d u “=” x y ra
3
2
===⇔ zyx ).
III. Bài t p t gi i:
1) Cho x + y = 15, tìm gía tr nh nh t, giá tr l n nh t c a bi u th c:
B = 34 −+− yx
2) Cho x, y, z ≥ 0 tho mãn i u ki n x + y + z = a.
Tìm giá tr l n nh t c a bi u th c A = xy + yz + xz.
Tìm giá tr nh nh t c a bi u th c B = x2
+ y2
+ z2
.
9
3) Cho x, y, z là các s dương tho mãn i u ki n x + y + z ≥ 12. Tìm giá tr
nh nh t c a bi u th c P = .
x
z
z
y
y
x
++
4) Cho a, b, c là các s dương tho mãn i u ki n a + b + c = 1. Tìm giá tr
nh nh t c a bi u th c A = .
)1)(1)(1(
)1)(1)(1(
cba
cba
−−−
+++
5) Cho x, y tho mãn i u ki n x + y = 1 và x > 0. Tìm giá tr l n nh t c a
bi u th c B = x2
y3
.
6) Tìm giá tr nh nh t c a
xy yz zx
A
z x y
= + + v i x, y, z là các s dương và:
a) 1x y z+ + = b) 2 2 2
1x y z+ + =
7) Tìm giá tr l n nh t c a 3 3 3 3 3 3
1 1 1
1 1 1
A
a b b c c a
= + +
+ + + + + +
v i a, b, c là
các s dương và abc = 1.
8)Tìm giá tr nh nh t, giá tr l n nh t c a
A x y z xy yz zx= + + + + + bi t r ng 2 2 2
3x y z+ + = .
9) Tìm giá tr nh nh t c a 3 3x y
A = + v i x + y = 4.
10) Tìm giá tr nh nh t c a 4
4 1A x x= − +
Hư ng d n gi i và áp s :
1.
KX : x ≥ 4, y ≥ 3
B ≥ ⇒8 min B = 8 ( khi và ch khi x = 4, y = 11 ho c x = 12, y = 3). max B2
=
16 nên max B = 4 ( khi và ch khi x = 8, y = 7).
2
.a. xy + yz + xz ≤ x2
+ y2
+ z2
(áp d ng b t Cauchy cho 2 s , r i c ng l i theo
v ).
Suy ra: 3(xy + yz + xz) ≤ ( x + y + z )2
Hay 3A ≤ a2
b. B = x2
+ y2
+ z2
= ( x + y + z )2
– 2( x + y + z )
B = a2
– 2A
B min ⇔ A max.
3.
P2
= .
222222
y
xz
x
zy
z
yx
x
z
z
y
y
x
+++++
Áp d ng b t Cauchy cho 4 s dương:
.4
...
44
222
x
yz
zyxx
z
z
yx
z
yx
y
x
=≥+++
Còn l i: tương t
C ng v v i v l i, ta ư c P2
≥ 4(x + y + z) – (x + y + z) = 3(x + y + z)
10
P2
≥ 3.12 = 36
Min P = 6.( khi và ch khi x = y = z = 4).
4.
a + b + c = 1 ⇒ 1 – a = b + c > 0. Tương t 1 – b > 0, 1 – c > 0.
Có: 1 + a = 1 + (1 – b – c) = (1 – b) + (1 – c) ≥ ( )( )cb −− 112
Suy ra (1 + a)(1 + b)(1 + c) ≥ ( ) ( ) ( )222
1118 cba −−−
A ≥ 8
V y min A = 8.
5. N u y ≤ 0 thì B ≤ 0.
N u y > 0 thì
1 = x + y =
3125
108
108
5
33322
325
32
≤⇒≥++++ yx
yxyyyxx
hay B ≤
3125
108
Suy ra max B =
3125
108
.
6.
Theo b t ng th c Cô-si
2. . 2
xy yz xy yz
y
z x z x
+ ≥ = tương t 2
yz zx
z
x y
+ ≥ ; 2
zx xy
x
y z
+ ≥
Suy ra 2A ≥ 2(x+y+z) = 2 ; min A = 1 v i
1
3
x y z= = =
b) Ta có
2 2 2 2 2 2
2
2 2 2
2
x y y z z x
A
z x y
= + + +
Hãy ch ng t 2
3A ≥ .
Min A = 3 v i x = y = z =
3
3
.
7.
D ch ng minh ( )3 3
a b ab a b+ ≥ + v i a > 0, b > 0. Do ó:
( )3 3
1 ( ).a b ab a b abc ab a b c+ + ≥ + + = + +
1 1 1
1
( ) ( ) ( ) ( )
max 1 1
a b c
A
ab a b c bc a b c ca a b c abc a b c
A a b c
+ +
≤ + + = =
+ + + + + + + +
= ⇔ = = =
8.
◦ Tìm giá tr l n nh t:
Áp d ng b t ng th c ( ) ( )2 2 2 2
3x y z x y z+ + ≤ + + ,ta ư c ( )
2
9x y z+ + ≤ nên
11
3x y z+ + ≤ (1)
Ta có b t ng th c 2 2 2
xy yz zx x y z+ + ≤ + + mà 2 2 2
3x y z+ + ≤ nên
3xy yz zx+ + ≤ (2)
T (1) và (2) suy ra 6A ≤ . Ta có max 6 1A x y z= ⇔ = = = .
◦ Tìm giá tr nh nh t : t x + y + z = m thì
( ) ( )2 2 2 2
2 3 2m x y z xy yz zx xy yz zx= + + + + + = + + +
Do ó
2
3
2
m
xy yz xz
−
+ + = . Ta có
2
3
2
m
A m
−
= + nên
( )
22
2 2 3 1 4 4.
2.
A m m m
A
= + − = + − ≥ −
⇒ ≥ −
2 2 2
1
min 2
3
x y z
A
x y z
+ + =
= − ⇔ 
+ + =
, ch ng h n x = -1, y = -1, z = 1.
9.
4
3 3 2 3 3 2 3 2 3x y x y x y
A +
= + ≥ = =
10.
Ta có x x≤ (x y ra d u b ng khi và ch khi 0x ≥ ) nên 4 4 .x x− ≥ − Do ó
4
4 1A x x≥ − + .
Áp d ng b t ng th c côsi v i b n s không âm
4 4 44
1 1 1 4 4 4 1 2.x x x x x+ + + ≥ = ⇒ − + ≥ −
4
min 2 1A x= − ⇔ = và 0 1x x≥ ⇔ = .
► B t ng th c Bunhiacopski:
I. Ki n th c c n n m:
• Cho a, b, c, d tuỳ ý, ta có
(a2
+ b2
)(c2
+ d2
) ≥ (ac + bd)2
D u b ng x y ra khi: ad = bc.
• Cho a1, … , an và b1, … , bn tuỳ ý, ta có:
(a1
2
+ … + an
2
)(b1
2
+ … + bn
2
) ≥ ( a1b1 + … + anbn)2
D u b ng x y ra khi:
n
n
b
a
b
a
== ...
1
1
II. M t s bài t p ví d :
Ví d 1:
Tìm giá tr l n nh t c a : P = xx −+− 5413
L i gi i:
KX : 1 ≤ x ≤ 5
Áp d ng b t Bunhiacopski có:
12
P2
≤ ( 32
+ 42
)(x – 1 + 5 – x) = 100
Suy ra max P = 10 khi ⇔
−
=
−
4
5
3
1 xx
x =
25
61
.
Ví d 2:
Cho a, b, c > 0. Tìm min P =
ba
c
ac
b
cb
a
+
+
+
+
+
345
.
L i gi i:
P =
( ) ( ) )345(
345
3453
3
4
4
5
5
++−





+
+
+
+
+
++=++−+
+
++
+
++
+ bacacb
cba
ba
c
ca
b
cb
a
= ( ) ( ) ( )[ ] )345(
345
.
2
1
++−





+
+
+
+
+
+++++
bacacb
accbba
≥ ( ) ( )345345
2
1 2
++−++ ( theo b t Bunhiacopski).
Vaäy min P = ( ) ( )345345
2
1 2
++−++ khi và ch khi
345
bacacb +
=
+
=
+
.
T ng quát:
Cho a, b, c > 0. Ch ng minh r ng:
( ) ( )222222
2
1
zyxxzyzxyz
ba
c
y
ca
b
x
cb
a
++−++≥
+
+
+
+
+
.
(c ng vào v trái (x2
+ y2
+z2
) r i tr i (x2
+ y2
+z2
), sau ó áp d ng b t
Bunhicopski).
Ví d 3:
Cho a, b, c > 0. Tìm min P =
ac
b
cb
bc
ba
ca
+
+
+
+
+
+
+ 433
L i gi i:
P = 106
4
2
3
2
3
−





+
+
+





+
+
+
+





+
+
+
ac
b
cb
ac
ba
ca
P = 10
664332323
−





+
++
+





+
++
+





+
++
ac
acb
cb
acb
ba
cba
P = ( ) 10
211
323 −





+
+
+
+
+
++
accbba
cba
P = ( ) ( ) ( )[ ] ( ) 6102.21110
211
.2
2
=−++≥−





+
+
+
+
+
+++++
accbba
cacbba
V y min P = 6 khi và ch khi (a + b)2
= (b + c)2
= (c + a)2
hay a = b = c.
Cơ s :
13
Ch n γβα ,, sao cho:
)323()(4)(3)(3 cbamacbcbacbaca ++=++=+++=+++ γβα .
T ó suy ra 2,6,2 ==== mγβα .
III. Bài t p t gi i:
1. Cho a, b, c > 0. Tìm giá tr nh nh t c a:
a) P =
ac
ba
cb
ba
ba
cb
+
+
+
+
+
+
+
+ 54893
.
b) Q =
ac
ba
cb
ba
ba
cb
+
+
+
+
+
+
+
+ 5243
.
c) R =
cba
c
cba
b
cba
ca
3
8
2
4
2
3
++
−
++
+
++
+
.
2. Tìm giá tr nh nh t, giá tr l n nh t c a 2 2
A x y= +
bi t r ng ( ) ( )
22 2 2 2
2 3 2 1.x x y y+ − + − =
3. Tìm giá tr nh nh t c a :
2 2 2
a b c
A
b c c a a b
= + +
+ + +
v i a, b, c là các s dương và a + b + c =6.
4. Tìm giá tr nh nh t c a
2 1
2
A
x x
= +
−
v i 0 < x < 2.
5. Cho a, b, c > 0 và abc = 1
Tìm giá tr nh nh t c a
( ) ( ) ( )3 3 3
1 1 1
A
a b c b a c c a b
= + +
+ + +
Hư ng d n gi và áp s :
1. Câu a và câu b làm tương t ví d 3
Câu c không th làm như ví d 3 ư c, ta làm như sau:
t a + 2b + c = x
a + b + 2c = y
a + b + 3c = z
t ó suy ra c = z – y; b = x + y – 2y; a = 5y – x – 3z.
khi ó R =
z
y
y
z
y
x
x
y
z
yz
y
yzx
x
xy 8
88
44
1
2888442
+−−++−=
−
+
−+
+
−
.
R i áp d ng b t ta tìm ư c min R.
2.
T gi thi t suy ra
( ) ( )
22 2 2 2 2
4 3 0.x y x y x+ − + + = − ≤
Do ó ( )( )2
4 3 0 1 3 0 1 3.A A A A A− + ≤ ⇔ − − ≤ ⇔ ≤ ≤
min 1 0, 1.
max 3 0, 3.
A x y
A x y
= ⇔ = = ±
= ⇔ = = ±
3.
14
Áp d ng b t ng th c Bunhiacópki cho 3 c p s
Ta có
( ) ( ) ( )
2 2 2
2 2 2a b c
b c a c a b
b c a c a b
        + + + + + + +         + + +       
( ) ( )
2
2 2 2
2
2 2 2
2
.
2
a b c
b c a c a b
b c a c a b
a b c
a b c a b c
b c a c a b
a b c a b c
b c a c a b
 
≥ + + + + + 
+ + + 
 
⇒ + + + + ≥ + +   + + + 
+ +
⇒ + + ≥
+ + +
Suy ra min A = 3.
4.
Áp d ng b t ng th c Bunhiacopski
( )( ) ( )
22 2 2 2
a b m n am bn+ + ≥ +
Ta có:
( ) ( ) ( )
( )
( )
2 2 2
2 2
2
2 2
2 2
2 1 2 1
2 2 2
2 2
2 2 1 3 2 2.
2 1
2 12min 2 3 2 2 2 4 4
2 2
A x x x x
x x x x
A
x xA x x x
x x xx
        = + − + ≥ − +            − −       
⇒ ≥ + = +
−= + ⇔ = ⇔ = ⇔ = − +
− −
( )
22
4 4 8 2 8 2 2 2x x x x⇔ + + = ⇔ + = ⇔ = − (chú ý x > 0).
V y
3
min 2 2 2 2 2
2
A x= + ⇔ = − .
5.
t
1 1 1
, ,a b c
x y z
= = =
thì
, , 0
1
x y z
xyz
>

=
Khi ó
2 2 2
x y z
A
y z z x x y
= + +
+ + +
Áp d ng b t ng th c Bunhiacopski, bi n i tương ương ta ư c:
( )
( ) ( ) ( )
2
2
x y z x y z
A
y z z x x y
+ + + +
≥ =
+ + + + +
M t khác theo BDT côsi ta có: 33 3x y z xyz+ + ≥ =
V y
15
3
min
2
1
1 .
x y z
y z z x x y
A x y z
xyz
x y z a b c

= = + + +

= ⇔ = =
 =


⇔ = = = ⇔ = =
► B t ng th c Bernoulli
I. Ki n th c c n n m
)0,1(
1
>≥
+−≥
x
xx
α
ααα
(1)
D u “ =” x y ra khi x =1
II. M t s bài t p ví d :
Ví d 1:
Cho x, y > 0 sao cho x + y = 1. Tim giá tr nh nh t :
a. P = x2
+ y2
b. Q = x5
+ y5
L i gi i:
a.
Áp d ng b t Bernoulli ta có:
(2x)2
≥ 1 – 2 + 2(2x)
(2y)2
≥ 1 – 2 + 2(2y)
C ng v theo v :
4P ≥ -2 + 4(x + y) = 2
P ≥
2
1
.
V y min P =
2
1
khi và ch khi x = y =
2
1
.
b.
Áp d ng b t Bernoulli ta có:
(2x)5
≥ 1 – 5 + 5(2x)
(2y)5
≥ 1 – 5 + 5(2y)
C ng v theo v ta có:
32Q ≥ -8 + 10(x + y) = 2
Q ≥
16
1
V y min Q =
16
1
. Khi và ch x = y =
2
1
.
T ng quát:
S = xm
+ ym
, m ≥ 1 v i x + y = 1.
16
*. Theo (1), v i m i 0>≥ βα , ta có:
xx
β
α
β
αβ
α
+−≥ 1 (1’)
t xtxt =⇔= ββ
1
(1’) ⇔
D u “=” x y ra khi t = 1.
Ví d 2:
Cho x, y > 0, sao cho x3
+ y3
= 1. Tìm min P = 3
10
3
10
yx + .
L i gi i:
Theo (2), ta có:
( ) ( )
( ) ( )
( ) 2)(2.
9
10
9
2
2
2
9
10
9
10
12
2
9
10
9
10
12
333
10
3
3
33
10
3
3
33
10
3
=++−≥⇒
+−≥
+−≥
yxP
yy
xx
V y P ≥ 9
2
1
Hay min P = 9
2
1
khi và ch khi x = y = 3
2
1
*. T (2) thay t b i
0t
t
, ta ư c:
D u “=” x y ra khi t = t0 v i t0 là i m t giá tr nh nh t.
Bài toán:
Cho 0,,,;.(1.. >≥=+ dcbaybxa βαββ
)
Tìm min P = αα
ydxc .. +
βα
β
α
β
α
tt +−≥ 1 (2)
ββααα
β
α
β
α
tttt ..1 00
−
+





−≥ (3)
17
t
Yyd
Xxc
=
=
α
α
Bài toán tr thành : Cho ββ
ynxm .. + = p (m,n > 0)
Tìm min A = αα
yx +
L i gi i:
Theo b t (3), ta có:
ββααα
ββααα
β
α
β
α
β
α
β
α
yyyy
xxxx
.1
.1
00
00
−
−
+





−≥
+





−≥
C ng l i : A ≥ ( ) ( )...1 0000
ββαββααα
β
α
β
α
yyxxyx −−
+++





−
Ch n (x0 , y0) tho mãn:
ββ
ynxm .. + = p
n
y
m
x βαβα −−
= 00
.
Khi ó: A ≥ ( ) ..1 0
00
m
x
yx
βα
αα
β
α
β
α −
++





− p.
V y min A = ( ) ..1 0
00
m
x
yx
βα
αα
β
α
β
α −
++





− p khi và ch khi x = x0, y = y0.
▼ D ng 4: Áp d ng b t ng th c trong tam giác và phuơng pháp t a , vectơ.
I. Phương pháp gi i:
V i 3 i m A, B, C, b t kì trong m t ph ng ta có: AB BC AC+ ≥ ( ng
th c khi B n m gi a A và C).
• V i hai véc tơ b t kì a và b ta có:
a b a b± ≤ + . ng th c khi a và b cùng hư ng ( )1
• N u ( )1 2,a a a= và ( )1 2b b b= +
( )1 ⇔ ( ) ( )
2 2 2 2 2 2
1 1 2 2 1 2 1 2a b a b a a b b± + ± ≤ + + +
18
CB
A
ng th c x y ra khi 1 1
2 2
.
.
a k b
a k b
=

=
( )k R∈
D ng toán tìm giá tr l n nh t c a hàm s :
( ) ( )2 2 2 2
y f x a g x b= + + + v i
( ) ( ) ( )
, 0a b
f x g x k k R
≠

± = ∈
S d ng b t ng th c tam giác: gi s ( ) ( )f x g x k− = .
Trong m t ph ng Oxy xét i m: ( )( ) ( )2 2
,M f x a OM f x a⇒ = + và
2 2
( ( ), ) ( )N g x b ON g x b− ⇒ = + .
Ta có: ( )
22 2 2
( ) ( ) ( )MN f x g x a b k a b= − + + = + + .
Vì 2 2
( )OM ON MN y k a b+ ≥ ⇔ ≥ + + .
ng th c x y ra khi M, N, O th ng hàng . ( ) . ( ) 0a f x b g x⇔ + = .
V y Min 2 2
( )y k a b= + + .
II. M t s bài t p ví d :
Ví d 1:
Tìm giá tr nh nh t c a bi u th c 2 2
1 1, .A a a a a a R= + + + − + ⊥ ∀ ∈
L i gi i:
D th y bi u th c không thay i khi thay a b i a− , do ó ch c n gi i v i 0a ≥ .
• Khi 0a = : 2A = .
• Khi 0a > : Xét ABC∆ có:
1
2
3
AB
AM MB
CM a
AMC
π

= = =

=

 =

Theo nh lí hàm côsi:
2 2 2
1 2.1. .cos 1 .
3
AC a a a a
π
= + − = + −
2
1.AC a a⇒ = − +
Tương t 2
1BC a a= + + , 2.AB =
Khi ó: 2 2
1 1 2 2.AC BC AB a a a a A+ ≥ ⇒ + + + − + ≥ ⇔ ≥
ng th c x y x y ra khi 0a = . V y 2MinA = khi 0.a =
Ví d 2:
Tìm giá tr nh nh t c a: 2 2 2 2
2 2 2 2 .y x px p x qx q= − + + − +
L i gi i:
3
πM
19
Ta có: 2 2 2 2
( ) ( ) .y x p p x q q= − + − +
Xét i m ( , ); ( , ).M x p p N x q q− −
Ta có: 2 2
( ) ( ) .MN p q p q= − + +
Vì 2 2
( ) ( ) .OM ON MN y p q p q+ ≥ ⇔ ≥ − + +
Min⇒ 2 2
( ) ( ) .y p q p q= − + +
Khi , ,M N O th ng hàng ( ) ( ) 0 .
p q q p
q x p q x q x
p q
+
⇔ − + − = ⇔ =
+
Ví d 3:
Tìm giá tr nh nh t c a: 2 2
cos 2.cos 5 cos 4.cos 8.y x x x x= − + + + +
L i gi i:
Trong m t ph ng Oxy , xét i m
(2;1 cos ); (4,3)M x N−
Ta có: (2,2 cos )MN x= + như v y .y OM MN= +
Do 0 1 cos 2x≤ − ≤ nên [ ]M AB∈ v i (2,0)A và
(2,2)B .
Ta có: 2 2
4 3 5.OM MN ON+ ≥ = + =
ng th c x y ra khi , ,O M N th ng hàng
6 4.(1 cos ) 0x⇔ − − =
1 2
cos 2 .
2 3
x x k
π
π⇔ = − ⇔ = ± +
V y Min 5y = khi
2
2 .
3
x k
π
π= ± +
Ví d 4:
Cho 3 s th c a, b, c tho mãn h sau
2 2
2
1
2 ( ) 6
a c
b b a c
 + =

+ + =
( )
( )
1
2
Tìm giá tr nh nh t c a ( ).M b c a= −
L i gi i:
T gi thi t ta có: 2 2 2
2 2 2 2 8a c b ab bc+ + + + =
2 2
( ) ( ) 4
2 2
b b
a c⇔ + + + =
Do ( )
2 2
(1) 2 ( 2 ) 4c a⇔ + − =
Xét ( ; ); (2 ; 2 )
2 2
b b
x a c y c a+ + −
20
Ta có: 2x = , 2y = , . .( ).x y b c a= −
Mà . .x y x y≤ cùng hư ng:
2 2 2 2
2
.( ) 2
2 2 .( ) 2.( ) 1
2 2
10
b b b a ca c
b a c a c a c
c c
b
+ = −+ +

⇔ = ⇔ + = − + ⇒ + =
−  =
(do (1) và (2) )
2 2
2 2
10
2
10
1 3 1 3 1
( , , ) ( , 10, );( , 10, )
10 10 10 1010
2
10
1
b
a c
a c
a b c
b
a c
a c
 =

 + = −

 + =
⇔ ⇒ = − − −
 = −

 + =

 + =
Max⇒ ( ) 4M b c a= − = khi ( , , )a b c như trên.
III. Bài t p t gi i:
1)Tìm giá tr l n nh t và giá tr nh nh t c a hàm s
4 4 2
1 sin cos 2cos 2y x x x= + + + +
2)Tìm giá tr nh nh t c a hàm s : 2 2
1 3 1y x x x x= − + + − +
3)Tìm giá tr l n nh t và giá tr nh nh t c a hàm s :
4 4 2 2
4 4 2 2
2 1
x y x y
y
y x y x
   
= + − + −   
   
4)Tìm giá tr nh nh t c a hàm s :
( ) ( ) ( )2 2 2
2 2 1 2 3 1 1 2 3 1 1f x x x x x x x= − + + + + + + − − +
Hư ng d n gi và áp s :
1.
Ta có: ( ) ( )
2 22 2
1 1 cos 1 1 cosy x x= + − + + +
x
y
O
1
1
2
B
N
2
A
M
21
Xét i m M(1, 1-cos2
x), N(2, 2) ta có: ( )2
1,1 cosMN x= +
Ta có: y = OM + MN
V i M thu c o n [AB] v i A(1, 0) và B(1, 1)
Ta có miny = ON = 2 2
2 2 2 2+ =
D u “=” x y ra khi O, M, N th ng hàng
( )2
2 2 1 cos 0x− − = 2
cos 0x⇔ =
Và maxy = OA + AN = 1+ 2
1 2 1 5+ = +
D u “=” x y ra khi M trùng v i A 2
cos 1x⇔ =
2.
Ta có:
22
1 3 3 1
2 4 2 4
y x x
  
= − + + − +       
Xét i m
1 3
,
2 2
M x
 
−  
 
và
3 1
,
2 2
N x
 
− −  
 
Hai i m M, N n m hai bên Ox. Ta có: y = OM + ON ≥ MN
2 2
3 1 3 1
min
2 2 2 2
3 1
min 2 2
4 4
y MN
y
   
= = − + + +      
   
 
= + = 
 
D u “=” x y ra khi M, O, N th ng hàng:
( )
1 1 3 3
0
2 2 2 2
3 1 2 3 1
x x
x x
  
− − − − =       
⇔ + = ⇔ = −
3.
4 4 2 2
4 4 2 2
2 1
x y x y
y
y x y x
   
= + − + −   
   
ch n
2 2
2 2
x y
u
y x
= + [ )2;u ∈ +∞
hàm s y(x) tr thành
f(u) = u2
– 2u – 3
phác h a th hàm f(u) trong mi n [ )2;+∞ ta thu
ư c k t qu :
max f(u) không t n t i
min f(u) = f(2) = -3
O
( )f u
u
M
2
3−
22
C
'
C
M
'
MA
B
C
'
C
M
'
M
V y: max y(x) không t n t i
min y(x) = -3 t ư c khi
2 2
2 2
2
x y
y x
+ =
2
2
1
x
y
⇒ = ⇒ m i i m (x; y) thu c 2 ư ng phân giác y = x và y = -x
(tr g c O(0; 0))
4.
Hàm s f(x) có th vi t l i dư i d ng:
( ) ( )
2 22 2
22 3 1 3 1
1
2 2 2 2
f x x x x x x x
      
= + − + + + + + − + +               
(1)
Xét trên m t ph ng t a các i m
( )
3 1 3 1
0,1 , , , ,
2 2 2 2
A B C
   
− − −      
   
Và i m M(x,x) n m trên ư ng phân giác th nh t.
D th y ABC là tam giác u, v i tâm là g c t a . Theo công th tính
kho ng cách gi a hai i m trên m t ph ng t a , ta có v ph i c a (1) chính là
MA + MB + MC.
B : N u ABC là tam giác u, thì v i m i i m M c a m t ph ng tam giác, ta
luôn có MA + MB + MC ≥ OA + OB + OC, trong ó O là tâm tam giác u.
Ch ng minh:
N u M là i m trong tam giác. Xét phép quay R(A,600
), khi ó
'
'
M M
B C
C C
A A
→
→
→
→
⇒ MC = M’C’, MA = MM’
V y MA + MB + MC = MM’
+ MB + M’
C’
≥ BC’
M t khác n u g i O là tâm tam giác u ABC thì
OA + OB + OC = BC’
⇒ MA + MB + MC ≥ OA + OB + OC
23
N u M ngoài tam giác, ch ng minh tương t .
Theo b ta có
f(x) ≥ 3 (do OA = OB = OC)
V y min f(x) = 3.
▼ Dang 5 :Phương pháp s d ng th hàm s :
I. Phuơng pháp gi i:
Phương pháp này thư ng dùng tìm c c tr c a các hàm s sau:
2. Các hàm s qui v tam th c b c hai.
3. Các hàm s ch a d u giá tr tuy t i.
4. Các bài toán chuy n ư c thành toán hình h c b ng cách dùng công th c
dài o n th ng: 2 2
( ) ( )A B A BAB x x y y= − + − .
ây là các bài toán mà trong ó ( )f x cho dư i d ng căn b c hai mà làm
dư i căn bi u di n ư c thành dài m t o n th ng nào ó. ây là ưu
th c a phương pháp th .
5. Các hàm s ( , )u x y v i ,x y tho mãn trư c i u ki n.
II. M t s bài t p ví d :
Ví d 1:
Tìm giá tr l n nh t và giá tr nh nh t c a các hàm s sau:
21 1
) ( ) ( ) 3.( ) 1.
3
1
) ( ) .(2 sin ).( sin ).
15
x x
a y x
x
b y x x b x
+ +
= + +
= + −
L i gi i:
a) t
1 1
2.
x
u x
x x
+
= = + ≥
Hàm ( )y x tr thành: 2
( ) 3 1.y u u u= + +
Theo th hàm ( )y u trên [2; ).+∞
Ta ư c max ( )y u = không có.
miny(u)=y(2)=11.
b) t sin 1 1.u x u= ⇒ − ≤ ≤
Hàm ( )y x thành
1
( ) .(2 ).( ).
15
y u u b u= + −
D a vào th ta có k t qu :
max ( ) (1) 1
7
min ( ) ( 1)
15
y u y
y u y
= =
= − =
24
Khi 2 1.
1 0
1.
x
x
x
≥
− ≥ ⇔  ≤ −
Khi 2
1 0 1 1.x x− ≤ ⇔ − ≤ ≤
Khi 1x ≤ − ho c 1.x ≥
Khi 1 1.x− ≤ ≤
Khi 1x ≥ ho c 1.x ≤ −
Khi 1 1.x− ≤ ≤
Khi 3x ≤ − ho c 1.x ≥ −
Khi 3 1.x− ≤ ≤ −
Khi 4 3.x− ≤ ≤ −
Khi 4 3.x− ≤ ≤ −
Ví d 2:
Tìm giá tr nh nh t c a: 2 2
( 1) .y x x= + −
L i gi i:
Tao có 2
1.y x x= + −
G i 2
1 1y x x= + − và 2
2 1y x x= − + +
thì 1
2
y
y
y

= 

V th c a y ta th y min ( 1) 1.y y= − = −
Ví d 3:
Tìm giá tr l n nh t và giá tr nh nh t c a: ( )( )2 2
6 9 2 9y x x x x x= + + + + + trong
5
[ 4; ]
4
− − .
L i gi i:
Ta có: ( 3)( 1).y x x x= + + +
Do
( 3).( 1)
( 3).( 1)
x x
x x
+ +
− + +
Ta ch xét nh ng giá tr c a
5
4; .
4
n
 
∈ − −  
Ta ư c
2
2
5 3
3 3
x x
y
x x
 + +
= 
− − −
V th 2
1 5 3y x x= + +
2
2 3 3y x x= − − −
D a vào th :
2
1
3 3
max ( ) .
2 4
min ( 3) 3.
y y
y y
= − = −
= − = −
2
2
1
1
x x
y
x x
 + −

⇒ = 
 + −
2
2
1
1
x x
y
x x
 + −
⇒ = 
− + +
Khi 4 3.x− ≤ ≤ −
Khi
5
3 .
4
x− ≤ ≤ −
Y2
Y1
25
III. Bài t p tương t :
1.Tìm giá tr l n nh t và giá tr nh nh t c a 1 3 2 2y x x x= − + − − + v i
2 4.x− ≤ ≤
2.Tìm giá tr nh nh t c a: 2 2
4 12 13 4 28 53.y x x x x= − + + − +
3.Tìm giá tr l n nh t và nh nh t c a hàm s : 2 5u x y= − + + bi t ,x y tho :
2 2
36 16 9.x y+ =
Hương d n và áp s :
1.
V i 2 1x− ≤ ≤ − thì (1 ) (3 ) (2 2) 6.y x x x= − + − − − =
V i 1 1x− ≤ ≤ thì 4 2.y x= − +
V i 1 3x≤ ≤ thì 2 .y x= −
V i 3 4x≤ ≤ thì 6.y = −
Ta v th c a hàm s 1 3 2 2y x x x= − + − − + v i 2 4.x− ≤ ≤
T th max 6 2 1.y x= ⇔ − ≤ ≤ −
min 6 3 4.y x= − ⇔ ≤ ≤
2.
Ta có: 2 2 2 2
(2 3) (0 2) (2 7) (0 2) .y x x= − + − + − + −
Trên m t ph ng to Oxy , xét
(2 ,0)M x , (3,2)A , (7,2)B
như v y .y MA MB= +
,A B n m cùng phía so v i Ox nên l y
'A i x ng A qua .Ox
'A B c t Ox t i H ta có: '(3, 2)A −
2 2
' ' (7 3) (2 2) 4 2.y MA MB MA MB A B= + = + ≥ = − + + =
ng th c x y ra khi
5
2 5 .
2
M H x x≡ ⇔ = ⇔ =
V y min 4 2y = khi
5
.
2
x =
26
3.
T i u ki n 2 2 2 2 2
36 16 9 (6 ) (4 ) 3 .x y x x+ = ⇔ + =
t
1
6 6
4 1
4
x X
x X
y Y
y Y

== 
⇔ 
=  =

Ta có 2 2 2
3 .X Y+ = (1)
(1) là phương trình ư ng tròn ( )» trong h tr c to Oxy có tâm O bán kính
3.R =
Hàm
1 1
2 5 5.
3 4
u x y u X Y= − + + ⇔ = − + +
4
4( 5).
3
Y X u⇔ = + − ta g i phương trình này là phương trình ư ng th ng
d ư ng th ng luôn song song v i ư ng th ng
4
3
Y X= và c t Oy t i
(0;4( 5)).P u −
-Ta v hai ư ng th ng 1 2,Y Y song song v i ư ng th ng
4
3
Y X= và ti p xúc
( )» .
- 1 2,Y Y c t Oy l n lư t t i N và M khi ó maxu là giá tr xác nh khi P N≡ hay
4( 5)m naxu= − trong ó (0; ).M m
Min u xác nh khi P N≡ t c là 4(min 5)n u= − trong ó (0; )N m do M i
x ng N qua O nên .m u= −
-K 1OH Y⊥ l y 1OH Y⊥ (3;0), (3;4),A B OAB OHM∆ = ∆ 5.m OM OB⇒ = = =
Khi ó
25
max .
5 4(max 5) 4
5 4(min 5) 15
min .
4
u
u
u
u

== − 
⇔ 
− = −  =

27
Ph n 2: C C TR TRONG HÌNH H C PH NG
▼ D ng 1: V n d ng quan h gi a ư ng xiên và ư ng vuông góc, quan h gi a ư ng
xiên và hình chi u.
I. Ki n th c c n nh :
II. M t s bài t p ví d :
Ví d 1:
Cho tam giác ( 90 ).o
ABC A = M là i m chuy n ng trên c nh BC . V
, ( , ).MD AB ME AC D AB E AC⊥ ⊥ ∈ ∈ Xác nh v trí c a i m M o n th ng DE có dài
nh nh t.
L i gi i:
V ( )AH BC H BC⊥ ∈ H c nh và AH không i.
T giác AEMD có 90o
A E D= = = nên AEMD là hình
ch nh t.
Suy ra DE AM= mà AM AH≥ (không i)
D u " "= x y ra .M H⇔ ≡
Ví d 2:
Cho tam giác .ABC Qua nh A c a tam giác hãy d ng
ư ng th ng d c t c nh BC sao cho t ng các kho ng cách t B và t C n d có giá tr nh
nh t.
L i gi i:
G i M là giao i m c a d và c nh .BC
V , ( ; )BH d CK D H K d⊥ ⊥ ∈
MAB MAC ABCS S S+ =
. .
2 2
ABC
BH AM CK AM
S+ =
2 ABCS
BH CK
AM
+ =
BH CK+ nh nh t
2 ABCS
AM
⇔ nh nh t AM⇔ nh nh t
Gi s AB AC≤ thì trong hai ư ng xiên ,AM AC ư ng xiên AC có hình chi u không nh
hơn, do ó AM AC≤ (h ng s )
D u " "= x y ra .M C⇔ ≡
Ví d 3:
Ta có AH d⊥ , A d∉ , B d∈ ,C d∈ , H d∈ .
a) .AB AH≥ D u " "= x y ra khi .B H⇔ ≡
b) .AB AC BH HC≤ ⇒ ≤
28
Cho hình bình hành ABCD .Qua A v ư ng th ng d không c t hình bình hành. G i ', ', 'B C D
l n lư t là hình chi u vuông góc c a các i m , ,B C D trên ư ng th ng d .
Xác nh v trí c a ư ng th ng d t ng ' ' 'BB CC DD+ + có giá tr l n nh t.
L i gi i:
G i O là giao i m c a AC và BD .
'O là hình chi u vuông góc c a O trên d .
' , 'DD d BB d⊥ ⊥
' 'DD BB⇒
' 'DD BB⇒ là hình thang.
Mà ' , 'OO d DD d⊥ ⊥
' 'OO DD⇒ và O là trung i m BD ( ABCD là
hình bình hành).
Do ó 'OO là ư ng trung bình c a hình thang
' 'DD B B
' '
' ' ' 2. '
2
BB DD
OO BB DD OO
+
⇒ = ⇒ + = .
' , ' ' 'OO d CC d OO CC⊥ ⊥ ⇒ và O là trung i m AC .( ABCD là hình bình hành).
Do ó 'OO là ư ng trung bình c a 'ACC
'
' ' 2. '
2
CC
OO CC OO⇒ = ⇒ =
A d∈ và 'OO d⊥ nên 'OO OA≤
Do ó ' ' ' 4. ' 4.BB CC DD OO OA+ + = ≤ (không i)
D u " "= x y ra "O A d⇔ ≡ ⇔ vuông góc AC t i A .
Ví d 4:
Cho n a ư ng tròn ( ; )O R ư ng kính .AB M là i m trên n a ư ng tròn. Xác nh v trí M
:
a) Di n tích tam giác MAB l n nh t.
b) Chu vi tam giác MAB l n nh t.
L i gi i:
V , .MH AB H AB⊥ ∈
a)
.
2
MAB
MH AB
S =
.MH R=
Ta có , .MH AB O AB⊥ ∈
Do ó MH OM R≤ =
Nên 2
MABS R≤ (không i)
D u " "= x y ra H O M⇔ ≡ ⇔ là trung i m AB
b) 90o
AMB = ( AMB là góc n i ti p ch n n a ư ng tròn)
MAB vuông t i M có . .MH AB MH AB MA MB⊥ ⇒ =
MAB vuông t i M theo nh lí Pitago có:
2 2 2 2
4 .MA MB AB R+ = =
,MABP MA MB AB= + + AB không i
2 2 2
( ) 2 .MA MB MA MB MA MB+ = + +
29
Do ó MABP l n nh t MA MB⇔ + l n nh t
2
( )MA MB⇔ + l n nh t .MA MB⇔ l n nh t
MABS⇔ l n nh t M⇔ là trung i m AB (câu a)
Ví d 5:
Cho n a ư ng tròn ( )O ư ng kính 2 .AB R= K hai ti p tuy n Ax, By c a n a ư ng tròn
( )O và ti p xúc v i ( )O t i i m M c t Ax t i D c t By t i E. Xác nh v trí c a M trên n a
ư ng tròn ( )O sao cho:
a) AD BE+ t giá tr nh nh t.
b) .OD OE t giá tr nh nh t.
L i gi i:
a) V , .DH By H By⊥ ∈
T giác ADHB có 90O
A B H= = = nên ADHB là hình ch nh t 2DH AB R⇒ = =
Ta có ,AD MD BE ME= = (tính ch t hai ti p tuy n c a ( )○ c t nhau t i m t i m).
Do ó AD BE MD ME DE+ = + = mà DE DH≥ (vì ,DH By E By⊥ ∈ )
Do v y 2AD BE R+ ≥ (không i)
D u " "= x y ra E H DE AB⇔ ≡ ⇔
OM AB M⇔ ⊥ ⇔ là trung i m AB .
b) DA và DM là ti p tuy n c a( )○ OD⇒ là phân giác AOM .
Tương t OE là phân giác MOB .
AOM và MOB k bù.
Do ó 90o
EOD =
ODE vuông t i O , OM DE⊥ nên
. .OD OE OM DE=
. .OD OE R DE=
.OD OE nh nh t DE⇔ nh nh t M⇔ là trung i m AB (câu a).
▼ Dang 2: V n d ng các b t ng th c trong tam giác và quy t c các i m :
I. Ki n th c c n n m:
• Tam giác ABC có
a) .AB AC BC AB AC− < < +
b) .ABC ACB AC AB≤ ⇔ ≤
• Tam giác ABC và tam giác ' ' 'A B C có
' ', ' 'AB A B AC A C= = thì: ' ' '.BC B C A A≤ ⇔ ≤
• Quy t c ba i m , ,A B C .
a) .BC AB AC≤ +
D u" "= x y ra [ ]A BC⇔ ∈
b) .BC AB AC≥ −
D u" "= x y ra , ,A B C⇔ th ng hàng.
30
Quy t c n i m 1 2; ;...; nA A A
Ta có 1 1 2 2 3 3 4 1...n n nA A A A A A A A A A−≤ + + + +
D u " "= x y ra 1 2 1; ;...; ;n nA A A A−⇔ th ng hàng và s p x p theo th t ó.
II. M t s bài t p ví d :
Ví d 1:
Cho hai i m A và B n m trong n a m t ph ng b là ư ng th ng d ,hai i m ,M N thu c d và
dài MN không i. Xác nh v trí hai i m ,M N ư ng g p khúc AMNB t giá tr nh
nh t.
L i gi i
D ng hình bình hành 'BNMB (hình bên) 'BB MN a⇒ = = (không i); ', 'NB MB B= c nh.
G i 'A là i m i x ng c a A qua ư ng th ng d .
Ta có 'AM A M= , 'A c nh.
Xét ba i m ', , 'A M B ta có ' ' ' 'A M MB A B+ ≥
Do ó ' 'AM MN NB A M MN MB+ + = + +
( ' ')A M MB MN= + +
' 'A B a≥ = không i
D u " "= x y ra [ ' '].M A B⇔ ∈
Ví d 2:
Cho góc nh n xOy . A là i m n m trong góc ó. Hãy tìm trên hai tiaOx và Oy l n lư t hai
i m B vàC sao cho chu vi tam giác ABC nh nh t.
L i gi i:
G i 1A và 2A l n lư t là i m i x ng c a A qua hai tiaOx vàOy .
A c nh, xOy c nh nên 1A và 2A c nh.
Theo tính ch t i x ng tr c ta có:
1 ;AB A B= 2 .AC A C=
ABCP AB BC AC= + +
1 2A B BC A C= + +
Xét các i m 1 2, , ,A B C A ta có 1 2 1 2A B BC A C A A+ + ≥
Do ó 1 2ABCP A A≥ (không i)
D u " "= x y ra 1 2, , ,A B C A⇔ th ng hàng và s p x p theo th t ó.
Ví d 3:
Cho hình vuông ABCD . , , ,M N P Q là nh c a t giác MNPQ l n lư t thu c các
c nh , , ,AB BC CD DA ( MNPQ g i là t giác n i ti p hình vuông). Tìm i u ki n t giác MNPQ có
chu vi nh nh t.
L i gi i:
G i , ,E F G l n lư t là trunh i m c a các o n th ng
, , .MQ MP NP
AMQ vuông góc t i A có
AE là trung i m nên
1
2
AE MQ= 2 .MQ AE⇒ =
31
Tương t 2NP GC=
M t khác ,EF FG l n lư t là ư ng trung bình
c a các tam giác MPQ và NPM
nên
1
2
EF PQ= và
1
2
FG MN=
Suy ra 2PQ EF= và 2 .MN FG=
Do ó MNPQP MN NP PQ QM= + + +
2 2 2 2FG GC EF AE= + + +
2( )AE EF FG GC AC= + + + ≥ (không i )
(Xét các i m , , , ,A E F G C )
D u " "= x y ra , , , ,A E F G C⇔ th ng hàng.
MN AC PQ⇔ và .MQ BD NP
Khi ó MNPQ là hình ch nh t.
Ví d 4:
Cho ư ng tròn( ; )O R ư ng kính AB c nh,C là m t i m c nh n m gi a A và .O M di ng
trên ư ng tròn( ; ).O R Tìm v trí c a M trên ( ; )O R tương ng lúc dài CM l n nh t, nh nh t.
L i gi i:
Xét ba i m , ,C O M ta có OM CO CM CO OM− ≤ ≤ +
OA OM OB R= = =
Do ó CA CM CB≤ ≤
CM CB≤ (không i)
D u " "= x y ra M B⇔ ≡
V y khi M B≡ thì o n th ngCM có dài l n nh t.
M t khácCM CA≥ (không i)
D u " "= x y ra M A⇔ ≡
V y khi M A≡ thì o n th ng CM có dài nh nh t.
Ví d 5:
Cho hai ư ng tròn ngoài nhau( ; )O R và( '; ').O R A n m trên ư ng tròn( )O , B n m trên ư ng
tròn( ').O Xác nh v trí các i m ,A B o n th ng AB có dài l n nh t, nh nh t.
L i gi i:
( ')OO c t( )O t i ,C D và c t( ')O t i , .E F
Xét ba i m , ',A O B , ta có
' ' ' 'O A O B AB O A O B− ≤ ≤ +
Xét ba i m , , 'O A O , ta có
' ' 'O O OA O B OA OO− ≤ ≤ +
Mà OA OC OD R= = = và
' ' ' 'O B O E O F R= = =
Do ó ' ' ' 'OO OD O E AB OC OO O F− − ≤ ≤ + +
DE AB EF⇒ ≤ ≤
* AB EF≤ (không i)
D u " "= x y ra ,A C⇔ ≡ B F≡
32
V y AB có dài l n nh t khi A C≡ và B F≡
* AB DE≥ (không i)
D u " "= x y ra A D⇔ ≡ và B E≡
V y AB có dài nh nh t khi A D≡ và B E≡ .
▼ Dang 3: V n d ng b t ng th c trong ư ng tròn.
I. Ki n th c c n nh :
- ư ng kính dây cung l n nh t c a ư ng tròn.
- Trong ư ng tròn ( )O : AB và CD
là hai dây cung, H và K l n lư t là
hình chi u vuông góc trên AB và CD .
Ta có OH OK AB CD≥ ⇔ ≤
AB CD AOB COD⇔ ≤ ⇔ ≤
Ví d 1:
Cho ư ng tròn ( ; );O R AC là ư ng kính.BD là dây cung c a ( ; )O R và BD vuông góc v i AC .
Xác nh v trí c a dây BD di n tích t giác ABCD l n nh t.
L i gi i
AB CD⊥ (gt)
Nên
1
. .
2
ABCDS AC BD R BD= =
Mà BD là dây cung c a ( ; )O R
do ó 2BD R≤
V y 2
2ABCDS R≤ .
D u " "= x y ra BD là ưòng kính c a ( )O .
Ví d 2:
Cho n a ư ng tròn ( ; )O R ư ng kính AB . M là i m di ng trên n a ư ng tròn. Qua M
v ti p tuy n v i ư ng tròn, g i D,C l n lư t là hình chi u, c a A; B trên ti p tuy n y. Xác
nh v trí c a i m M di n tích cùa t giác ABCD có giá tr l n nh t.
L i gi i
Ta có AD DC⊥ (gt)
BC DC⊥ (gt) AD BC⇒
ABCD⇒ là hình thang mà 90o
D =
nên ABCD là hình thang vuông.
OM DC⊥ nên OM AD và O là trung i m AB
Nên OM là ư ng trung bình c a hình thang ABCD
2
AD BC
OM
+
⇒ =
Do ó . .
2
ABCD
AD BC
S DC OM DC
+
= =
V AE BC⊥ . T giác ADCE là hình ch nh t
( 90 )O
ADC DCE AEC= = = DC AE⇒ =
90O
AEC = E⇒ thu c ư ng tròn ư ng kính AB.
33
AE⇒ là dây cung c a ư ng tròn ( )O .
2DC R⇒ ≤ (trong ư ng tròn ư ng kính là dây cung l n nh t)
Do ó 2
.2 2ABCDS R R R≤ =
D u " "= x y ra AE⇔ là ư ng kính c a ( )O
OM AB M⇔ ⊥ ⇔ là trung i m AB .
Ví d 3:
Cho tam giác u ABC n i ti p trong ư ng tròn ( ; )O R . M là i m di ng trên trên ( )O . Xác
nh các v trí c a i m M t ng MA MB MC+ + t giá tr l n nh t.
L i gi i
Xét M thu c cung BC.
Trên dây MA l y i m D sao cho
MD MB MBD= ⇒ cân.
60o
BMA BCA= = (hai góc n i ti p cùng ch n AB )
Do dó MBD u.
,BD MB⇒ = 60o
DBM =
60o
ABD ABC DBC DBC= − = −
60o
MBC MBD DBC DBC= − = −
Suy ra ABD MBC= .
Xét MBC và DBA có
MB BD= , MBC ABD= , BC AB= ( ABC u)
Do ó MBC = DBA(c.g.c)
Suy ra MC DA=
Ta có MA MD DA MB MC= + = +
2.MA MB MC MA⇒ + + = .
MA là dây cung c a ( ; )O R 2MA R⇒ ≤
( ư ng kính là dây cung l n nh t c a ư ng tròn)
Do ó 4MA MB MC R+ + ≤ (không i)
D u " "= x y ra MA⇔ là ư ng kính c a ( )O
M⇔ là trung i m cung BC.
L p lu n tương t ta có ba v trí MA MB MC+ + t giá tr l n nh là trung i m các cung
BC; AC; AB.
Ví d 4:
Cho ư ng tròn ( ; )O R ; BC là dây cung c nh ( 2BC R≠ ). A là i m chuy n ng trên cung
l n BC. Xác nh v trí c a A chu vi tam giác ABC l n nh t.
L i gi i
.ABCP AB AC BC= + +
BC không i.
Trên tia i tia AB l y i m D sao cho AD AC=
ADC cân t i A 2BAC ADC⇒ =
BAC không i ADC⇒ không i.
BDC không i, BC c nh
34
D⇒ thu c cung ch a góc có s o
1
4
s BC c a ( )O
d ng trên o n th ng BC.
ABCP l n nh t ( )max ( )maxAB AC AB CD⇔ + ⇔ +
maxBD⇔ ⇔ BD là ư ng kính
c a cung ch a góc nói trên.
Khi ó 90o
BDC = .
Mà 90o
ABC BDC ACB ACD+ = + =
BDC ACD= ( )AC AD=
Do ó ABC ACB AB AC= ⇔ = ⇔ A là trung i m cung l n BC.
Ví d 5 :
Cho ư ng tròn ( ; )O R .A i m c nh trong ư ng tròn ( A O≠ ). Xác nh v trí c a di m B
trên ư ng tròn ( )O sao cho OBA l n nh t.
L i gi i
V dây BC c a ư ng tròn ( )O qua A.
OBC cân ( )OB OC=
180
2
o
BOC
OBC
−
=
v OH BC⊥ ( )H BC∈
A BC∈ nên OH OA≤ (không i)
D u " "= x y ra H A⇔ ≡ AB OA⇔ ⊥ t i A.
Ta có OBAl n nh t BOC⇔ nh nh t
BC⇔ nh nh t ⇔ dây BC nh nh t
⇔ OH l n nh t H A⇔ ≡ AB OA⇔ ⊥ t i A.
▼ D ng 4:V n d ng b t ng th c i s
I. Ki n th c c n n m:
● B t ng th c côsi cho 2 s dương:
Cho 2 s dương a và b ta có:
2
a b
ab
+
≥
D u “=” x y ra khi và ch khi a=b .
● B t ng th c Bunhiacopxki Sraxo (B.C.S):
Cho 4 s th c a,b,x,y ta có:
( ) ( )( )2 2 2 2 2
ax by a b x y+ ≤ + +
D u “=” x y ra khi và ch khi ax=by.
II. M t s bài t p ví d :
Ví d 1:
Cho o n th ng AB=a. C là i m trên o n th ng AB. V các hình vuông ABCD và CBFG.
Xác nh v trí di m C ACDE CBFGS S+ t giá tr nh nh t.
L i gi i:
t AC = x
35
Ta có CB a x= − (0 x a≤ ≤ )
2
ACDES x= , 2
( )CBFGS a x= −
2 2
( )ACDE CBFGS S x a x+ = + −
2 2 2
2x a ax x= + − +
2 2
2
2( )
4 2
a a
x ax= − + +
2 2 2
2
2 2 2
a a a
x
 
= − + ≥ 
 
(không i)
D u " "= x y ra 0
2 2
a a
x x⇔ − = ⇔ =
Ví d 2:
Cho o n th ng BC c nh. A là i m di ng sao cho tam giac ABC nh n. AA’ là ư ng cao
và H là tr c tâm c a tam giác ABC. Xác nh v trí i m A
'. 'AA HA t giá tr l n nh t.
L i gi i:
Xét 'A BH và 'A AC có ( )' ' 90 , ' 'o
BA H AA C A BH A AC= = =
(hai góc nh n có c nh tương ng vuông góc)
Do ó
' '
' ' '. ' ' . ' .
' '
HA A B
A BH A AC AA HA A B A C
A C AA
⇒ = ⇒ =∼
Ta có 2
' . ' ' ( ' ) ' . 'A B A C A B BC A B A B BC A B= − = −
2
2
( ' . ' )
4 2
BC BC
A B BC A B= − − +
22 2
' .
4 2 4
BC BC BC
A B
 
= − − ≤ 
 
V y
2
AA'.HA' .
4
BC
≤ (không i)
D u " "= x y ra '
2
BC
A B⇔ =
'A⇔ là trung i m BC A⇔ thu c trung tr c BC.
V y ABC nh n nên A n m ngoài ư ng tròn ư ng kính BC.
Ví d 3:
Trong các t giác n i ti p hình ch nh t cho trư c. Tìm t giác có t ng bình phương các c nh
nh nh t.
L i gi i:
AMQ có 90o
A = theo nh lí Pitago ta có 2 2 2
QM AM AQ= +
Tương t 2 2 2
MN BM BN= + , 2 2 2
NP CN CP= + , 2 2 2
PQ DP DQ= +
Do ó 2 2 2 2 2 2 2 2 2
MN NP PQ BM BN CN CP DP DQ+ + = + + + + +
Ta có
2 2 2
2 2 2( ) ( ) ( ) 1
2 2 2
AM BM AM BM AM BM
AM BM AB
+ + − +
+ = ≥ =
36
Ch ng minh tương t ta có
2 2 21
2
CP DP CD+ ≥
2 2 21
2
DQ AQ AD+ ≥
Do ó ( )2 2 2 2 2 2 2 21
2
MN NP PQ QM AB BC CD DA+ + + ≥ + + + (không i)
D u " "= x y ra
AM BM
BN CN
CP DP
DQ AQ
=
 =
⇔ ⇔
=
 =
MNPQ là hình thoi.
Ví d 4:
Cho i m A c nh n m ngoài ư ng tròn ( );O R . Qua A v ư ng th ng d c t ư ng tròn ( )O
t i hai i m B;C.Xác nh vi trí c a d t ng AB AC+ t giá tr nh nh t.
L i gi i:
V cát tuy n ADE qua O
Xét ABE và ACD có A (chung); AEB ACD=
(hai góc n i ti p cùng ch n cung BD)
Do ó . .
AB AE
ABE ACD AB AC AE AD
AD AC
⇒ = ⇒ =∼
Mà ( )( )
2
2 2 2
.AE AD OA OE OA OE OA OE OA R= + − = − = −
Ta có ( 2 . )AB AC AB AC AB AC+ = + −
2
( ) 2 . 2 .AB AC AB AC AB AC= − + ≥
2 2
2 OA R= − (không i)
D u " "= x y ra d⇔ là ti p tuy n c a ( );O R .
Ví d 5:
Cho n a ư ng tròn ( );O R ư ng kính AB. M là i m chuy n ng trên n a ư ng tròn. Xác
nh v trí i m M 3MA MB+ t giá tr l n nh t.
L i gi i:
90o
AMB = (góc n i ti p ch n n a ư ng tròn)
MAB có 90o
M = nên theo nh lí Pitago ta có
2 2 2 2
4MA MB AB R+ = =
Áp d ng B T 2 2 2 2
( )( )ax by a b x y= ≤ + +
Ta có: ( )2 2
3 3 (1 3)MA MB MA MB MA MB+ = + ≤ + +
2 2 21
2
BN CN BC+ ≥
37
2
4.4 4 .R R= =
3 4MA MB R= ≤ (không i)
D u " "= x y ra 3.MA MB⇔ =
MAB⇔ là n a tam giác u
⇔ s 60o
MA = .
38
Ph n 3:
C C TR TRONG LƯ NG GIÁC
▼ D ng 1: S d ng b t ng th c c a hàm Sinx va Cosx.
I. Phương pháp gi i:
Thông thư ng gi i m t bài c c tr ta s d ng các b t ã ư c ch ng minh. Tương t ,
lư ng giác v n có nh ng b t riêng bi t.
i v i hàm s ơn gi n ch có sin và cos. Ta s d ng:
1 1
1 1
Sinx
Cosx
− ≤ ≤

− ≤ ≤
II. M t s bài t p ví d :
Ví d 1:
Tìm GTLN, GTNN c a hàm s :
a) y = 1 - 2 sin3x
b) y = 1 - 1 sin x−
L i gi i:
a) Vì 0≤ sin3x ≤1 nên 1≥1 - 2 sin3x ≥1-2 = -1.
D u b ng x y ra khi : sin3x = 0⇔ sin3x = 0⇔ 3x = kπ ⇔ x =
3
kπ
Vâ GTLN c a hàm s là 1 và GTNN c a hàm s là -1t i x =
3
kπ
Vì -1≤ sinx≤1 nên 2 ≥1- sinx ≥0 ⇔ 2 ≥ 1 sin x− ≥0 ⇔ 1+ 2 ≥1+ 1 sin x− ≥1.
1+ 1 sin x− = 1 + 2 khi sinx = -1 ⇔ x = 2
2
k
π
π− +
1+ 1 sin x− = 1 khi sinx = 1 ⇔ x = 2
2
k
π
π+
Vâ GTLN c a hàm s là 1+ 2 t i x = x = 2
2
k
π
π− + và GTNN c a hàm s là 1 t i x =
2
2
k
π
π+ .
V n có th s d ng m t s kĩ năng cơ b n tìm c c tr :
Ví d 2:
Tìm GTLN c a sin12
x + cos12
x
L i gi i:
Cách 1:
Vì -1≤sinx≤1 và -1≤cosx⇒ 1 nên ta có : sin12
x ≤ sin2
x và cos12
x ≤ cos2
x
⇒ sin12
x + cos12
x ≤ sin2
x + cos2
x = 1
Cách 2:
39
Ta sin12
x + cos12
x = 1 – 2sin6
x.cos6
x ≤1.
V y GTLN cu sin12
x + cos12
x là 1.
Ví d 3:
Tìm GTLN, GTNN cu : sinx + sin
2
3
x
π 
+ 
 
( Bài t p c n qua bư c bi n i)
L i gi i:
Ta có sinx + sin
2
3
x
π 
+ 
 
= 2sin
3
x
π 
+ 
 
.cos
3
π
= sin
3
x
π 
+ 
 
.
Mà
-1≤ sin
3
x
π 
+ 
 
≤1 nên GTNN c a sinx + sin
2
3
x
π 
+ 
 
là -1 và GTLN là 1.
Ví d 4:
Tìm GTLN , GTNN c a bi u th c : 2 2
( )(1 )
(1 )(1 )
a b ab
a b
+ −
+ +
∀ a,b
L i gi i:
( i v i bài t p này, ban u không ph i là d ng lư ng, ta ph i ưa v lư ng giác qua
các phép bi n i tìm c c tr ).
t a= tan x
b= tan y
Ta có:
2 2
( )(1 )
(1 )(1 )
a b ab
a b
+ −
+ +
= 2 2
(tan tan )(1 tan .tan )
(1 tan )(1 tan )
x y x y
x y
+ −
+ +
=
2 2
sin( ) c ( )
.
cos .cos cos .cos
1 1
.
cos cos
x y os x y
x y x y
x y
+ +
=
1
2
sin 2( )x y+ ≤
1
2
( vì sin 2( )x y+ ≤1) ⇒ -
1
2
≤ 2 2
( )(1 )
(1 )(1 )
a b ab
a b
+ −
+ +
≤
1
2
V y GTNN c a bi u th c là -
1
2
và GTLN là
1
2
.
▼ D ng 2: Hình thành bình phương
I. Phương pháp gi i:
D a trên s chuy n i qua l i gi a sin và cos, s d ng các công th c lư ng giác.
II. M t s bài t p ví d :
Ví d 1:
Tìm GTLN c a bi u th c A= cosA.cosB.cosC
40
L i gi i : Ta có: cosA.cosB.cosC = [ ]
1
( ) ( ) cos
2
cos A B cos A B C+ + − =
[ ]
1
cos ( ) cos
2
C cos A B C− + − = 2 21 1
cos . ( ) ( )
2 4
cos C C cos A B cos A B
 
− − − + −  
+ 21
1 sin ( )
8
A B − − 
=
1
8
2
2 21 1 1
cos ( ) sin ( )
2 2 8
C cos A B A B
 
− − − − −  
≤
1
8
D u b ng x y ra khi cosC =
1
( )
2
cos A B− cosC =
1
2
⇔
sin( )A B− = 0 A B=
⇔ A B= =C = 60o
.
V y GTLN c a bi u th c A là
1
8
khi ∆ ABC u
▼ Dang 3: S d ng các b t ng th c lư ng giác trong tam giác.
I. Phương pháp gi i:
Trong tam giác ABC nh n:
tan tan tan tan tan tan 1
2 2 2 2 2 2
cot .cot cot cot cot cot 1
tan tan tan tan .tan .tan
A B B C C A
A B B C C A
A B C A B C
∗ + + =
∗ + + =
∗ + + =
a) tanA+ tanB+ tanC 3 3≥
b) tan tan tan 3
2 2 2
A B C
+ + ≥
c)
1
tan .tan .tan
2 2 2 3 3
A B C
≤
d)
3
cos cos cos
2
A B C+ + ≤
► Ch ng minh:
a) Áp d ng b t Côsi cho 3 s dương:
3tanA tanB tanC 3 tan .tan .tan
3tanA tanB tanC 3 tanA tanB tanC
3 2(tanA tanB tanC) 27(tanA tanB tanC) (tanA tanB tanC) 27
tanA tanB tanC 3 3
A B C+ + ≥
⇔ + + ≥ + +
⇔ + + ≥ + + ⇔ + + ≥
⇔ + + ≥
( pcm)
41
b) Áp d ng b t ng th c: ( ) ( )
2
3a b c ab bc ac+ + ≥ + + ta coù:
2
tan tan tan 3 tan tan tan tan tan tan
2 2 2 2 2 2 2 2 2
A B C A B B C C A   
+ + ≥ + +   
   
= 3
⇔ tan tan tan
2 2 2
A B C
+ + ≥ 3 ( pcm)
c) Áp d ng b t Côsi 2 2 23tan tan tan tan tan tan 3 tan tan tan
2 2 2 2 2 2 2 2 2
A B B C C A A B C
+ + ≥
⇔
3
2 2 2 1
tan tan tan
2 2 2 3
A B B  
≤  
 
⇔
1
tan tan tan
2 2 2 3 3
A B B
≤ ( pcm)
D u b ng x y ra khi ∆ ABC u.
Cách 1:
Xét cos cos cos 60A B C cos+ + + = 2
2 2
A B A B
cos cos
+ −   
   
   
+
60 60
2
2 2
C C
cos cos
   + −
   
   
60
2
2 2
A B C
cos cos
  + + 
≤ + =   
    
60 60
4 .
4 4
A B C A B C
cos cos
+ + + + − −
= 4 60cos≤
1
cos cos cos 2
2
A B C⇔ + + + ≤
3
cos cos cos
2
A B C⇔ + + ≤ ( pcm)
Cách 2: ( ng d ng tích vô hư ng ch ng minh)
L y các vectơ 1e , 2e , 3e như hình v và có dài là 1: 1 2 3e e e= = =1.
Hi n nhiên ta có:
( )
( ) ( ) ( )
2
1 2 3
1 2 2 3 3 1
0
3 2 , 2 , 2 , 0
e e e
cos e e cos e e cos e e
+ + ≥
⇔ + + + ≥
3
cos cos cos
2
A B C⇔ + + ≤ ⇒ pcm
D u b ng x y ra khi 1 2 3e e e+ + = 0 ABC⇔ ∆ ABC⇔ ∆ u
Có th s d ng các b t th c trên ho c khai thác thêm các b t sau trong tam giác (ph i
ch ng minh trư c khi áp d ng):
) tan tan tan 3 3n n n n
e A B C+ + ≥
3 3
)sin sin sin
2
f A C B+ + ≤
A
B C
42
2 2 2
)cot cot cot 3
3
)
4
g A B C
h cos A cos B cos C
+ + ≥
+ + ≥
2 2 2
3
)sin sin sin
2 2 2 2
3
)
2 2 2 2
3
)sin sin sin
2 2 2 4
A B C
i
A B C
j cos cos cos
A B C
k
+ + ≤
+ + ≤
+ + ≥
2 2 2
)tan tan tan 9m A B C+ + ≥ (∆ nh n)
2 2 2
) tan tan tan 1
2 2 2
3 3
)sin .sin .sin
8
1
)sin .sin .sin
2 2 2 8
3 3
) os . os . os
2 2 2 8
A B C
n
o A B C
A B C
p
A B C
q c c c
+ + ≥
≤
≤
≤
2 2 2 9
) os os os
2 2 2 4
A B C
l c c c+ + ≤
II. M t s bài t p ví d :
Ví d 1:
Tìm GTNN c a bi u th c:
A= 2 2 2
sin sin sin
2 2 2
A B C
+ + sin .sin .sin
2 2 2
A B C
43
L i gi i:
Ta có A=
1 cos 1 cos 1 cos
sin .sin .sin
2 2 2 2 2 2
A B C A B C− − −
+ + +
=
3 1
1 4sin .sin .sin sin .sin .sin
2 2 2 2 2 2 2 2
A B C A B C 
− + + 
 
(vì cos cos cos 1 4sin .sin .sin
2 2 2
A B C
A B C+ + = + )
=1− sin .sin .sin
2 2 2
A B C 1
1
8
≥ − =
7
8
V y giá tr nh nh t c a bi u th c là :
7
8
.
Ví d 2:
Tìm GTNN c a bi u th c B= 6 6 6
tan tan tan
2 2 2
A B C
+ +
L i gi i:
Không s d ng gián ti p các b t th c cm trên, ta s d ng các b t toán h c
quen thu c.
Ta có:
( )6 6 6 3 3 3 3 3 3
tan tan tan tan tan .tan tan .tan tan 1
2 2 2 2 2 2 2 2 2
A B C A B B C C A
+ + ≥ .
N u x = tan tan
2 2
A B
, y = tan tan
2 2
B C
, z = tan tan
2 2
C A
, thì x+y+z = 1.
Áp d ng BCS v i hai dãy , ,x y z vaø 3 3 3
, ,x y z ta có:
( )( ) ( )
23 3 3 2 2 2
x y z x y z x y z+ + + + ≥ + +
( )
22
3
x y z + +
≥  
  
=
1
9
( )3 3 3 1
2
9
x y z⇔ + + ≥
T (1) và (2) 6 6 6 1
tan tan tan
2 2 2 9
A B C
⇒ + + ≥ . D u b ng x y ra khi x = y = z
⇔ A=B=C .
V y GTNN c a B là
1
9
.
1. Tìm GTLN c a bi u th c D= tan .tan .tan
4 4 4
A B C
44
2. Cho
, , 0
. .
a b c
a Sinx b Siny c
≥

+ =
Tìm GTLN c a bi u th c
E=
2 2
os osc x c y
a b
+ .
▼ D ng 4: Bi u th c ch a các hàm s lư ng giác.
I. Phương pháp gi i:
Gi s các góc A, B, C tho mãn hai i u ki n:
1)
)
( ) ( ) 2
2
A B
f A f B f
+ 
+ ≥  
 
ho c 2
( ). ( )
2
A B
f A f B f
+ 
≥  
 
ng th c x y ra khi A=B;
2) 3( ) 2
3 2
C
f C f f
π
π
 
+  
+ ≥   
   
 
ho c 2 3( ).
3 2
C
f C f f
π
π
 
+  
≥   
   
 
ng th c x y ra khi C=
3
π
.
Khi c ng ho c nhân (1), (2) ta s có b t ( ) ( ) ( ) 3
3
f A f B f C f
π 
+ + ≥  
 
(3)
ho c 3
( ). ( ). ( )
3
f A f B f C f
π 
≥  
 
(4)
ng th c x y ra khi A=B=C. Tương t ta cũng có b t v i chi u ngư c l i
Xét các VD sau:
Ví d 1:
Trong tam giác ABC, tìm GTNN c a bi u th c
1 1 1
1 sin 1 sin 1 sinA B C
+ +
+ + +
L i gi i:
Ta có:
1 1
1 sin 1 sinA B
+
+ +
4
2 sin sinA B
≥
+ +
(áp d ng
1 1 4
x y x y
+ ≥
+
)
( )
4 4
2 2 sin sin
2 2 sin .
2 2
A B A BA B
cos
≥ =
+ −+ +
+
(áp d ng BCS)
2
1 sin
2
A B
≥
+
+
⇒ ≥
1 1 2
+ (5)
1+ sinA 1+ sinB A + B
1+ sin
2
(có d ng (1))
45
Tương t
sin 60 60
≥
1 1 2
+ (6)
1+ sinC 1+ C+
1+ sin
2
.C ng (5) và (6) ta có:
1 1 1 1
1 sin 1 sin 1 sin 1 sin 60A B C
+ + +
+ + + +
1 1
2
60
 
 
 ≥ +
 
 
 
A + B C+1+ sin 1+ sin
2 2
4
1 sin 60
≥
+
( Cũng làm tương t các bư c (5), (6))
Suy ra
1 1 1
1 sin 1 sin 1 sinA B C
+ +
+ + +
3
1 sin 60
≥
+
4
3 2
2 3
=
+
V y GTNN c a bi u th c là 4
3 2
2 3+
. D u b ng x y ra khi ABC∆ u.
Ví d 2:
Trong tam giác ABC, tìm GTNN c a bi u th c
1 1 1
1 . 1 . 1
sin sin sinA B C
     
+ + +     
     
L i gi i:
Ta có:
1 1
1 . 1
sin sinA B
   
+ +   
   
1 1 1
1
sin sin sin .sinA B A B
= + + +
2
2 1
1
sin .sin sin .sinA B A B
 
≥ + +  
 
2
1
1
sin .sinA B
 
= + 
  ( ) ( )
2
2
1
cos A B cos A B
 
 = +
 − − + 
( )
2
2
2 1
1 1
1 sin
2
A Bcos A B
 
   
 ≥ + = + + − +   
 
2
1
1
60
sin
2
C
 
 
≥ + 
+  
 
(có d ng (1))
46
Tương t
2
1 1 1
1 . 1 1
60sin sin 60
sin
2
CC
 
    
+ + ≥ +    
+      
 
(8)
Nhân (7) và (8) ta ư c
1 1 1 1
1 . 1 . 1 . 1
sin sin sin sin 60A B C
       
+ + + +       
       
2
1 1
1 . 1
60sin sin
2 2
A B C
   
   
≥ + +   + +          
4
1
1
sin 60
 
≥ + 
 
Suy ra
1 1 1
1 . 1 . 1
sin sin sinA B C
     
+ + +     
     
3
1
1
sin 60
 
≥ + 
 
3
2
1
3
 
= + 
 
V y GTNN c a bi u th c là
3
2
1
3
 
+ 
 
khi ABC∆ u.
▼ D ng 5: S dung o hàm
I . Ki n th c c n n m:
gi i các d ng bài toán này c n s d ng t i m t s cong th c tính o
hàm sau ây:
( )
( ) ( )
( )
( ) ( )
( )
( )
( )
2
2
'
' '.
'
os ' '.
1
tan '
'
tan '
Sinx Cosx
Sinu u Sinx
Cosx Sinx
C u u Sinx
x
Cos x
u
u
Cos u
=
=
= −
= −
=
=
II. M t s bài t p ví d :
.Ví d 1:
Tìm giá tr l n nh t và nh nh t c a hàm s
( ) 3cos3 2cos2 9cos 2y f x x x x= = + + +
L i gi i:
L i gi i:
TX : D=R
Ta có ( ) ( ) ( )3 2
3 4cos 3cos 2 2cos 1 9cos 2y f x x x x x= = − + − + +
3 2
12cos 4cosx x= +
47
10
243
16
8−
0
t t = cos x , 1 1t− ≤ ≤
Ta có ( ) 3 2
12 4y g t t t= = +
( )' ' 2
' 2
36 8
0 36 8 0
y g t t t
y t t
= = +
= ⇔ + =
( )4 9 2 0
0
2
9
t t
t
t
⇔ + =
=
⇔
 = −

B ng bi n thiên
t
-1
2
9
− 0 1
( )'
g t + 0 - 0 +
( )g t
Căn c vào b ng bi n thiên ta ư c:
max f(x) = max g(t) = 16
min f(x) = min g(t) = -8
Ví d 2:
Tìm giá tr l n nh t và giá tr nh nh t c a hàm s
( ) 2 2
cos 2cos 5 cos 4cos 8y f x x x x x= = − + + + +
L i gi i:
TX : D=R
t t = cos x, 1 1t− ≤ ≤
Ta có ( ) 2 2
2 5 4 8y g t t t t t= = − + + + +
Dg(x)= [-1,1]
( )' '
2 2
1 2
2 5 4 8
t t
y g t
t t t t
− +
= = +
− + + +
( ) ( )
( )( )
2 2
2 2
1 4 8 2 2 5
2 5 4 8
t t t t t t
t t t t
− + + + + − +
=
− + + +
48
2 2 5+
2 13+
5
'
0y =
( ) ( )2 2
1 4 8 2 2 5 0t t t t t t⇔ − + + + + − + =
( ) ( )2 2
2 2 5 1 4 8 0t t t t t t⇔ + − + = − + + = (do 1 1t− ≤ ≤ )
( ) ( ) ( ) ( )
( )( )
2 22 2
2 2 5 1 4 8
2 1 0
24 12 0
2 1
1
1
2
2
2 1
t t t t t t
t t
t
t
t
t
t
 + − + = − + +
⇔ 
+ − ≥
+ =
⇔ 
− ≤ ≤

= −
⇔ ⇔ = −
− ≤ ≤
B ng bi n thiên
t
-1
1
2
− 1
( )'
g t + 0 -
( )g t
Căn c vào b ng bi n thiên ta có
max f(x) = max g(x) = 2 13+
min f(x) = min g(x) = 5
Ví d 3:
Cho cos 2 cos 2 1, ,x y x y R+ = ∀ ∈ .Tìm giá tr nh nh t c a hàm s :
2 2
tan tanA x y= +
L i gi i:
Ta có: 2 2
tan tanA x y= + ( ) ( )2 2
tan 1 tan 1 2x y= + + + −
2 2
2
2
1 1 2 2
2 2
cos cos 1 cos2 1 cos2
2cos 2 2cos2 2
cos 2 cos2 2
x y x y
x x
x x
= + − = + −
+ +
− +
=
− + +
t cos2t x= v i 1 1t− ≤ ≤ , ta có :
49
( )
( )
( )
2
2
22
2 1
( ) , 1 1
2
6 2 1 1
'( ) 0
22
t t
A f t t
t t
t
f t t
t t
− +
= = − ≤ ≤
− + +
−
⇒ = = ⇔ =
− + +
t
1−
1
2
1
'( )f t − 0 +
( )f t
2
3
V y min A
2
3
= khi ,
6
x
π
π= ± + ∈»k k
( )g x nh nh t 2 1
sin
3
x⇔ = ⇒ min ( )g x
2
1 1 5 5
3
3 3 3 3
 
= − + = 
 
Do ó :
1 3 4 8
1 1
3 5 3 5
y y+ ≤ ≤ + ⇔ ≤ ≤
V y max
8
5
y = ; min
4
3
y =
► . M t s bài t p d ng tương t :
1.Tìm giá tr l n nh t c a hàm s :
2 21 1
1 cos 5 2sin
2 2
y x x= + + +
2.Tìm giá tr nh nh t c a:
2 2
2 2
2 2
1 1
sin cos
sin cos
y x x
x x
   
= + + +   
   
3.Tìm giá tr l n nh t và giá tr nh nh t c a hàm s :
1
50
2 2
( ) 2sin 3sin cos 5cosy f x x x x x= = + +
4.Tim giá tr l n nh t và giá tr nh nh t c a hàm s :
2
4sin 2 sin 2
4
y x x
π 
= + + 
 
5.Tìm giá tr nh nh t c a hàm s :
1 1
,0
sin cos 2
y x
x x
π
= + < <
6.Tìm giá tr l n nh t và giá tr nh nh t c a hàm s :
( )( )( ) 2sin cos 2cos sinf x x x x x= + −
7. Tìm giá tr l n nh t và giá tr nh nh t c a hàm s :
4 2
4 2
3cos 4sin
3sin 2cos
x x
y
x x
+
=
+
8.Cho tam giác ABC tìm giá tr l n nh t c a:
( )3 cos 3 cos cosP B A C= + +
9.Tìm giá tr nh nh t c a bi u th c :
4 4 2 2
cot cot 2tan .tan 2P a b a b= + + +
10.Cho , ,α β δ tho mãn i u ki n :
2 2 2
cos cos cos 1α β δ+ + =
Tìm giá tr l n nh t c a:
2 2 2
1 cos 1 cos 1 cosy α β δ= + + + + +
11.Tìm giá tr nh nh t c a hàm s
( )
3
2 2
1
cos sin
cos sin
y x x
x x
= + +
12.Tìm giá tr nh nh t c a hàm s
10 10
cos siny x x= +
13. Cho ∆ ABC. Tìm giá tr l n nh t c a bi u th c
M= cos2A + cos2B – cos2C
14. Tìm GTLN và GTNN c a hàm s
2 cos
sin cos 2
x
y
x x
+
=
+ −
15. Tìm GTLN và GTNN c a hàm s
2
2
cos sin cos
sin 1
x x x
y
x
+
=
+
(1)
16. nh m hàm s ( )4 4
2 sin cos sin cos cos2y x x m x x x= + + (1)
51
có giá tr l n nh t không l n hơn 2
17. Tìm GTLN và GTNN c a hàm s cos siny x x= +
18. Tìm giá tr l n nh t c a hàm s
( )2 2
4
3sin 1 4sin
cos
x x
y
x
−
= v i 0
6
x
π
< <
19. Cho ABC∆ có 3 góc nh n, tìm giá tr nh nh t c a bi u th c
P = tanA.tanB.tanC
20. Tìm giá tr l n nh t c a bi u th c
1 tan tan 1 tan tan 1 tan tanM A B B C C A= + + + + +
v i A, B, C >0 và A + B + C =
2
π
21. Trong m i tam giác ABC,nh ng tam giác nào làm cho bi u th c sau t
giá tr l n nh t:
Hư ng d n và áp s :
1. Ta có: 2 21 5 1
1 cos sin
2 4 2
y x x= + + +
Áp d ng B T Bunhiacopski cho 4 s , ta có:
2 21 5 1
1 cos sin
2 4 2
x x+ + + 2 2 2 21 5 1
1 1 . 1 cos sin
2 4 2
x x≤ + + + +
9 1 22
2
4 2 2
≤ + =
V y max
22
2
y =
D u “=” x y ra khi : 2 21 5
1 cos sin
2 4
x x+ =
2. Ta có:
( )
2 2 2
2 2 2 2 2 2
2 2 2 2
1 1 1 1
sin cos 1 1 sin cos
sin cos cos cos
x x x x
x x x x
         
+ + + ≤ + + + +         
           
2 2 2
2 2 2 2
2 2 2 2
1 1 1 1 1
sin cos sin cos
sin cos 2 sin cos
x x x x
x x x x
     
⇒ + + + ≥ + + +     
     
3 3 3
3 3 3
sin sin sin
cos cos cos
A B C
M
A B C
+ +
=
+ +
52
2
2 2
1 1
1
2 sin .cosx x
 
≥ + 
 
( )
2
2
2
1 4 1 25
1 1 4
2 sin 2 2 2x
 
≥ + ≥ + = 
 
V y min
25
2
y =
D u “=” x y ra khi:
2 2
sin cos
4sin 2 1
x x
x
x
π
π
 =
⇔ = ± +
=
k
3.Ta có : 2 2
2sin 3sin cos 5cosy x x x x= + +
( )
3 5
1 cos2 sin 2 1 cos2
2 2
x x x= − + + +
( )
7 3
sin 2 +cos2x
2 2
x= +
7 3 2
cos 2
2 2 4
x
π 
= + − 
 
Ta có:
( ) ( )
3 2 3 2 3 2
1 cos 2 1 cos 2
4 2 2 4 2
1 7 3 2 1
7 3 2 cos 2 7 3 2
2 2 2 4 2
x x
x
π π
π
   
− ≤ − ≤ ⇔ − ≤ − ≤   
   
 
⇔ − ≤ + − ≤ + 
 
V y max ( )1
7 3 2
2
y = + ,min ( )1
7 3 2
2
y = −
4. Ta có: 2
4sin 2 sin 2
4
y x x
π 
= + + 
 
( )2 1 2cos sin 2 +cos2x
=2+sin 2x - cos2x
=2+ 2 sin 2
4
x x
x
π
= − +
 
− 
 
V i 1 sin 2 1 2 2 2 2
4
x y
π 
− ≤ − ≤ ⇔ − ≤ ≤ + 
 
V y max 2 2y = + , min 2 2y = −
5. V i 0
2
x
π
< < cos 0x⇒ > và sin 0x >
53
Áp d ng B T Cauchy, ta có:
1 1
sin cos
y
x x
= +
2 2 2
2 2
sin cos sin 2x x x
≥ = ≥
D u “=” x y ra khi:
sin 2 1
1 1
cos sin 2
x
x x
=


=
tan 1 0,
4 2
x x
π π 
⇔ = ⇔ = ∈ 
 
6.Ta có: ( )( )( ) 2sin cos 2cos sinf x x x x x= + −
2 2
4sin cos 2sin 2cos sin cosx x x x x x= − + −
( )2 2
3sin cos 2 cos sinx x x x= + −
3
sin 2 2cos2
2
x x= +
( )
1
4cos2 3sin 2
2
x x= +
5 4 3
cos2 sin 2
2 5 5
x x
 
= + 
 
t
4 3
cos ,sin
5 5
α α= = v i 0
2
π
α< <
Ta có : ( ) ( )
5 5
( ) cos2 cos sin 2 sin cos 2
2 2
f x x x xα α α= + = −
V i ( ) ( )
5 5 5
1 cos 2 1 cos 2
2 2 2
x xα α− ≤ − ≤ ⇔ − ≤ − ≤
5 5
( )
2 2
f x⇔ − ≤ ≤
V y max
5
( ) ;
2
f x = min
5
( )
2
f x = −
7. Ta có:
( )
( )
22 24 2
4 2 4 2
3 1 sin 4sin3cos 4sin
3sin 2cos 3sin 2 1 sin
xx x
y
x x x x
− ++
= =
+ + −
4 2
4 2 4 2
3sin 2sin 3 1
1
3sin 2sin 2 3sin 2sin 2
x x
x x x x
− +
= = +
− + − +
t
2
4 2 2 1 5
( ) 3sin 2sin 2 3 sin
3 3
g x x x x
 
= − + = − + 
 
( )g x l n nh t 2
sin 1x⇔ = ⇒ max
2
1 5
( ) 3 1 3
3 3
g x
 
= − + = 
 
54
8. Ta có: 3 cos 6cos cos
2 2
A C A C
P B
+ −
= +
2
2
2
3 cos 6sin cos
2 2
3 1 2sin 6sin
2 2
2 3 sin 6sin 3
2 2
3 5 3 5 3
2 3 sin
2 2 2 2
B A C
B
B B
B B
B
−
= +
 
≤ − + 
 
≤ − + +
 
≤ − − + ≤  
 
Suy ra : max
5 3
2
P = khi
cos 1
302
3 120
sin
2 2
A C
A C
B B
−
=  = = 
⇔ 
= =

9. Ta có: ( )
2
4 4 2 2 2 2
cot cot cot cot 2cot .cota b a b a b+ = − +
( )
2
2 2 2 2 2 2
cot cot 2cot .cot 2tan .tan 2P a b a b a b⇒ = − + + +
( ) ( )
22 2 2 2 2 2
cot cot 2 cot .cot tan .tan 2cot .cot .tan .tan
4cot .cot .tan .tan 2
a b a b a b a b a b
a b a b
= − + + −
+ +
( ) ( )
2 22 2
cot cot 2 cot .cot tan .tan 4 2 6a b a b a b= − + − + + ≥
D u “=” x y ra khi và ch khi :
cot cot 0
cot .cot tan .tan 0 cot 1 4
a b a b
a b
a b a b a
π− = = 
⇔ ⇔ = = 
− = = 
V y min 6P =
10. Áp d ng B T Bunhiacopski cho 6 s , ta có:
2 2 2
1 cos 1 cos 1 cosα β δ+ + + + +
2 2 2 2 2 2
1 1 1 . 1 cos 1 cos 1 cosα β δ≤ + + + + + + +
2 2 2
3. 3 cos cos cosα β δ≤ + + +
vì 2 2 2
cos cos cos 1α β δ+ + = nên ta có :
2 2 2
1 cos 1 cos 1 cosα β δ+ + + + + 3. 4≤ 2 3=
V y max 2 3y =
55
D u “=” x y ra khi: 2 2 2
1 cos 1 cos 1 cosα β δ+ = + = +
2 2 2
2 2 2
1 cos 1 cos 1 cos
1
cos cos cos
3
1
cos cos cos
3
α β δ
α β δ
α β δ
⇔ + = + = +
⇒ = = =
⇔ = = = ±
11.Ta có ( )
3
2 2
1
cos sin
cos sin
y x x
x x
= + +
3
2
1
2 cos
4 1
sin 2
2
x
x
π  
= − +  
    
 
 
3
2
4
2 2 cos
4 sin 2
x
x
π 
= − + 
 
vì 3
cos 1 cos 1
4 4
x x
π π   
− ≥ − ⇒ − ≥ −   
   
3
2 2 cos 2 2
4
x
π 
⇔ − ≥ − 
 
và 2
2
4
0 sin 2 1 4
sin 2
x
x
≤ ≤ ⇒ ≥
suy ra 3
2
4
2 2 cos 4 2 2
4 sin 2
y x
x
π 
= − + ≥ − 
 
D u “=” x y ra
2
cos 1
4
sin 2 1
x
x
π  
− = −  
⇔  
 =
5
4
x
π
⇔ =
V y min 4 2 2y = −
12. Ta có: 10 10
cos siny x x= +
5 5
1 cos2 1 cos2
2 2
x x+ −   
= +   
   
( ) ( )
5 5
5
1
1 cos2 1 cos2
2
x x = + + −
 
56
( 2 3 4 5
5
1
1 5cos2 10cos 2 10cos 2 5cos 2 cos 2
2
x x x x x= + + + + +
)2 3 4 5
1 5cos2 10cos 2 10cos 2 5cos 2 cos 2x x x x x+ − + − + −
( )2 41
2 20cos 2 10cos 2
32
x x= + +
( )2 41
1 10cos 2 5cos 2
16
x x= + +
( )
221
5 cos 2 1 4
16
x = + −
  
M t khác 2 2
0 cos 2 1 1 1 cos 2 2x x≤ ≤ ⇔ ≤ + ≤
( )
22
1 1 cos 2 4x⇔ ≤ + ≤
( )
22
5 5 1 cos 2 20x⇔ ≤ + ≤
( ) ( ) ( )
221 1 1
5 4 5 1 cos 2 4 20 4
16 16 16
x ⇔ − ≤ + − ≤ −
  
1
1
16
y⇔ ≤ ≤
V y max y =1, d u “=” x y ra khi x=0
min y =
1
16
, d u “=” x y ra khi x=
4
π
13.
Ta có
M= cos2A + cos2B – cos2C
= 2cos(A+B)cos(A-B) + 1 – 2cos2
C
= -2cosC cos(A-B) + 1 – 2cos2
C
= -2[cos2
C + cos(A-B) cosC +
1
4
cos2
(A-B)] +
1
2
[cos2
(A-B)]
+1
= -2[cosC +
1
2
cos(A-B)]2
+
1
2
[1 - sin2
(A-B)] +1
=
3
2
-2[cosC +
1
2
cos(A-B)]2
-
1
2
sin2
(A-B)]
3
2
≤
D u “=” x y ra
( )
( )
1
cos cos 0
2
sin 0
C A B
A B

+ − =
⇔ 
 − =
57
1
cos 0
2
0
C
A B

+ =
⇔ 
 − =
1
cos 6
2
2
3
A B
C
A B C
π
π

= = = − 
⇔ ⇔ 
 = = 
V y max M =
3
2
ng v i ∆ ABC có
A = B =
6
π
và C =
2
3
π
14. Vì sin cos 2 cos
4
x x x
π 
+ = − 
 
sin cos 2 2
sin cos 2 0
x x
x x
⇒ + ≤ <
⇔ + − <
hay sin cos 2 0x x+ − ≠ x R∀ ∈
Do ó
2 cos
sin cos 2
x
y
x x
+
=
+ −
(1)
( )sin cos 2 2 cosy x x x⇔ + − = +
( )sin 1 cos 2 2y x y x y⇔ + − = + (2)
(1) có nghi m i v i x ⇔ (2) có nghi m i v i x
( ) ( )
2 22
2 2
2
1 2 2
2 2 1 4 8 4
2 10 3 0
5 19 5 19
2 2
y y y
y y y y
y y
y
⇔ + − ≥ +
⇔ − + ≥ + +
⇔ + + ≤
− − − +
⇔ ≤ ≤
V y min y =
5 19
2
− −
và max y =
5 19
2
− +
15. Ta có :
( )
24 4 2 2 2 2
sin cos sin cos 2sin cosx x x x x x+ = + −
2
21 1
1 2 sin 2 1 sin 2
2 2
1 1 cos 4 3 1
1 cos4
2 2 4 4
x x
x
x
 
= − = − 
 
− 
= − = + 
 
58
Và
1 1
sin cos cos2 sin 2 cos2 sin4
2 4
x x x x x x= =
Nên
( )
3 1
1 2 cos4 sin4
4 4 4
4 6 2cos4 sin4
2cos4 sin4 4 6
m
y x x
y x m x
x m x y
 
⇔ = + + 
 
⇔ = + +
⇔ + = −
PT trên có nghi m i v i x
( )
22 2
2 2
2 2
2 4 6
16 48 32 0
6 4 6 4
4 4
m y
y y m
m m
y
⇔ + ≥ −
⇔ − + − ≤
− + + +
⇔ ≤ ≤
Do ó
2
6 4
max
4
m
y
+ +
=
Ta có
2
6 4
max 2 2
4
m
y
+ +
≤ ⇔ ≤
2
2
4 2
4 4
0
m
m
m
⇔ + ≤
⇔ + ≤
⇔ =
16.
Ta có
(1) ( )2 2
sin 1 cos sin cosy x x x x⇔ + = + (do sin2
x +1≠ 0 )
1 cos2 1 cos2 1
1 sin2
2 2 2
cos2 2y 1 cos 2 sin2
x x
y x
y y x x x
− + 
⇔ + = + 
 
⇔ − + = + +
( )1 cos2 sin2 3 1y x x y⇔ + + = − (2)
(1) có nghi m i v i x ⇔ (2) có nghi m i v i x
( ) ( )
2 22
1 1 3 1y y⇔ + + ≥ −
2 2
2
2 2 9 6 1
8 8 1 0
y y y y
y y
⇔ + + ≥ − +
⇔ − − ≤
2 6 2 6
4 4
− +
⇔ ≤
v y
2 6
max
4
y
+
= và
2 6
min
4
y
−
=
59
17. Tìm GTLN và GTNN c a hàm s cos siny x x= +
L i gi i:
Ta có
cos siny x x= +
( )( )2 2
1 cos 1 sin 1 1 cos sinx x x x= + ≤ + + (B T
Bunhiacopski)
( )2 cos siny x x⇒ ≤ +
m t khác cos sin 2 cos 2
4
x x x
π 
+ = − ≤ 
 
suy ra 2 2y ≤
D u “=” x y ra
cos sin
4cos 1
4
x x
x
x
π
π
 =

⇔ ⇔ =  
− = 
 
V y max 2 2y = khi
4
x
π
=
Ta có 0 cos 1x≤ ≤ ( K y xác nh)
và 0 sin 1x≤ ≤
2
2
2 2
cos cos cos
sin sin sin
1 cos sin cos sin
x x x
x x x
x x x x y
 ≤ ≤
⇒ 
≤ ≤
⇒ = + ≤ + =
nên 1y ≥ , d u “=” x y ra khi x = 0
V y min y = 1 khi x = 0
18. Tìm giá tr l n nh t c a hàm s
( )2 2
4
3sin 1 4sin
cos
x x
y
x
−
= v i 0
6
x
π
< <
L i gi i:
Vì
1
0 0 sin
6 2
x x
π
< < ⇒ < <
2 21
0 sin 1 4sin 0
4
x x⇒ < < ⇒ − >
Áp d ng B T cô-si cho 2 s 2
3sin x và 2
1 4sin x− ta ư c
60
( )
( )
( )
( )
2 2
2 2
22
2 2
4
2 2
3sin 1 4sin
3sin 1 4sin
2
1 sin
3sin 1 4sin
2
cos
3sin 1 4sin (1)
4
x x
x x
x
x x
x
x x
+ −
≥ −
 −
⇔ ≥ − 
 
⇒ ≥ −
Chia 2 v c a (1) cho 4
cos x ( vì 4
0 cos 0
6
x x
π
< < ⇒ > )
Ta ư c
( )2 2
4
3sin 1 4sin 1
4cos
x x
y
x
−
= ≤
d u “=” x y ra 2 2 2 1
3sin 1 4sin sin
7
x x x⇔ = − ⇔ =
ta tìm ư c 0 0,
6
x
π 
∈ 
 
thì 2 1
sin
7
x =
V y
1
max
4
y =
19.
Ta có 2 2
1 2cos 1 3siny x x= + + +
2 21 1
3 6cos 2 6sin
3 2
x x= + + +
Áp d ng B T Bunhiacopski ta có :
( )
( )
2 21 1
3 6cos 2 6sin
3 2
5 55
5 6
6 6
y x x
y
 
≤ + + + + 
 
⇒ ≤ + =
D u “=” x y ra
2 2
2 2
2 2
2 2
3. 3 6cos 2. 2 6sin
3.(3 6cos ) 2.(2 6sin )
9 18cos 4 12(1 cos )
7
30cos 7 cos
30
x x
x x
x x
x x
⇔ + = +
⇔ + = +
⇔ + = + −
⇔ = ⇔ =
V y
55
max
6
y =
20.
Ta có A + B + C = π
61
tan( ) tan( )
A B C
A B C
π
π
⇔ + = −
⇒ + = −
tan tan
tan
1 tan .tan
A B
C
A B
+
⇒ = −
−
tan tan tan (1 tan .tan )A B C A B⇔ + = − −
tan tan tan tan .tan .tanA B C A B C⇔ + + = (1)
Vì ABC∆ có 3 góc nh n tan ,tan ,tan 0A B C⇒ >
Áp d ng B T cô-si cho 3 s tgA, tgB, tgC
3
tan tan tan 3 tan tan tanA B C A B C+ + ≥ (2)
t (1) và (2) ta ư c
3
tan .tan .tan 3 tan tan tanA B C A B C≥
3
2
(tan .tan .tan ) 27 tan .tan .tan
(tan .tan .tan ) 27
tan .tan .tan 3 3
A B C A B C
A B C
A B C
⇔ ≥
⇔ ≥
⇔ ≥
D u “=” x y ra khi tanA = tanB = tanC
A B C⇔ = = hay ABC∆ u
21.
Vì A + B + C =
2
π
( )
2
tan tan
2
A B C
A B C
π
π
⇔ + = −
 
⇒ + = − 
 
( )
tan tan 1
cot
1 tan tan tan
tan tan tan 1 tan tan
tan tan tan tan tan tan 1
A B
C
A B C
A B C A B
A B B C C A
+
⇔ = =
−
⇔ + = −
⇔ + + =
M t khác áp d ng B T Bunhiacôpski ta ư c
( )( )2 2 2
1 1 1 1 tan tan 1 tan tan 1 tan tanM A B B C C A≤ + + + + + + +
( )3 3 1 2 3= + =
d u b ng x y ra khi
tanA tanB = tanB tanC = tanC tanA
tan tan tanA B C⇔ = =
6
A B C
π
⇔ = = = (do A + B + C =
2
π
)
Ta có: sin sin 2sin cos 2cos cos 2cos
2 2 2 2 2
A B A B C A B C
A B
+ − −
+ = = ≤ (1)
62
Áp d ng B T :
33 3
2 2
a b a b+ + 
≥  
 
, d u “=” x y ra khi a b=
Ta có:
3
3 3
sin sin sin sin
cos
2 2 2
A B A B C + +
≤ ≤  
 
( theo(1) )
3 3
3
sin sin
cos
2 2
A B C+
⇔ ≤ (2)
Tương t :
3 3
3
sin sin
cos
2 2
B C A+
≤ (3)
3 3
3
sin sin
cos
2 2
C A B+
≤ (4)
C ng (2),(3),(4) ta có:
3 3 3 3 3 3
sin sin sin cos cos cosA B C A B C+ + ≤ + +
⇔
3 3 3
3 3 3
sin sin sin
cos cos cos
A B C
M
A B C
+ +
=
+ +
1≤
D u “=” x y ra khi
sin sin sin
3cos 1
2
A B C
A B CA B
π
= =

⇔ = = = −
=
V y max 1M = ⇔ ABC là tam giác u
63
Ph n 5 :
BÀI T P TR C NGHI M
Bài 1:
Cho a + b ≥ 1, giá tr nh nh t c a bi u th c a3
+ b3
là
A) 1 B) 1/2 C) 1/4 D) 2
Bài 2:
Giá tr l n nh t c a hàm s
2
2
1
1
x x
y
x x
+ +
=
− +
là
A.
1
3
B. 3 C.
3
2
D.5
Bài 3:
Giá tr nh nh t c a hàm s
2
2
2 1
2 1
x x
y
x x
− +
=
+ +
:
A.
1 5
2
+
B.
1 5
2
−
C.
9 4 2
7
+
D.
9 4 2
7
−
Bài 4:
Cho a + b = 1, giá tr nh nh t c a bi u th c a4
+ b4
là
A) 2 B) 1 C) 1/8 D) 1/4
Bài 5:
Cho a, b, c >0 tho mãn
1 1 2
a c b
+ = , giá tr nh nh t c a
2 2
a b c b
a b c b
+ +
+
− −
là
A.1 B.2 C.3 D.4
Bài 6:
Giá tr nh nh t c a hàm s ( )
2
2
2 1
1
x x
f x
x x
+ −
=
− +
Bài 7:
GTNN, GTLN c a hàm s
2
2
2 4 5
1
x x
y
x
+ +
=
− +
A. Min y = 1, max y = 6
B. Min y = -6, max y = -1
C. Min y =2, max y = 5
D. Min y = -5, max y = -2
A) 0 B) 2 C) 3 D) 4
Bài 8:
Cho a, b, c >0, giá tr nh nh t c a bi u th c
a b c
b c c a a b
+ +
+ + +
là
A) 1 B) 1/2 C) 3/2 D) 2
Bài 9:
64
Cho a, b, c, d >0. Giá tr nh nh t c a bi u th c
a b b c c d d a
b c d c d a d a b a b c
+ + + +
+ + +
+ + + + + + + +
là
A) 8/3 B)1/3 C) 2/3 D) 1
Bài 10:
Cho hàm s 6 5
cos siny x x= − . Giá tr l n nh t c a y là
A) -1 B) 0 C) 1/2 D) 1
Bài 11:
Giá tr l n nh t và giá tr nh nh t c a hàm s 2
1
1
x
y
x x
+
=
+ +
l n lư t là
A) max y = 1, min y = -1/3 B) max y = 2, min y = 1/2
C) max y = 1/2, min y = 1/3 D) max y = 3, min y = 1/3
Bài 12:
Gi s x, y, z là nh ng s dương thay i th a x + y + z = 1. Giá tr l n
nh t c a bi u th c
1 1 1
x y z
x y z
+ +
+ + +
là
A) 3/4 B) 1/3 C) 1 D) 2
Bài 13:
Cho các s dương x, y, z sao cho xyz = 1 và n là s nguyên dương. Giá tr nh
nh t c a bi u th c
1 1 1
2 2 2
n n n
x y z+ + +     
+ +     
     
là
A) 1 B) 2 C) 3 D) 4
Bài 14:
Cho sin sin sin 0x y z+ + = . Giá tr l n nh t c a bi u th c
2 4 6
sin sin sinP x x x= + + là
A) 0 B) 1/2 C) 1 D) 2
Bài 15:
Giá tr nh nh t c a bi u th c:
( ) ( )2 2 2 2 2 2
4cos cos sin 4sin sin sinx y x y x y x y+ − + + − là
A) 0 B) 1 C) 2 D) 4
Bài 16:
Giá tr l n nh t c a bi u th c
( )( )
( ) ( )
2 22 2
1
1 1
x y xy
x y
+ −
+ +
là
A) 0 B) 1/2 C) 1 D) 2
Bài 17:
Cho x, y, z dương và x + y + z = 1. Giá tr l n nh t c a
S = xyz(x+y)(y+z)(z+x) là
A) 8/729 B) 1/729 C) 0 D) 1/2
Bài 18:
65
Cho x, y thay i sao cho
0 3
0 4
x
y
≤ ≤

≤ ≤
.
giá tr l n nh t c a bi u th c (3-x)(4-y)(2x+3y) là
A) 1 B) 6 C) 2 D) 0
Bài 19:
Giá tr nh nh t c a bi u th c
2 2 2 2
2 12 37 6 6 18a b a b a b a b+ − − + + + + − +
A) 2 B) 5/2 C) 3 D) 5
Bài 20:
Cho x2
+ y2
= 1. Giá tr l n nh t, giá tr nh nh t c a P = x + y l n lư t là
A) max P = 1, min P = 0 B) max P = 0, min P = - 2
C) max P = 2 , min P = 1 D) max P = 2 , min P = - 2
Bài 21:
Cho x2
+ y2
= u2
+ v2
= 1.Giá tr l n nh t c a P= ( ) ( )x u v y u v− + + là
A) 2 B) 1 C) 0 D) - 2
Bài 22:
Cho ∆ ABC giá tr l n nh t c a
2 2 2
2 2 2
sin sin sin
cos cos cos
A B C
P
A B C
+ +
=
+ +
là
A) 0 B) 1/2 C) 2 D) 3
Bài 23:
Cho x, y, z là 3 góc nh n th a x + y + z = 90o.
Giá tr l n nh t c a bi u
th c
5 tan tan 5 tan tan 5 tan tanP x y y z z x= + + + + + là
A) 2 B) 3 C) 4 3 D) 2 2
Bài 24:
Cho
, 0
1
x y
x y
>

+ =
, giá tr nh nh t c a
22
1 1
P x y
x y
  
= + + +  
   
là
A) 25/2 B) 1/2 C) 1 D) 2
Hư ng d n và áp án :
1. T gi thi t 1a b+ ≥ bi n i tương ương ta ư c
3 3 2
3 3 1a b b b+ ≥ − +
mà
2
2 1 1 1
3 3 1 3
2 4 4
b b b
 
− + = − + ≥ 
 
2.B
3.D
66
4. T a + b = 1 suy ra a2
+ 2ab + b2
=1
m t khác a2
– 2ab + b2
≥ 0
t ó ta có 2 2 1
2
a b+ ≥ bình phương hai v , k t h p v i bdt
4 2 2 4
2 0a a b b− + ≥ ta ư c 4 4 1
8
a b+ ≥ .
5. T gi thi t ta có
2ac
b
a c
=
+
v y :
( )2 2
2 33 3
4
2 2 2 2 2
ac a ca b c b a b c a
a b c b a c ac
+ ++ + + +
+ = + = ≥
− −
6.C
7.A
8. t P =
a b c
b c c a a b
+ +
+ + +
Ta có 2(P + 3) = ( ) ( ) ( )
1 1 1
9a b b c c a
a b b c C a
 
+ + + + + + + ≥    + + + 
(Bunhiacopski cho 3 c p s )
Suy ra P ≥ 3/2
9. A
10. D
11. A
12. Áp d ng bunhiacopski cho ba c p s tìm ư c max = ¾
13. Ta có
1 1
2 2
n
na a
a a
+ + 
≥ ⇒ ≥ 
 
Áp d ng ta tìm ư c min = 3
14. D
15. C
16. B
17. Áp d ng côsi cho 3 s :
( ) ( ) ( ) ( )( )( )
3
3
1 3
2 3
x y z xyz
x y y z z x x y y z z x
= + + ≥
= + + + + + ≥ + + +
Nhân v theo v , bi n i tìm ư c max = 8/729
18. Có th vi t l i bi u th c ã cho thành: ( )( )( )
1
6 2 12 3 2 3
6
x y x y− − +
Áp d ng côsi cho ba s tìm ư c max = 36.
19. D
20. D
21. A
22. D
67
23. C
24. Áp d ng B.C.S cho 2 c p s (1, 1) và
1 1
,x y
x y
 
+ + 
 
Sau ó bi n i tương ương ta ư c
2 22
1 1 1 1
1
2
x y
xy x y
    
+ ≤ + + +    
    
vì
2
1
2 4
x y
xy
+ 
≤ = 
 
2
1
4
1
1 25
xy
xy
⇒ ≥
 
⇒ + ≥ 
 
v y min = 25/2
Ch de cuctri-gtln-gtnn
Ch de cuctri-gtln-gtnn
Ch de cuctri-gtln-gtnn
Ch de cuctri-gtln-gtnn
Ch de cuctri-gtln-gtnn
Ch de cuctri-gtln-gtnn
Ch de cuctri-gtln-gtnn
Ch de cuctri-gtln-gtnn
Ch de cuctri-gtln-gtnn
Ch de cuctri-gtln-gtnn
Ch de cuctri-gtln-gtnn
Ch de cuctri-gtln-gtnn
Ch de cuctri-gtln-gtnn
Ch de cuctri-gtln-gtnn
Ch de cuctri-gtln-gtnn
Ch de cuctri-gtln-gtnn
Ch de cuctri-gtln-gtnn
Ch de cuctri-gtln-gtnn
Ch de cuctri-gtln-gtnn
Ch de cuctri-gtln-gtnn
Ch de cuctri-gtln-gtnn
Ch de cuctri-gtln-gtnn
Ch de cuctri-gtln-gtnn
Ch de cuctri-gtln-gtnn
Ch de cuctri-gtln-gtnn
Ch de cuctri-gtln-gtnn
Ch de cuctri-gtln-gtnn
Ch de cuctri-gtln-gtnn
Ch de cuctri-gtln-gtnn
Ch de cuctri-gtln-gtnn
Ch de cuctri-gtln-gtnn
Ch de cuctri-gtln-gtnn
Ch de cuctri-gtln-gtnn
Ch de cuctri-gtln-gtnn
Ch de cuctri-gtln-gtnn
Ch de cuctri-gtln-gtnn
Ch de cuctri-gtln-gtnn
Ch de cuctri-gtln-gtnn
Ch de cuctri-gtln-gtnn
Ch de cuctri-gtln-gtnn
Ch de cuctri-gtln-gtnn
Ch de cuctri-gtln-gtnn
Ch de cuctri-gtln-gtnn
Ch de cuctri-gtln-gtnn
Ch de cuctri-gtln-gtnn
Ch de cuctri-gtln-gtnn
Ch de cuctri-gtln-gtnn
Ch de cuctri-gtln-gtnn

More Related Content

What's hot

Khảo sát hàm số để chứng mình bất đẳng thức
Khảo sát hàm số để chứng mình bất đẳng thứcKhảo sát hàm số để chứng mình bất đẳng thức
Khảo sát hàm số để chứng mình bất đẳng thứctuituhoc
 
Bai tap-nang-cao-toan-hinh-lop-8-bai-tap-nang-cao-toan-dai-lop-8
Bai tap-nang-cao-toan-hinh-lop-8-bai-tap-nang-cao-toan-dai-lop-8Bai tap-nang-cao-toan-hinh-lop-8-bai-tap-nang-cao-toan-dai-lop-8
Bai tap-nang-cao-toan-hinh-lop-8-bai-tap-nang-cao-toan-dai-lop-8phanvantoan021094
 
Chuyên dề dấu tam thức bậc hai
Chuyên dề dấu tam thức bậc haiChuyên dề dấu tam thức bậc hai
Chuyên dề dấu tam thức bậc haiNhập Vân Long
 
Chuyên đề luyện thi Đại học 2014
Chuyên đề luyện thi Đại học 2014Chuyên đề luyện thi Đại học 2014
Chuyên đề luyện thi Đại học 2014tuituhoc
 
Chứng minh bất đẳng thức bằng phương pháp chọn điểm rơi. (1)
Chứng minh bất đẳng thức bằng phương pháp chọn điểm rơi. (1)Chứng minh bất đẳng thức bằng phương pháp chọn điểm rơi. (1)
Chứng minh bất đẳng thức bằng phương pháp chọn điểm rơi. (1)Sao Băng Lạnh Giá
 
Chuyen de 12 tim gtnngtln tinh dt
Chuyen de 12 tim gtnngtln tinh dtChuyen de 12 tim gtnngtln tinh dt
Chuyen de 12 tim gtnngtln tinh dtHạnh Nguyễn
 
Bat dang thuc ltdh
Bat dang thuc ltdhBat dang thuc ltdh
Bat dang thuc ltdhtruongdung
 
Tai lieu luyen thi mon toan de thi dh mon toan khoi a - nam 2008
Tai lieu luyen thi mon toan   de thi dh mon toan khoi a - nam 2008Tai lieu luyen thi mon toan   de thi dh mon toan khoi a - nam 2008
Tai lieu luyen thi mon toan de thi dh mon toan khoi a - nam 2008Trungtâmluyệnthi Qsc
 
245 Đề thi đại học môn toán 1996 - 2005
245 Đề thi đại học môn toán 1996 - 2005245 Đề thi đại học môn toán 1996 - 2005
245 Đề thi đại học môn toán 1996 - 2005Anh Pham Duy
 
De cuong on tap toan 8 hoc ki 2
De cuong on tap toan 8  hoc ki 2De cuong on tap toan 8  hoc ki 2
De cuong on tap toan 8 hoc ki 2hotramy
 
15 bai toan_boi_duong_hsg_toan_l8_8383
15 bai toan_boi_duong_hsg_toan_l8_838315 bai toan_boi_duong_hsg_toan_l8_8383
15 bai toan_boi_duong_hsg_toan_l8_8383Manh Tranduongquoc
 
Tuyen tap cac_bat_dang_thuc_trong_cac_de_thi_tuyen_sing_dai_hoc(ca_hd)
Tuyen tap cac_bat_dang_thuc_trong_cac_de_thi_tuyen_sing_dai_hoc(ca_hd)Tuyen tap cac_bat_dang_thuc_trong_cac_de_thi_tuyen_sing_dai_hoc(ca_hd)
Tuyen tap cac_bat_dang_thuc_trong_cac_de_thi_tuyen_sing_dai_hoc(ca_hd)Nguyen KienHuyen
 
Can thuc [2014]
Can thuc [2014]Can thuc [2014]
Can thuc [2014]Yo Yo
 
19 phuong phap chung minh bat dang thu ccb
 19 phuong phap chung minh bat dang thu ccb 19 phuong phap chung minh bat dang thu ccb
19 phuong phap chung minh bat dang thu ccbPTAnh SuperA
 
Toan pt.de136.2011
Toan pt.de136.2011Toan pt.de136.2011
Toan pt.de136.2011BẢO Hí
 
Bdt của tran si tung
Bdt của tran si tungBdt của tran si tung
Bdt của tran si tungCam huynh
 

What's hot (19)

Khảo sát hàm số để chứng mình bất đẳng thức
Khảo sát hàm số để chứng mình bất đẳng thứcKhảo sát hàm số để chứng mình bất đẳng thức
Khảo sát hàm số để chứng mình bất đẳng thức
 
Bai tap-nang-cao-toan-hinh-lop-8-bai-tap-nang-cao-toan-dai-lop-8
Bai tap-nang-cao-toan-hinh-lop-8-bai-tap-nang-cao-toan-dai-lop-8Bai tap-nang-cao-toan-hinh-lop-8-bai-tap-nang-cao-toan-dai-lop-8
Bai tap-nang-cao-toan-hinh-lop-8-bai-tap-nang-cao-toan-dai-lop-8
 
Chuyen de bat_dang_thuc
Chuyen de bat_dang_thucChuyen de bat_dang_thuc
Chuyen de bat_dang_thuc
 
Chuyên dề dấu tam thức bậc hai
Chuyên dề dấu tam thức bậc haiChuyên dề dấu tam thức bậc hai
Chuyên dề dấu tam thức bậc hai
 
Bdt hình học
Bdt hình họcBdt hình học
Bdt hình học
 
Chuyên đề luyện thi Đại học 2014
Chuyên đề luyện thi Đại học 2014Chuyên đề luyện thi Đại học 2014
Chuyên đề luyện thi Đại học 2014
 
Chuyen de hsg
Chuyen de hsgChuyen de hsg
Chuyen de hsg
 
Chứng minh bất đẳng thức bằng phương pháp chọn điểm rơi. (1)
Chứng minh bất đẳng thức bằng phương pháp chọn điểm rơi. (1)Chứng minh bất đẳng thức bằng phương pháp chọn điểm rơi. (1)
Chứng minh bất đẳng thức bằng phương pháp chọn điểm rơi. (1)
 
Chuyen de 12 tim gtnngtln tinh dt
Chuyen de 12 tim gtnngtln tinh dtChuyen de 12 tim gtnngtln tinh dt
Chuyen de 12 tim gtnngtln tinh dt
 
Bat dang thuc ltdh
Bat dang thuc ltdhBat dang thuc ltdh
Bat dang thuc ltdh
 
Tai lieu luyen thi mon toan de thi dh mon toan khoi a - nam 2008
Tai lieu luyen thi mon toan   de thi dh mon toan khoi a - nam 2008Tai lieu luyen thi mon toan   de thi dh mon toan khoi a - nam 2008
Tai lieu luyen thi mon toan de thi dh mon toan khoi a - nam 2008
 
245 Đề thi đại học môn toán 1996 - 2005
245 Đề thi đại học môn toán 1996 - 2005245 Đề thi đại học môn toán 1996 - 2005
245 Đề thi đại học môn toán 1996 - 2005
 
De cuong on tap toan 8 hoc ki 2
De cuong on tap toan 8  hoc ki 2De cuong on tap toan 8  hoc ki 2
De cuong on tap toan 8 hoc ki 2
 
15 bai toan_boi_duong_hsg_toan_l8_8383
15 bai toan_boi_duong_hsg_toan_l8_838315 bai toan_boi_duong_hsg_toan_l8_8383
15 bai toan_boi_duong_hsg_toan_l8_8383
 
Tuyen tap cac_bat_dang_thuc_trong_cac_de_thi_tuyen_sing_dai_hoc(ca_hd)
Tuyen tap cac_bat_dang_thuc_trong_cac_de_thi_tuyen_sing_dai_hoc(ca_hd)Tuyen tap cac_bat_dang_thuc_trong_cac_de_thi_tuyen_sing_dai_hoc(ca_hd)
Tuyen tap cac_bat_dang_thuc_trong_cac_de_thi_tuyen_sing_dai_hoc(ca_hd)
 
Can thuc [2014]
Can thuc [2014]Can thuc [2014]
Can thuc [2014]
 
19 phuong phap chung minh bat dang thu ccb
 19 phuong phap chung minh bat dang thu ccb 19 phuong phap chung minh bat dang thu ccb
19 phuong phap chung minh bat dang thu ccb
 
Toan pt.de136.2011
Toan pt.de136.2011Toan pt.de136.2011
Toan pt.de136.2011
 
Bdt của tran si tung
Bdt của tran si tungBdt của tran si tung
Bdt của tran si tung
 

Viewers also liked

Cuadro contrata docente 2016 Pangoa inicial y primaria
Cuadro contrata docente 2016 Pangoa inicial y primariaCuadro contrata docente 2016 Pangoa inicial y primaria
Cuadro contrata docente 2016 Pangoa inicial y primariaRonald Jose Rodriguez Santos
 
Scrum in 5 minutes
Scrum in 5 minutesScrum in 5 minutes
Scrum in 5 minutesJa Nes AC
 
Линия для кудрявых волос Joico
Линия для кудрявых волос JoicoЛиния для кудрявых волос Joico
Линия для кудрявых волос JoicoOlesya Slavina
 
Makalah hubungan asfiksia dengan neonatus prematur
Makalah hubungan asfiksia dengan neonatus prematurMakalah hubungan asfiksia dengan neonatus prematur
Makalah hubungan asfiksia dengan neonatus prematurSeptian Muna Barakati
 
Presentazione onlus riccardo riccardi -finale
Presentazione onlus  riccardo riccardi  -finalePresentazione onlus  riccardo riccardi  -finale
Presentazione onlus riccardo riccardi -finalericcardir
 
Blended Analytics Thats Whats Next for IT Management Information Management
Blended Analytics  Thats Whats Next for IT Management  Information ManagementBlended Analytics  Thats Whats Next for IT Management  Information Management
Blended Analytics Thats Whats Next for IT Management Information ManagementEvolven Software
 
An efficient, coherent and well coordinated security sector
An efficient, coherent and well coordinated security sectorAn efficient, coherent and well coordinated security sector
An efficient, coherent and well coordinated security sectorEquip_Africa_Institute
 
Pascal triangle
Pascal trianglePascal triangle
Pascal triangleEslam Omar
 
Makalah hubungan olahraga dengan kesehatan
Makalah hubungan olahraga dengan kesehatanMakalah hubungan olahraga dengan kesehatan
Makalah hubungan olahraga dengan kesehatanSeptian Muna Barakati
 
Auto verkopen, hoe zit met het de kentekencard?
Auto verkopen, hoe zit met het de kentekencard?Auto verkopen, hoe zit met het de kentekencard?
Auto verkopen, hoe zit met het de kentekencard?Auto Verkopen
 
07 hetoadodungtrong gis[1]
07 hetoadodungtrong gis[1]07 hetoadodungtrong gis[1]
07 hetoadodungtrong gis[1]Quoc Nguyen
 
Makalah hubungan asfiksia dengan air ketuban bercampur dengan mekonium
Makalah hubungan asfiksia dengan air ketuban bercampur dengan mekoniumMakalah hubungan asfiksia dengan air ketuban bercampur dengan mekonium
Makalah hubungan asfiksia dengan air ketuban bercampur dengan mekoniumSeptian Muna Barakati
 
28 11 2014- Conferencia del C. Gobernador con el Tema “Situación de Seguridad...
28 11 2014- Conferencia del C. Gobernador con el Tema “Situación de Seguridad...28 11 2014- Conferencia del C. Gobernador con el Tema “Situación de Seguridad...
28 11 2014- Conferencia del C. Gobernador con el Tema “Situación de Seguridad...Javier Duarte de Ochoa
 

Viewers also liked (20)

Cuadro contrata docente 2016 Pangoa inicial y primaria
Cuadro contrata docente 2016 Pangoa inicial y primariaCuadro contrata docente 2016 Pangoa inicial y primaria
Cuadro contrata docente 2016 Pangoa inicial y primaria
 
Scrum in 5 minutes
Scrum in 5 minutesScrum in 5 minutes
Scrum in 5 minutes
 
Линия для кудрявых волос Joico
Линия для кудрявых волос JoicoЛиния для кудрявых волос Joico
Линия для кудрявых волос Joico
 
Makalah hubungan asfiksia dengan neonatus prematur
Makalah hubungan asfiksia dengan neonatus prematurMakalah hubungan asfiksia dengan neonatus prematur
Makalah hubungan asfiksia dengan neonatus prematur
 
Presentazione onlus riccardo riccardi -finale
Presentazione onlus  riccardo riccardi  -finalePresentazione onlus  riccardo riccardi  -finale
Presentazione onlus riccardo riccardi -finale
 
Zule
ZuleZule
Zule
 
Blended Analytics Thats Whats Next for IT Management Information Management
Blended Analytics  Thats Whats Next for IT Management  Information ManagementBlended Analytics  Thats Whats Next for IT Management  Information Management
Blended Analytics Thats Whats Next for IT Management Information Management
 
An efficient, coherent and well coordinated security sector
An efficient, coherent and well coordinated security sectorAn efficient, coherent and well coordinated security sector
An efficient, coherent and well coordinated security sector
 
Pascal triangle
Pascal trianglePascal triangle
Pascal triangle
 
Makalah hubungan olahraga dengan kesehatan
Makalah hubungan olahraga dengan kesehatanMakalah hubungan olahraga dengan kesehatan
Makalah hubungan olahraga dengan kesehatan
 
تربیت
تربیتتربیت
تربیت
 
¡Siempre el mejor precio!
¡Siempre el mejor precio!¡Siempre el mejor precio!
¡Siempre el mejor precio!
 
Makalah hpp
Makalah hppMakalah hpp
Makalah hpp
 
Auto verkopen, hoe zit met het de kentekencard?
Auto verkopen, hoe zit met het de kentekencard?Auto verkopen, hoe zit met het de kentekencard?
Auto verkopen, hoe zit met het de kentekencard?
 
07 hetoadodungtrong gis[1]
07 hetoadodungtrong gis[1]07 hetoadodungtrong gis[1]
07 hetoadodungtrong gis[1]
 
Makalah hubungan asfiksia dengan air ketuban bercampur dengan mekonium
Makalah hubungan asfiksia dengan air ketuban bercampur dengan mekoniumMakalah hubungan asfiksia dengan air ketuban bercampur dengan mekonium
Makalah hubungan asfiksia dengan air ketuban bercampur dengan mekonium
 
Online class: January 27, 2016
Online class: January 27, 2016Online class: January 27, 2016
Online class: January 27, 2016
 
PPS
PPSPPS
PPS
 
28 11 2014- Conferencia del C. Gobernador con el Tema “Situación de Seguridad...
28 11 2014- Conferencia del C. Gobernador con el Tema “Situación de Seguridad...28 11 2014- Conferencia del C. Gobernador con el Tema “Situación de Seguridad...
28 11 2014- Conferencia del C. Gobernador con el Tema “Situación de Seguridad...
 
Informe formación
Informe formaciónInforme formación
Informe formación
 

Similar to Ch de cuctri-gtln-gtnn

Chuyen de he pt
Chuyen de he ptChuyen de he pt
Chuyen de he ptTam Ho Hai
 
Chuyen%20de%20phuong%20trinh%20nghiem%20nguyen
Chuyen%20de%20phuong%20trinh%20nghiem%20nguyenChuyen%20de%20phuong%20trinh%20nghiem%20nguyen
Chuyen%20de%20phuong%20trinh%20nghiem%20nguyenTam Vu Minh
 
[Www.toan trunghoccoso.toancapba.net] toan 9 chuyen de bd hsg nang khieu
[Www.toan trunghoccoso.toancapba.net] toan 9  chuyen de bd hsg  nang khieu[Www.toan trunghoccoso.toancapba.net] toan 9  chuyen de bd hsg  nang khieu
[Www.toan trunghoccoso.toancapba.net] toan 9 chuyen de bd hsg nang khieuTam Vu Minh
 
Chuyên đề giá trị tuyệt đối
Chuyên đề giá trị tuyệt đốiChuyên đề giá trị tuyệt đối
Chuyên đề giá trị tuyệt đốiyoungunoistalented1995
 
De cuong on tap hki toan 9 bam sat gk
De cuong on tap hki toan 9 bam sat gkDe cuong on tap hki toan 9 bam sat gk
De cuong on tap hki toan 9 bam sat gkVu Van van Hieu
 
Chuyen de boi_duong_hoc_sinh_gioi_lop_12_2802
Chuyen de boi_duong_hoc_sinh_gioi_lop_12_2802Chuyen de boi_duong_hoc_sinh_gioi_lop_12_2802
Chuyen de boi_duong_hoc_sinh_gioi_lop_12_2802baolanchi
 
Các phương pháp hay giải Phuong trinh-vo-ty
Các phương pháp hay giải Phuong trinh-vo-tyCác phương pháp hay giải Phuong trinh-vo-ty
Các phương pháp hay giải Phuong trinh-vo-tyroggerbob
 
Toan cao cap 2 de thi mau 01 apr 2012
Toan cao cap 2 de thi mau 01 apr 2012Toan cao cap 2 de thi mau 01 apr 2012
Toan cao cap 2 de thi mau 01 apr 2012Vivian Tempest
 
Phuong trinh vo ty
Phuong trinh vo tyPhuong trinh vo ty
Phuong trinh vo tytututhoi1234
 
04 nguyen ham cua ham huu ti p1
04 nguyen ham cua ham huu ti p104 nguyen ham cua ham huu ti p1
04 nguyen ham cua ham huu ti p1Huynh ICT
 
Tiếp tuyến của đồ thị hàm số
Tiếp tuyến của đồ thị hàm sốTiếp tuyến của đồ thị hàm số
Tiếp tuyến của đồ thị hàm sốtuituhoc
 
04 nguyen ham cua ham huu ti p2
04 nguyen ham cua ham huu ti p204 nguyen ham cua ham huu ti p2
04 nguyen ham cua ham huu ti p2Huynh ICT
 
11 phuong phap giai pth
11 phuong phap giai pth11 phuong phap giai pth
11 phuong phap giai pthPhuc Nguyen
 
Cac phuong phap giai pt ham thuong dung
Cac phuong phap giai pt ham thuong dungCac phuong phap giai pt ham thuong dung
Cac phuong phap giai pt ham thuong dungljmonking
 

Similar to Ch de cuctri-gtln-gtnn (20)

Chuyen de he pt
Chuyen de he ptChuyen de he pt
Chuyen de he pt
 
Chuyen%20de%20phuong%20trinh%20nghiem%20nguyen
Chuyen%20de%20phuong%20trinh%20nghiem%20nguyenChuyen%20de%20phuong%20trinh%20nghiem%20nguyen
Chuyen%20de%20phuong%20trinh%20nghiem%20nguyen
 
[Www.toan trunghoccoso.toancapba.net] toan 9 chuyen de bd hsg nang khieu
[Www.toan trunghoccoso.toancapba.net] toan 9  chuyen de bd hsg  nang khieu[Www.toan trunghoccoso.toancapba.net] toan 9  chuyen de bd hsg  nang khieu
[Www.toan trunghoccoso.toancapba.net] toan 9 chuyen de bd hsg nang khieu
 
Chuyên đề giá trị tuyệt đối
Chuyên đề giá trị tuyệt đốiChuyên đề giá trị tuyệt đối
Chuyên đề giá trị tuyệt đối
 
bdt dua ve mot bien
bdt dua ve mot bienbdt dua ve mot bien
bdt dua ve mot bien
 
De cuong on tap hki toan 9 bam sat gk
De cuong on tap hki toan 9 bam sat gkDe cuong on tap hki toan 9 bam sat gk
De cuong on tap hki toan 9 bam sat gk
 
Bt daiso10-c3
Bt daiso10-c3Bt daiso10-c3
Bt daiso10-c3
 
Chuyen de boi_duong_hoc_sinh_gioi_lop_12_2802
Chuyen de boi_duong_hoc_sinh_gioi_lop_12_2802Chuyen de boi_duong_hoc_sinh_gioi_lop_12_2802
Chuyen de boi_duong_hoc_sinh_gioi_lop_12_2802
 
Các phương pháp hay giải Phuong trinh-vo-ty
Các phương pháp hay giải Phuong trinh-vo-tyCác phương pháp hay giải Phuong trinh-vo-ty
Các phương pháp hay giải Phuong trinh-vo-ty
 
Bdt dua ve mot bien
Bdt dua ve mot bienBdt dua ve mot bien
Bdt dua ve mot bien
 
Toan cao cap 2 de thi mau 01 apr 2012
Toan cao cap 2 de thi mau 01 apr 2012Toan cao cap 2 de thi mau 01 apr 2012
Toan cao cap 2 de thi mau 01 apr 2012
 
Phuong trinh vo ty
Phuong trinh vo tyPhuong trinh vo ty
Phuong trinh vo ty
 
04 nguyen ham cua ham huu ti p1
04 nguyen ham cua ham huu ti p104 nguyen ham cua ham huu ti p1
04 nguyen ham cua ham huu ti p1
 
Tiếp tuyến của đồ thị hàm số
Tiếp tuyến của đồ thị hàm sốTiếp tuyến của đồ thị hàm số
Tiếp tuyến của đồ thị hàm số
 
04 nguyen ham cua ham huu ti p2
04 nguyen ham cua ham huu ti p204 nguyen ham cua ham huu ti p2
04 nguyen ham cua ham huu ti p2
 
Chde hamsobac4
Chde hamsobac4Chde hamsobac4
Chde hamsobac4
 
Hambac4
Hambac4Hambac4
Hambac4
 
Hàm bậc 4
Hàm bậc 4Hàm bậc 4
Hàm bậc 4
 
11 phuong phap giai pth
11 phuong phap giai pth11 phuong phap giai pth
11 phuong phap giai pth
 
Cac phuong phap giai pt ham thuong dung
Cac phuong phap giai pt ham thuong dungCac phuong phap giai pt ham thuong dung
Cac phuong phap giai pt ham thuong dung
 

Recently uploaded

Chuong trinh dao tao Su pham Khoa hoc tu nhien, ma nganh - 7140247.pdf
Chuong trinh dao tao Su pham Khoa hoc tu nhien, ma nganh - 7140247.pdfChuong trinh dao tao Su pham Khoa hoc tu nhien, ma nganh - 7140247.pdf
Chuong trinh dao tao Su pham Khoa hoc tu nhien, ma nganh - 7140247.pdfhoangtuansinh1
 
BỘ ĐỀ KIỂM TRA CUỐI KÌ 2 VẬT LÝ 11 - KẾT NỐI TRI THỨC - THEO CẤU TRÚC ĐỀ MIN...
BỘ ĐỀ KIỂM TRA CUỐI KÌ 2 VẬT LÝ 11 - KẾT NỐI TRI THỨC - THEO CẤU TRÚC ĐỀ MIN...BỘ ĐỀ KIỂM TRA CUỐI KÌ 2 VẬT LÝ 11 - KẾT NỐI TRI THỨC - THEO CẤU TRÚC ĐỀ MIN...
BỘ ĐỀ KIỂM TRA CUỐI KÌ 2 VẬT LÝ 11 - KẾT NỐI TRI THỨC - THEO CẤU TRÚC ĐỀ MIN...Nguyen Thanh Tu Collection
 
NQA Lợi ích Từ ISO và ESG Tăng Trưởng và Bền Vững ver01.pdf
NQA Lợi ích Từ ISO và ESG Tăng Trưởng và Bền Vững ver01.pdfNQA Lợi ích Từ ISO và ESG Tăng Trưởng và Bền Vững ver01.pdf
NQA Lợi ích Từ ISO và ESG Tăng Trưởng và Bền Vững ver01.pdfNguyễn Đăng Quang
 
Kiểm tra chạy trạm lí thuyết giữa kì giải phẫu sinh lí
Kiểm tra chạy trạm lí thuyết giữa kì giải phẫu sinh líKiểm tra chạy trạm lí thuyết giữa kì giải phẫu sinh lí
Kiểm tra chạy trạm lí thuyết giữa kì giải phẫu sinh líDr K-OGN
 
Sáng kiến “Sử dụng ứng dụng Quizizz nhằm nâng cao chất lượng ôn thi tốt nghiệ...
Sáng kiến “Sử dụng ứng dụng Quizizz nhằm nâng cao chất lượng ôn thi tốt nghiệ...Sáng kiến “Sử dụng ứng dụng Quizizz nhằm nâng cao chất lượng ôn thi tốt nghiệ...
Sáng kiến “Sử dụng ứng dụng Quizizz nhằm nâng cao chất lượng ôn thi tốt nghiệ...Nguyen Thanh Tu Collection
 
Trích dẫn trắc nghiệm tư tưởng HCM5.docx
Trích dẫn trắc nghiệm tư tưởng HCM5.docxTrích dẫn trắc nghiệm tư tưởng HCM5.docx
Trích dẫn trắc nghiệm tư tưởng HCM5.docxnhungdt08102004
 
SÁNG KIẾN “THIẾT KẾ VÀ SỬ DỤNG INFOGRAPHIC TRONG DẠY HỌC ĐỊA LÍ 11 (BỘ SÁCH K...
SÁNG KIẾN “THIẾT KẾ VÀ SỬ DỤNG INFOGRAPHIC TRONG DẠY HỌC ĐỊA LÍ 11 (BỘ SÁCH K...SÁNG KIẾN “THIẾT KẾ VÀ SỬ DỤNG INFOGRAPHIC TRONG DẠY HỌC ĐỊA LÍ 11 (BỘ SÁCH K...
SÁNG KIẾN “THIẾT KẾ VÀ SỬ DỤNG INFOGRAPHIC TRONG DẠY HỌC ĐỊA LÍ 11 (BỘ SÁCH K...Nguyen Thanh Tu Collection
 
Chàm - Bệnh án (da liễu - bvdlct ctump) .pptx
Chàm - Bệnh án (da liễu - bvdlct ctump) .pptxChàm - Bệnh án (da liễu - bvdlct ctump) .pptx
Chàm - Bệnh án (da liễu - bvdlct ctump) .pptxendkay31
 
Sơ đồ tư duy môn sinh học bậc THPT.pdf
Sơ đồ tư duy môn sinh học bậc THPT.pdfSơ đồ tư duy môn sinh học bậc THPT.pdf
Sơ đồ tư duy môn sinh học bậc THPT.pdftohoanggiabao81
 
Thong bao 337-DHPY (24.4.2024) thi sat hach Ngoai ngu dap ung Chuan dau ra do...
Thong bao 337-DHPY (24.4.2024) thi sat hach Ngoai ngu dap ung Chuan dau ra do...Thong bao 337-DHPY (24.4.2024) thi sat hach Ngoai ngu dap ung Chuan dau ra do...
Thong bao 337-DHPY (24.4.2024) thi sat hach Ngoai ngu dap ung Chuan dau ra do...hoangtuansinh1
 
TỔNG HỢP ĐỀ THI CHÍNH THỨC KỲ THI TUYỂN SINH VÀO LỚP 10 THPT MÔN NGỮ VĂN NĂM ...
TỔNG HỢP ĐỀ THI CHÍNH THỨC KỲ THI TUYỂN SINH VÀO LỚP 10 THPT MÔN NGỮ VĂN NĂM ...TỔNG HỢP ĐỀ THI CHÍNH THỨC KỲ THI TUYỂN SINH VÀO LỚP 10 THPT MÔN NGỮ VĂN NĂM ...
TỔNG HỢP ĐỀ THI CHÍNH THỨC KỲ THI TUYỂN SINH VÀO LỚP 10 THPT MÔN NGỮ VĂN NĂM ...Nguyen Thanh Tu Collection
 
QUẢN LÝ HOẠT ĐỘNG GIÁO DỤC KỸ NĂNG SỐNG CHO HỌC SINH CÁC TRƯỜNG TRUNG HỌC CƠ ...
QUẢN LÝ HOẠT ĐỘNG GIÁO DỤC KỸ NĂNG SỐNG CHO HỌC SINH CÁC TRƯỜNG TRUNG HỌC CƠ ...QUẢN LÝ HOẠT ĐỘNG GIÁO DỤC KỸ NĂNG SỐNG CHO HỌC SINH CÁC TRƯỜNG TRUNG HỌC CƠ ...
QUẢN LÝ HOẠT ĐỘNG GIÁO DỤC KỸ NĂNG SỐNG CHO HỌC SINH CÁC TRƯỜNG TRUNG HỌC CƠ ...ThunTrn734461
 
ôn tập lịch sử hhhhhhhhhhhhhhhhhhhhhhhhhh
ôn tập lịch sử hhhhhhhhhhhhhhhhhhhhhhhhhhôn tập lịch sử hhhhhhhhhhhhhhhhhhhhhhhhhh
ôn tập lịch sử hhhhhhhhhhhhhhhhhhhhhhhhhhvanhathvc
 
Sáng kiến Dạy học theo định hướng STEM một số chủ đề phần “vật sống”, Khoa họ...
Sáng kiến Dạy học theo định hướng STEM một số chủ đề phần “vật sống”, Khoa họ...Sáng kiến Dạy học theo định hướng STEM một số chủ đề phần “vật sống”, Khoa họ...
Sáng kiến Dạy học theo định hướng STEM một số chủ đề phần “vật sống”, Khoa họ...Nguyen Thanh Tu Collection
 
30 ĐỀ PHÁT TRIỂN THEO CẤU TRÚC ĐỀ MINH HỌA BGD NGÀY 22-3-2024 KỲ THI TỐT NGHI...
30 ĐỀ PHÁT TRIỂN THEO CẤU TRÚC ĐỀ MINH HỌA BGD NGÀY 22-3-2024 KỲ THI TỐT NGHI...30 ĐỀ PHÁT TRIỂN THEO CẤU TRÚC ĐỀ MINH HỌA BGD NGÀY 22-3-2024 KỲ THI TỐT NGHI...
30 ĐỀ PHÁT TRIỂN THEO CẤU TRÚC ĐỀ MINH HỌA BGD NGÀY 22-3-2024 KỲ THI TỐT NGHI...Nguyen Thanh Tu Collection
 
10 ĐỀ KIỂM TRA + 6 ĐỀ ÔN TẬP CUỐI KÌ 2 VẬT LÝ 11 - KẾT NỐI TRI THỨC - THEO C...
10 ĐỀ KIỂM TRA + 6 ĐỀ ÔN TẬP CUỐI KÌ 2 VẬT LÝ 11 - KẾT NỐI TRI THỨC - THEO C...10 ĐỀ KIỂM TRA + 6 ĐỀ ÔN TẬP CUỐI KÌ 2 VẬT LÝ 11 - KẾT NỐI TRI THỨC - THEO C...
10 ĐỀ KIỂM TRA + 6 ĐỀ ÔN TẬP CUỐI KÌ 2 VẬT LÝ 11 - KẾT NỐI TRI THỨC - THEO C...Nguyen Thanh Tu Collection
 
30 ĐỀ PHÁT TRIỂN THEO CẤU TRÚC ĐỀ MINH HỌA BGD NGÀY 22-3-2024 KỲ THI TỐT NGHI...
30 ĐỀ PHÁT TRIỂN THEO CẤU TRÚC ĐỀ MINH HỌA BGD NGÀY 22-3-2024 KỲ THI TỐT NGHI...30 ĐỀ PHÁT TRIỂN THEO CẤU TRÚC ĐỀ MINH HỌA BGD NGÀY 22-3-2024 KỲ THI TỐT NGHI...
30 ĐỀ PHÁT TRIỂN THEO CẤU TRÚC ĐỀ MINH HỌA BGD NGÀY 22-3-2024 KỲ THI TỐT NGHI...Nguyen Thanh Tu Collection
 
bài 5.1.docx Sinh học di truyền đại cương năm nhất của học sinh y đa khoa
bài 5.1.docx Sinh học di truyền đại cương năm nhất của học sinh y đa khoabài 5.1.docx Sinh học di truyền đại cương năm nhất của học sinh y đa khoa
bài 5.1.docx Sinh học di truyền đại cương năm nhất của học sinh y đa khoa2353020138
 
BỘ ĐỀ PHÁT TRIỂN THEO CẤU TRÚC ĐỀ MINH HỌA BGD NGÀY 22-3-2024 KỲ THI TỐT NGHI...
BỘ ĐỀ PHÁT TRIỂN THEO CẤU TRÚC ĐỀ MINH HỌA BGD NGÀY 22-3-2024 KỲ THI TỐT NGHI...BỘ ĐỀ PHÁT TRIỂN THEO CẤU TRÚC ĐỀ MINH HỌA BGD NGÀY 22-3-2024 KỲ THI TỐT NGHI...
BỘ ĐỀ PHÁT TRIỂN THEO CẤU TRÚC ĐỀ MINH HỌA BGD NGÀY 22-3-2024 KỲ THI TỐT NGHI...Nguyen Thanh Tu Collection
 

Recently uploaded (19)

Chuong trinh dao tao Su pham Khoa hoc tu nhien, ma nganh - 7140247.pdf
Chuong trinh dao tao Su pham Khoa hoc tu nhien, ma nganh - 7140247.pdfChuong trinh dao tao Su pham Khoa hoc tu nhien, ma nganh - 7140247.pdf
Chuong trinh dao tao Su pham Khoa hoc tu nhien, ma nganh - 7140247.pdf
 
BỘ ĐỀ KIỂM TRA CUỐI KÌ 2 VẬT LÝ 11 - KẾT NỐI TRI THỨC - THEO CẤU TRÚC ĐỀ MIN...
BỘ ĐỀ KIỂM TRA CUỐI KÌ 2 VẬT LÝ 11 - KẾT NỐI TRI THỨC - THEO CẤU TRÚC ĐỀ MIN...BỘ ĐỀ KIỂM TRA CUỐI KÌ 2 VẬT LÝ 11 - KẾT NỐI TRI THỨC - THEO CẤU TRÚC ĐỀ MIN...
BỘ ĐỀ KIỂM TRA CUỐI KÌ 2 VẬT LÝ 11 - KẾT NỐI TRI THỨC - THEO CẤU TRÚC ĐỀ MIN...
 
NQA Lợi ích Từ ISO và ESG Tăng Trưởng và Bền Vững ver01.pdf
NQA Lợi ích Từ ISO và ESG Tăng Trưởng và Bền Vững ver01.pdfNQA Lợi ích Từ ISO và ESG Tăng Trưởng và Bền Vững ver01.pdf
NQA Lợi ích Từ ISO và ESG Tăng Trưởng và Bền Vững ver01.pdf
 
Kiểm tra chạy trạm lí thuyết giữa kì giải phẫu sinh lí
Kiểm tra chạy trạm lí thuyết giữa kì giải phẫu sinh líKiểm tra chạy trạm lí thuyết giữa kì giải phẫu sinh lí
Kiểm tra chạy trạm lí thuyết giữa kì giải phẫu sinh lí
 
Sáng kiến “Sử dụng ứng dụng Quizizz nhằm nâng cao chất lượng ôn thi tốt nghiệ...
Sáng kiến “Sử dụng ứng dụng Quizizz nhằm nâng cao chất lượng ôn thi tốt nghiệ...Sáng kiến “Sử dụng ứng dụng Quizizz nhằm nâng cao chất lượng ôn thi tốt nghiệ...
Sáng kiến “Sử dụng ứng dụng Quizizz nhằm nâng cao chất lượng ôn thi tốt nghiệ...
 
Trích dẫn trắc nghiệm tư tưởng HCM5.docx
Trích dẫn trắc nghiệm tư tưởng HCM5.docxTrích dẫn trắc nghiệm tư tưởng HCM5.docx
Trích dẫn trắc nghiệm tư tưởng HCM5.docx
 
SÁNG KIẾN “THIẾT KẾ VÀ SỬ DỤNG INFOGRAPHIC TRONG DẠY HỌC ĐỊA LÍ 11 (BỘ SÁCH K...
SÁNG KIẾN “THIẾT KẾ VÀ SỬ DỤNG INFOGRAPHIC TRONG DẠY HỌC ĐỊA LÍ 11 (BỘ SÁCH K...SÁNG KIẾN “THIẾT KẾ VÀ SỬ DỤNG INFOGRAPHIC TRONG DẠY HỌC ĐỊA LÍ 11 (BỘ SÁCH K...
SÁNG KIẾN “THIẾT KẾ VÀ SỬ DỤNG INFOGRAPHIC TRONG DẠY HỌC ĐỊA LÍ 11 (BỘ SÁCH K...
 
Chàm - Bệnh án (da liễu - bvdlct ctump) .pptx
Chàm - Bệnh án (da liễu - bvdlct ctump) .pptxChàm - Bệnh án (da liễu - bvdlct ctump) .pptx
Chàm - Bệnh án (da liễu - bvdlct ctump) .pptx
 
Sơ đồ tư duy môn sinh học bậc THPT.pdf
Sơ đồ tư duy môn sinh học bậc THPT.pdfSơ đồ tư duy môn sinh học bậc THPT.pdf
Sơ đồ tư duy môn sinh học bậc THPT.pdf
 
Thong bao 337-DHPY (24.4.2024) thi sat hach Ngoai ngu dap ung Chuan dau ra do...
Thong bao 337-DHPY (24.4.2024) thi sat hach Ngoai ngu dap ung Chuan dau ra do...Thong bao 337-DHPY (24.4.2024) thi sat hach Ngoai ngu dap ung Chuan dau ra do...
Thong bao 337-DHPY (24.4.2024) thi sat hach Ngoai ngu dap ung Chuan dau ra do...
 
TỔNG HỢP ĐỀ THI CHÍNH THỨC KỲ THI TUYỂN SINH VÀO LỚP 10 THPT MÔN NGỮ VĂN NĂM ...
TỔNG HỢP ĐỀ THI CHÍNH THỨC KỲ THI TUYỂN SINH VÀO LỚP 10 THPT MÔN NGỮ VĂN NĂM ...TỔNG HỢP ĐỀ THI CHÍNH THỨC KỲ THI TUYỂN SINH VÀO LỚP 10 THPT MÔN NGỮ VĂN NĂM ...
TỔNG HỢP ĐỀ THI CHÍNH THỨC KỲ THI TUYỂN SINH VÀO LỚP 10 THPT MÔN NGỮ VĂN NĂM ...
 
QUẢN LÝ HOẠT ĐỘNG GIÁO DỤC KỸ NĂNG SỐNG CHO HỌC SINH CÁC TRƯỜNG TRUNG HỌC CƠ ...
QUẢN LÝ HOẠT ĐỘNG GIÁO DỤC KỸ NĂNG SỐNG CHO HỌC SINH CÁC TRƯỜNG TRUNG HỌC CƠ ...QUẢN LÝ HOẠT ĐỘNG GIÁO DỤC KỸ NĂNG SỐNG CHO HỌC SINH CÁC TRƯỜNG TRUNG HỌC CƠ ...
QUẢN LÝ HOẠT ĐỘNG GIÁO DỤC KỸ NĂNG SỐNG CHO HỌC SINH CÁC TRƯỜNG TRUNG HỌC CƠ ...
 
ôn tập lịch sử hhhhhhhhhhhhhhhhhhhhhhhhhh
ôn tập lịch sử hhhhhhhhhhhhhhhhhhhhhhhhhhôn tập lịch sử hhhhhhhhhhhhhhhhhhhhhhhhhh
ôn tập lịch sử hhhhhhhhhhhhhhhhhhhhhhhhhh
 
Sáng kiến Dạy học theo định hướng STEM một số chủ đề phần “vật sống”, Khoa họ...
Sáng kiến Dạy học theo định hướng STEM một số chủ đề phần “vật sống”, Khoa họ...Sáng kiến Dạy học theo định hướng STEM một số chủ đề phần “vật sống”, Khoa họ...
Sáng kiến Dạy học theo định hướng STEM một số chủ đề phần “vật sống”, Khoa họ...
 
30 ĐỀ PHÁT TRIỂN THEO CẤU TRÚC ĐỀ MINH HỌA BGD NGÀY 22-3-2024 KỲ THI TỐT NGHI...
30 ĐỀ PHÁT TRIỂN THEO CẤU TRÚC ĐỀ MINH HỌA BGD NGÀY 22-3-2024 KỲ THI TỐT NGHI...30 ĐỀ PHÁT TRIỂN THEO CẤU TRÚC ĐỀ MINH HỌA BGD NGÀY 22-3-2024 KỲ THI TỐT NGHI...
30 ĐỀ PHÁT TRIỂN THEO CẤU TRÚC ĐỀ MINH HỌA BGD NGÀY 22-3-2024 KỲ THI TỐT NGHI...
 
10 ĐỀ KIỂM TRA + 6 ĐỀ ÔN TẬP CUỐI KÌ 2 VẬT LÝ 11 - KẾT NỐI TRI THỨC - THEO C...
10 ĐỀ KIỂM TRA + 6 ĐỀ ÔN TẬP CUỐI KÌ 2 VẬT LÝ 11 - KẾT NỐI TRI THỨC - THEO C...10 ĐỀ KIỂM TRA + 6 ĐỀ ÔN TẬP CUỐI KÌ 2 VẬT LÝ 11 - KẾT NỐI TRI THỨC - THEO C...
10 ĐỀ KIỂM TRA + 6 ĐỀ ÔN TẬP CUỐI KÌ 2 VẬT LÝ 11 - KẾT NỐI TRI THỨC - THEO C...
 
30 ĐỀ PHÁT TRIỂN THEO CẤU TRÚC ĐỀ MINH HỌA BGD NGÀY 22-3-2024 KỲ THI TỐT NGHI...
30 ĐỀ PHÁT TRIỂN THEO CẤU TRÚC ĐỀ MINH HỌA BGD NGÀY 22-3-2024 KỲ THI TỐT NGHI...30 ĐỀ PHÁT TRIỂN THEO CẤU TRÚC ĐỀ MINH HỌA BGD NGÀY 22-3-2024 KỲ THI TỐT NGHI...
30 ĐỀ PHÁT TRIỂN THEO CẤU TRÚC ĐỀ MINH HỌA BGD NGÀY 22-3-2024 KỲ THI TỐT NGHI...
 
bài 5.1.docx Sinh học di truyền đại cương năm nhất của học sinh y đa khoa
bài 5.1.docx Sinh học di truyền đại cương năm nhất của học sinh y đa khoabài 5.1.docx Sinh học di truyền đại cương năm nhất của học sinh y đa khoa
bài 5.1.docx Sinh học di truyền đại cương năm nhất của học sinh y đa khoa
 
BỘ ĐỀ PHÁT TRIỂN THEO CẤU TRÚC ĐỀ MINH HỌA BGD NGÀY 22-3-2024 KỲ THI TỐT NGHI...
BỘ ĐỀ PHÁT TRIỂN THEO CẤU TRÚC ĐỀ MINH HỌA BGD NGÀY 22-3-2024 KỲ THI TỐT NGHI...BỘ ĐỀ PHÁT TRIỂN THEO CẤU TRÚC ĐỀ MINH HỌA BGD NGÀY 22-3-2024 KỲ THI TỐT NGHI...
BỘ ĐỀ PHÁT TRIỂN THEO CẤU TRÚC ĐỀ MINH HỌA BGD NGÀY 22-3-2024 KỲ THI TỐT NGHI...
 

Ch de cuctri-gtln-gtnn

  • 1. 1 Ph n 1: C C TR TRONG I S : M t s d ng toán thư ng g p: ▼ D ng 1: ưa v d ng bình phương I. Phương pháp gi : ưa v d ng A2 ≥0, ho c A2 + c≥ c (v I c là h ng s ) d u b ng x y ra khi A=0 II. M t s bài t p ví d : Ví d 1: Tìm giá tr l n nh t c a P ( )1x x= − L i gi i: ( ) 2 1 1 1 1 2 4 4 P x x x x x   = − = − + = − − + ≤    ng th c x y ra khi 1 2 x = và 1 4 x = Do ó giá tr l n nh t c a P là 1 4 t khi 1 4 x = Ví d 2: Tìm giá tr c a x bi u th c 2 1 2 2 5x x− + có giá tr l n nh t L i gi i: Ta có: ( ) 2 2 2 2 2 5 2 3 3 1 1 32 2 5 x x x x x − + = − + ≥ ⇒ ≤ − + Do ó, khi 2x = thì b êu th c 2 1 2 2 5x x− + có giá tr l n nh t là 1 3 V í d 3: V I x,y không âm; tìm giá tr nh nh t c a bi u th c: 2 3 2 2004,5P x xy y x= − + − + L i gi i: t ,x a y b= = v I , 0a b ≥ ta có:
  • 2. 2 ( ) ( ) ( ) ( ) ( ) 2 2 2 2 22 2 2 2 2 2 2 3 2 2004,5 2 1 3 2004,5 2 1 1 2 2 2003,5 1 1 1 2 2003,5 4 2 1 1 2 2003 2003 2 P a ab b a a b a b a b a b b b a b b b a b b = − + − + = − + + + = − + + + + − +   = − − + − + + −      = − − + − + ≥    Vì ( ) 2 1 0a b− − ≥ và 2 1 0 , 2 b a b   − ≥ ∀    1a b= + 3 2 a = 2003P = ⇔ ⇔ 1 2 b = 1 2 b = V y P t giá tr nh nh t là 2003 khi 3 2 x = và 1 2 y = hay 9 4 x = và 1 4 y = III. Bài t p t gi i: 1) Tìm giá tr l n nh t c a bi u th c: 2 2 2 5 4 2P x y xy x= − − − + 2) Tìm giá tr nh nh t c a ( ) 2 2 , 2 6 12 45f x y x xy y x= − + − + 3) Cho hai s x,y tho mãn ng th c: 2 2 2 1 8 4 4 x y x + + = Xác nh x,y tích xy t giá tr nh nh t 4) Cho a là s c nh, còn x, y là nh ng s bi n thiên. Hãy tìm giá tr nh nh t c a bi u th c: A = (x– 2y + 1)2 + (2x + ay +5)2 Hư ng d n gi I và áp s : 1)Max P = 3 khi (x,y) = (1, -2) 2) ( ) ( ) 2 2 , 6 5 9 9f x y x y y= − − + + ≥ 3) Thêm 2 4 4xy x+ vào 2 v K t qu : xy t GTNN là 1 2 − khi 1 2 x = ± 1y = ± 4) 0A ≥ khi a ≠ -4, 9 5 A = khi a = -4
  • 3. 3 ▼ D ng 2: s d ng mi n giá tr c a hàm s I. Phương pháp gi : Cho y = f(x) xác nh trên D ( )0y f D∈ ⇔ phương trình ( )0y f x= có nghi m 0a y b⇔ ≤ ≤ Khi ó min y = a, max y = b II. M t s bài t p ví d : Ví d 1: Tìm Max và Min c a: 2 1 x y x = + L i gi i: T p xác nh D = R ⇒ 0y là m t giá tr c a hàm s ⇔ phương trình 0 2 1 x y x = + có 1 nghi m x∈R ⇔ phương trình 2 0 0x y y x+ = có nghi m x∈R ⇔ phương trình 2 0 0 0x y x y− + = có nghi m x∈R ⇔ 0∆ ≥ ⇔ 2 1 4 0y− ≥ ⇔ 2 4y ≤ ⇔ 1 1 2 2 y− ≤ ≤ V y Min y = 1 2 − , Max y = 1 2 Ví d 2: Xác inh các tham s a, b sao cho hàm s 2 ax 1 b y x + = + t giá tr l n nh t b ng 4, giá tr nh nh t b ng –1 L i gi i: T p xác nh D = R 0y là m t giá tr c a hàm s ⇔ phương trình 0 2 ax+b 1 y x = + có nghi m x∈R ⇔ phương trình 2 0 0ax 0y x y b− + − = có nghi m x∈R (1) • N u 0 0y = thì (1) ⇔ ax = -b có nghi m a = b = 0 ⇔ a ≠ 0 • N u 0 0y ≠ thì (1) có nghi m ⇔ 0∆ ≥ ⇔ 2 0 04( ) 0a y b y− − ≥
  • 4. 4 ⇔ 2 2 0 04 4 0y by a− + + ≥ Theo 0y t giá tr l n nh t là 4, giá tr nh nh t là –1 nên phương trình 2 2 0 04 4y by a− + + ph I có nghi m là –1 và 4 (do -1.4 = -4 < 0) 2 4 4 a− = − 4a = ± Theo nh lý Viet ta có : ⇔ 3b = 3b = V y v I a = 4, b = 3 ho c a = -4, b = 3 thì min y = -1, max y = 4 Ví d 3: Tìm giá tr l n nh t c a hàm s : 3 4 2 12 ( ) 36 x x a y x −  =  +  L i gi i: Hàm s ã cho xác nh khi ( ) 0x x a− ≥ t 2 12 ( ) 36 x x a z x −  =  +  (1) thì 4 3 y z= , 0z ≥ 0z là m t giá tr c a hàm s (1) ⇔ phương trình 0 2 12 ( ) 36 x x a z x − = + có nghi m hay phương trình 2 0 0(12 ) 12ax 36 0z x z− − − = có nghi m (2) • 0z =12 : (2) ⇔ ax = -36 có nghi m khi 0a ≠ • 0 12z ≠ : (2) có nghi m ⇔ 2 0 036 36 (12 ) 0a z z∆ = + − ≥ 2 2 0 0 2 2 0 0 2 2 0 12 0 12 0 6 36 6 36 a z z z z a a z a ⇔ + − ≥ ⇔ − − ≤ ⇔ − + ≤ ≤ + + Vì 0 0z ≥ nên 2 00 6 36z a≤ ≤ + + V y max 2 6 36z a= + + ; max 2 34 (6 36)y a= + + III. Bài t p t gi i: 1) Tìm giá tr l n nh t, nh nh t c a bi u th c: 2 2 2 2 2 2 x x y x x − + = + + 2) Tìm giá tr l n nh t, nh nh t c a bi u th c: 3 3 4 1 1 4 3 3 1 1 x x y x x + + − + = + + − + 3) Tìm giá tr nh nh t c a hàm s : 2 1 ( )f x x x x = + + , x > 0 Hư ng d n gi I và áp s :
  • 5. 5 1) Max 3 2 2y = + , Min 3 2 2y = − 2) k: 3 1x− ≤ ≤ t 2 2 3 2. 1 t x t + = + ; 2 2 1 1 2. 1 t x t − + = + v I t = tg [ ]0;1 2 ϕ ∈ Ta có 2 2 7 12 9 5 16 7 t t y t + + = − − + + Max y 9 7 y = khi x = -3; min 7 9 y = khi x = 1 0 < x ≤ 0y (1) 2 0 1 y x x x = + + ⇔ x > 0 2 2 0 02 1 0y x y x− + = (2) i u ki n (2) có nghi m là 0 2y ≥ Áp d ng Vi-et ta ch ng minh ư c 1 2 0x x y< < V y min f(x) = 2 v I x >0 ▼ Dang 3: S d ng m t s b t ng th c quen thu c ► B t ng th c Cauchy I. Ki n th c c n n m: • Cho hai s a, b ≥ 0, ta coù: ab ba ≥ + 2 D u “ =” x y ra khi ⇔ a = b • Cho n s a1, a2, … , an ≥ 0, ta có: n n n aaa n aaa ... ... 21 21 ≥ +++ D u “=” x y ra ⇔ a1 = a2 = … = an II. M t s bài t p ví d : ◦ Bi n pháp 1: Áp d ng b t ng th c tr c ti p. Ví d 1: Cho x > 0 ; y > 0 tho mãn i u ki n 2 111 =+ yx . Tìm giá tr nh nh t c a bi u th c A = yx + L i gi i: 3)Tìm nghi m c a h
  • 6. 6 Vì x > 0 ; y > 0 nên x 1 > 0 ; y 1 > 0 ; 0;0 >> yx , theo b t Cauchy có:       +≤ yxyx 11 2 11 . 1 => 4 4 11 ≥=>≤ xy xy V n d ng b t Cauchy v i hai s dương x và y ta ư c A = yx + ≥ 42.2 ≥yx = 4 ( D u “=” x y ra ⇔ x = y = 4) V y min A = 4 ( khi và ch khi x = y = 4). Nh n xét: không ph i lúc nào ta cũng có th dùng tr c ti p b t Cauchy i v i các s trong bài. Dư i ây ta s nghiên c u m t s bi n pháp bi n i m t bi u th c có th v n d ng b t Cauchy r i tìm c c tr c a nó. Bi n pháp 1 : tìm c c tr c a m t bi u th c ta tìm c c tr c a bình phương bi u th c ó. Ví d 2: Tìm giá tr l n nh t c a bi u th c : A = .3753 xx −+− L i gi i: KX : . 3 7 3 5 ≤≤ x A2 = (3x – 5) + (7- 3x) + )37).(53(2 xx −− A2 ≤ 2 + ( 3x – 5 + 7 – 3x) = 4 ( d u “=” x y ra ⇔ 3x – 5 = 7 – 3x ⇔ x = 2). V y max A2 = 4 => max A = 2 ( khi và ch khi x = 2). Nh n xét: Bi u th c A ư c cho dư i d ng t ng c a hai căn th c. Hai bi u th c l y căn có t ng không i (b ng 2). Vì v y, n u ta bình phương bi u th c A thì s xu t hi n h ng t là hai l n tích c a căn th c. n ây có th v n d ng b t ng th c Cauchy. ◦ Bi n pháp 2: Nhân và chia bi u th c v i cùng m t s khác 0. Ví d 3: Tìm giá tr l n nh t c a bi u th c A = x x 5 9− L i gi i: KX : x ≥ 9
  • 7. 7 A = x x 5 9− = 30 1 10 3 99 5 3 3 9 2 1 5 3. 3 9 = +− =       + − ≤ − x x x x x x (d u “ =” x y ra khi và ch khi 183 3 9 =⇔= − x x ). V y max A = 30 1 ( khi và ch khi x = 18). Nh n xét: Trong cách gi i trên, x – 9 ư c bi u di n thành 3. 3 9−x và khi vân d ng b t Cauchy, tích 3. 3 9−x ư c làm tr i tr thành t ng x x 3 1 3 3 9 =+ − có d ng kx có th rút g n cho x m u, k t qu là m t h ng s . Con s 3 tìm ư c b ng cách l y căn b c hai c a 9, s 9có trong bài. Bi n pháp 3: Bi n i bi u th c ã cho thành t ng c a các bi u th c sao cho tích c a chúng là m t h ng s . 1. Tách m t h ng t thành t ng c a nhi u h ng t b ng nhau. Ví d 4 : Cho x > 0, tìm giá tr nh nh t c a bi u th c : A = . 163 3 4 x x + L i gi i: A = 3x + 4 333 16 ....4 1616 x xxx x xxx x ≥+++= A ≥ 4.2 = 8 ( d u “ =” x y ra khi và ch khi 2 16 3 =⇔= x x x V y min A = 8 ( khi và ch khi x = 2). Nh n xét: Hai s dương 3x và x3 16 có tích không ph i là m t h ng s .Mu n kh ư c x3 thì ph i có x3 = x.x.x do ó ta ph i bi u di n 3x = x + x + x r i dùng b t Cauchy v i 4 s dương. 2. Tách m t h ng t ch a bi n thành t ng c a m t h ng s v i m t h ng t ch a bi n sao cho h ng t này là ngh ch o c a h ng t khác có trong bi u th c ã cho ( có th sai khác m t h ng s ). Ví d 5: Cho 0 < x < 2, tìm giá tr nh nh t c a bi u th c A = . 2 2 9 xx x + −
  • 8. 8 L i gi i: A = 1 2 2 9 + − + − x x x x A 71921 2 . 2 9 .2 =+=+ − − ≥ x x x x ( d u “=” x y ra 2 12 2 9 =⇔ − = − ⇔ x x x x x ). V y min A = 7 ( khi và ch khi 2 1 =x ). ◦ Bi n pháp 4: Thêm m t h ng t vào bi u th c ã cho. Ví d 6: Cho ba s dương x, y, z tho mãn i u ki n x + y + z = 2. Tìm giá tr nh nh t c a bi u th c : P = . 222 yx z xz y zy x + + + + + L i gi i: Áp d ng b t Cauchy i v i hai s dương zy x + 2 và 4 zy + ta ư c: x xzy zy xzy zy x == + + ≥ + + + 2 .2 4 ..2 4 22 Tương t : z yx yx z y xz xz y ≥ + + + ≥ + + + 4 4 2 2 V y zyx zyx yx z xz y zy x ++≥ ++ +      + + + + + 2 222 P ( ) 1 2 = ++ −++≥ zyx zyx (d u “=” x y ra 3 2 ===⇔ zyx ). III. Bài t p t gi i: 1) Cho x + y = 15, tìm gía tr nh nh t, giá tr l n nh t c a bi u th c: B = 34 −+− yx 2) Cho x, y, z ≥ 0 tho mãn i u ki n x + y + z = a. Tìm giá tr l n nh t c a bi u th c A = xy + yz + xz. Tìm giá tr nh nh t c a bi u th c B = x2 + y2 + z2 .
  • 9. 9 3) Cho x, y, z là các s dương tho mãn i u ki n x + y + z ≥ 12. Tìm giá tr nh nh t c a bi u th c P = . x z z y y x ++ 4) Cho a, b, c là các s dương tho mãn i u ki n a + b + c = 1. Tìm giá tr nh nh t c a bi u th c A = . )1)(1)(1( )1)(1)(1( cba cba −−− +++ 5) Cho x, y tho mãn i u ki n x + y = 1 và x > 0. Tìm giá tr l n nh t c a bi u th c B = x2 y3 . 6) Tìm giá tr nh nh t c a xy yz zx A z x y = + + v i x, y, z là các s dương và: a) 1x y z+ + = b) 2 2 2 1x y z+ + = 7) Tìm giá tr l n nh t c a 3 3 3 3 3 3 1 1 1 1 1 1 A a b b c c a = + + + + + + + + v i a, b, c là các s dương và abc = 1. 8)Tìm giá tr nh nh t, giá tr l n nh t c a A x y z xy yz zx= + + + + + bi t r ng 2 2 2 3x y z+ + = . 9) Tìm giá tr nh nh t c a 3 3x y A = + v i x + y = 4. 10) Tìm giá tr nh nh t c a 4 4 1A x x= − + Hư ng d n gi i và áp s : 1. KX : x ≥ 4, y ≥ 3 B ≥ ⇒8 min B = 8 ( khi và ch khi x = 4, y = 11 ho c x = 12, y = 3). max B2 = 16 nên max B = 4 ( khi và ch khi x = 8, y = 7). 2 .a. xy + yz + xz ≤ x2 + y2 + z2 (áp d ng b t Cauchy cho 2 s , r i c ng l i theo v ). Suy ra: 3(xy + yz + xz) ≤ ( x + y + z )2 Hay 3A ≤ a2 b. B = x2 + y2 + z2 = ( x + y + z )2 – 2( x + y + z ) B = a2 – 2A B min ⇔ A max. 3. P2 = . 222222 y xz x zy z yx x z z y y x +++++ Áp d ng b t Cauchy cho 4 s dương: .4 ... 44 222 x yz zyxx z z yx z yx y x =≥+++ Còn l i: tương t C ng v v i v l i, ta ư c P2 ≥ 4(x + y + z) – (x + y + z) = 3(x + y + z)
  • 10. 10 P2 ≥ 3.12 = 36 Min P = 6.( khi và ch khi x = y = z = 4). 4. a + b + c = 1 ⇒ 1 – a = b + c > 0. Tương t 1 – b > 0, 1 – c > 0. Có: 1 + a = 1 + (1 – b – c) = (1 – b) + (1 – c) ≥ ( )( )cb −− 112 Suy ra (1 + a)(1 + b)(1 + c) ≥ ( ) ( ) ( )222 1118 cba −−− A ≥ 8 V y min A = 8. 5. N u y ≤ 0 thì B ≤ 0. N u y > 0 thì 1 = x + y = 3125 108 108 5 33322 325 32 ≤⇒≥++++ yx yxyyyxx hay B ≤ 3125 108 Suy ra max B = 3125 108 . 6. Theo b t ng th c Cô-si 2. . 2 xy yz xy yz y z x z x + ≥ = tương t 2 yz zx z x y + ≥ ; 2 zx xy x y z + ≥ Suy ra 2A ≥ 2(x+y+z) = 2 ; min A = 1 v i 1 3 x y z= = = b) Ta có 2 2 2 2 2 2 2 2 2 2 2 x y y z z x A z x y = + + + Hãy ch ng t 2 3A ≥ . Min A = 3 v i x = y = z = 3 3 . 7. D ch ng minh ( )3 3 a b ab a b+ ≥ + v i a > 0, b > 0. Do ó: ( )3 3 1 ( ).a b ab a b abc ab a b c+ + ≥ + + = + + 1 1 1 1 ( ) ( ) ( ) ( ) max 1 1 a b c A ab a b c bc a b c ca a b c abc a b c A a b c + + ≤ + + = = + + + + + + + + = ⇔ = = = 8. ◦ Tìm giá tr l n nh t: Áp d ng b t ng th c ( ) ( )2 2 2 2 3x y z x y z+ + ≤ + + ,ta ư c ( ) 2 9x y z+ + ≤ nên
  • 11. 11 3x y z+ + ≤ (1) Ta có b t ng th c 2 2 2 xy yz zx x y z+ + ≤ + + mà 2 2 2 3x y z+ + ≤ nên 3xy yz zx+ + ≤ (2) T (1) và (2) suy ra 6A ≤ . Ta có max 6 1A x y z= ⇔ = = = . ◦ Tìm giá tr nh nh t : t x + y + z = m thì ( ) ( )2 2 2 2 2 3 2m x y z xy yz zx xy yz zx= + + + + + = + + + Do ó 2 3 2 m xy yz xz − + + = . Ta có 2 3 2 m A m − = + nên ( ) 22 2 2 3 1 4 4. 2. A m m m A = + − = + − ≥ − ⇒ ≥ − 2 2 2 1 min 2 3 x y z A x y z + + = = − ⇔  + + = , ch ng h n x = -1, y = -1, z = 1. 9. 4 3 3 2 3 3 2 3 2 3x y x y x y A + = + ≥ = = 10. Ta có x x≤ (x y ra d u b ng khi và ch khi 0x ≥ ) nên 4 4 .x x− ≥ − Do ó 4 4 1A x x≥ − + . Áp d ng b t ng th c côsi v i b n s không âm 4 4 44 1 1 1 4 4 4 1 2.x x x x x+ + + ≥ = ⇒ − + ≥ − 4 min 2 1A x= − ⇔ = và 0 1x x≥ ⇔ = . ► B t ng th c Bunhiacopski: I. Ki n th c c n n m: • Cho a, b, c, d tuỳ ý, ta có (a2 + b2 )(c2 + d2 ) ≥ (ac + bd)2 D u b ng x y ra khi: ad = bc. • Cho a1, … , an và b1, … , bn tuỳ ý, ta có: (a1 2 + … + an 2 )(b1 2 + … + bn 2 ) ≥ ( a1b1 + … + anbn)2 D u b ng x y ra khi: n n b a b a == ... 1 1 II. M t s bài t p ví d : Ví d 1: Tìm giá tr l n nh t c a : P = xx −+− 5413 L i gi i: KX : 1 ≤ x ≤ 5 Áp d ng b t Bunhiacopski có:
  • 12. 12 P2 ≤ ( 32 + 42 )(x – 1 + 5 – x) = 100 Suy ra max P = 10 khi ⇔ − = − 4 5 3 1 xx x = 25 61 . Ví d 2: Cho a, b, c > 0. Tìm min P = ba c ac b cb a + + + + + 345 . L i gi i: P = ( ) ( ) )345( 345 3453 3 4 4 5 5 ++−      + + + + + ++=++−+ + ++ + ++ + bacacb cba ba c ca b cb a = ( ) ( ) ( )[ ] )345( 345 . 2 1 ++−      + + + + + +++++ bacacb accbba ≥ ( ) ( )345345 2 1 2 ++−++ ( theo b t Bunhiacopski). Vaäy min P = ( ) ( )345345 2 1 2 ++−++ khi và ch khi 345 bacacb + = + = + . T ng quát: Cho a, b, c > 0. Ch ng minh r ng: ( ) ( )222222 2 1 zyxxzyzxyz ba c y ca b x cb a ++−++≥ + + + + + . (c ng vào v trái (x2 + y2 +z2 ) r i tr i (x2 + y2 +z2 ), sau ó áp d ng b t Bunhicopski). Ví d 3: Cho a, b, c > 0. Tìm min P = ac b cb bc ba ca + + + + + + + 433 L i gi i: P = 106 4 2 3 2 3 −      + + +      + + + +      + + + ac b cb ac ba ca P = 10 664332323 −      + ++ +      + ++ +      + ++ ac acb cb acb ba cba P = ( ) 10 211 323 −      + + + + + ++ accbba cba P = ( ) ( ) ( )[ ] ( ) 6102.21110 211 .2 2 =−++≥−      + + + + + +++++ accbba cacbba V y min P = 6 khi và ch khi (a + b)2 = (b + c)2 = (c + a)2 hay a = b = c. Cơ s :
  • 13. 13 Ch n γβα ,, sao cho: )323()(4)(3)(3 cbamacbcbacbaca ++=++=+++=+++ γβα . T ó suy ra 2,6,2 ==== mγβα . III. Bài t p t gi i: 1. Cho a, b, c > 0. Tìm giá tr nh nh t c a: a) P = ac ba cb ba ba cb + + + + + + + + 54893 . b) Q = ac ba cb ba ba cb + + + + + + + + 5243 . c) R = cba c cba b cba ca 3 8 2 4 2 3 ++ − ++ + ++ + . 2. Tìm giá tr nh nh t, giá tr l n nh t c a 2 2 A x y= + bi t r ng ( ) ( ) 22 2 2 2 2 3 2 1.x x y y+ − + − = 3. Tìm giá tr nh nh t c a : 2 2 2 a b c A b c c a a b = + + + + + v i a, b, c là các s dương và a + b + c =6. 4. Tìm giá tr nh nh t c a 2 1 2 A x x = + − v i 0 < x < 2. 5. Cho a, b, c > 0 và abc = 1 Tìm giá tr nh nh t c a ( ) ( ) ( )3 3 3 1 1 1 A a b c b a c c a b = + + + + + Hư ng d n gi và áp s : 1. Câu a và câu b làm tương t ví d 3 Câu c không th làm như ví d 3 ư c, ta làm như sau: t a + 2b + c = x a + b + 2c = y a + b + 3c = z t ó suy ra c = z – y; b = x + y – 2y; a = 5y – x – 3z. khi ó R = z y y z y x x y z yz y yzx x xy 8 88 44 1 2888442 +−−++−= − + −+ + − . R i áp d ng b t ta tìm ư c min R. 2. T gi thi t suy ra ( ) ( ) 22 2 2 2 2 4 3 0.x y x y x+ − + + = − ≤ Do ó ( )( )2 4 3 0 1 3 0 1 3.A A A A A− + ≤ ⇔ − − ≤ ⇔ ≤ ≤ min 1 0, 1. max 3 0, 3. A x y A x y = ⇔ = = ± = ⇔ = = ± 3.
  • 14. 14 Áp d ng b t ng th c Bunhiacópki cho 3 c p s Ta có ( ) ( ) ( ) 2 2 2 2 2 2a b c b c a c a b b c a c a b         + + + + + + +         + + +        ( ) ( ) 2 2 2 2 2 2 2 2 2 . 2 a b c b c a c a b b c a c a b a b c a b c a b c b c a c a b a b c a b c b c a c a b   ≥ + + + + +  + + +    ⇒ + + + + ≥ + +   + + +  + + ⇒ + + ≥ + + + Suy ra min A = 3. 4. Áp d ng b t ng th c Bunhiacopski ( )( ) ( ) 22 2 2 2 a b m n am bn+ + ≥ + Ta có: ( ) ( ) ( ) ( ) ( ) 2 2 2 2 2 2 2 2 2 2 2 1 2 1 2 2 2 2 2 2 2 1 3 2 2. 2 1 2 12min 2 3 2 2 2 4 4 2 2 A x x x x x x x x A x xA x x x x x xx         = + − + ≥ − +            − −        ⇒ ≥ + = + −= + ⇔ = ⇔ = ⇔ = − + − − ( ) 22 4 4 8 2 8 2 2 2x x x x⇔ + + = ⇔ + = ⇔ = − (chú ý x > 0). V y 3 min 2 2 2 2 2 2 A x= + ⇔ = − . 5. t 1 1 1 , ,a b c x y z = = = thì , , 0 1 x y z xyz >  = Khi ó 2 2 2 x y z A y z z x x y = + + + + + Áp d ng b t ng th c Bunhiacopski, bi n i tương ương ta ư c: ( ) ( ) ( ) ( ) 2 2 x y z x y z A y z z x x y + + + + ≥ = + + + + + M t khác theo BDT côsi ta có: 33 3x y z xyz+ + ≥ = V y
  • 15. 15 3 min 2 1 1 . x y z y z z x x y A x y z xyz x y z a b c  = = + + +  = ⇔ = =  =   ⇔ = = = ⇔ = = ► B t ng th c Bernoulli I. Ki n th c c n n m )0,1( 1 >≥ +−≥ x xx α ααα (1) D u “ =” x y ra khi x =1 II. M t s bài t p ví d : Ví d 1: Cho x, y > 0 sao cho x + y = 1. Tim giá tr nh nh t : a. P = x2 + y2 b. Q = x5 + y5 L i gi i: a. Áp d ng b t Bernoulli ta có: (2x)2 ≥ 1 – 2 + 2(2x) (2y)2 ≥ 1 – 2 + 2(2y) C ng v theo v : 4P ≥ -2 + 4(x + y) = 2 P ≥ 2 1 . V y min P = 2 1 khi và ch khi x = y = 2 1 . b. Áp d ng b t Bernoulli ta có: (2x)5 ≥ 1 – 5 + 5(2x) (2y)5 ≥ 1 – 5 + 5(2y) C ng v theo v ta có: 32Q ≥ -8 + 10(x + y) = 2 Q ≥ 16 1 V y min Q = 16 1 . Khi và ch x = y = 2 1 . T ng quát: S = xm + ym , m ≥ 1 v i x + y = 1.
  • 16. 16 *. Theo (1), v i m i 0>≥ βα , ta có: xx β α β αβ α +−≥ 1 (1’) t xtxt =⇔= ββ 1 (1’) ⇔ D u “=” x y ra khi t = 1. Ví d 2: Cho x, y > 0, sao cho x3 + y3 = 1. Tìm min P = 3 10 3 10 yx + . L i gi i: Theo (2), ta có: ( ) ( ) ( ) ( ) ( ) 2)(2. 9 10 9 2 2 2 9 10 9 10 12 2 9 10 9 10 12 333 10 3 3 33 10 3 3 33 10 3 =++−≥⇒ +−≥ +−≥ yxP yy xx V y P ≥ 9 2 1 Hay min P = 9 2 1 khi và ch khi x = y = 3 2 1 *. T (2) thay t b i 0t t , ta ư c: D u “=” x y ra khi t = t0 v i t0 là i m t giá tr nh nh t. Bài toán: Cho 0,,,;.(1.. >≥=+ dcbaybxa βαββ ) Tìm min P = αα ydxc .. + βα β α β α tt +−≥ 1 (2) ββααα β α β α tttt ..1 00 − +      −≥ (3)
  • 17. 17 t Yyd Xxc = = α α Bài toán tr thành : Cho ββ ynxm .. + = p (m,n > 0) Tìm min A = αα yx + L i gi i: Theo b t (3), ta có: ββααα ββααα β α β α β α β α yyyy xxxx .1 .1 00 00 − − +      −≥ +      −≥ C ng l i : A ≥ ( ) ( )...1 0000 ββαββααα β α β α yyxxyx −− +++      − Ch n (x0 , y0) tho mãn: ββ ynxm .. + = p n y m x βαβα −− = 00 . Khi ó: A ≥ ( ) ..1 0 00 m x yx βα αα β α β α − ++      − p. V y min A = ( ) ..1 0 00 m x yx βα αα β α β α − ++      − p khi và ch khi x = x0, y = y0. ▼ D ng 4: Áp d ng b t ng th c trong tam giác và phuơng pháp t a , vectơ. I. Phương pháp gi i: V i 3 i m A, B, C, b t kì trong m t ph ng ta có: AB BC AC+ ≥ ( ng th c khi B n m gi a A và C). • V i hai véc tơ b t kì a và b ta có: a b a b± ≤ + . ng th c khi a và b cùng hư ng ( )1 • N u ( )1 2,a a a= và ( )1 2b b b= + ( )1 ⇔ ( ) ( ) 2 2 2 2 2 2 1 1 2 2 1 2 1 2a b a b a a b b± + ± ≤ + + +
  • 18. 18 CB A ng th c x y ra khi 1 1 2 2 . . a k b a k b =  = ( )k R∈ D ng toán tìm giá tr l n nh t c a hàm s : ( ) ( )2 2 2 2 y f x a g x b= + + + v i ( ) ( ) ( ) , 0a b f x g x k k R ≠  ± = ∈ S d ng b t ng th c tam giác: gi s ( ) ( )f x g x k− = . Trong m t ph ng Oxy xét i m: ( )( ) ( )2 2 ,M f x a OM f x a⇒ = + và 2 2 ( ( ), ) ( )N g x b ON g x b− ⇒ = + . Ta có: ( ) 22 2 2 ( ) ( ) ( )MN f x g x a b k a b= − + + = + + . Vì 2 2 ( )OM ON MN y k a b+ ≥ ⇔ ≥ + + . ng th c x y ra khi M, N, O th ng hàng . ( ) . ( ) 0a f x b g x⇔ + = . V y Min 2 2 ( )y k a b= + + . II. M t s bài t p ví d : Ví d 1: Tìm giá tr nh nh t c a bi u th c 2 2 1 1, .A a a a a a R= + + + − + ⊥ ∀ ∈ L i gi i: D th y bi u th c không thay i khi thay a b i a− , do ó ch c n gi i v i 0a ≥ . • Khi 0a = : 2A = . • Khi 0a > : Xét ABC∆ có: 1 2 3 AB AM MB CM a AMC π  = = =  =   =  Theo nh lí hàm côsi: 2 2 2 1 2.1. .cos 1 . 3 AC a a a a π = + − = + − 2 1.AC a a⇒ = − + Tương t 2 1BC a a= + + , 2.AB = Khi ó: 2 2 1 1 2 2.AC BC AB a a a a A+ ≥ ⇒ + + + − + ≥ ⇔ ≥ ng th c x y x y ra khi 0a = . V y 2MinA = khi 0.a = Ví d 2: Tìm giá tr nh nh t c a: 2 2 2 2 2 2 2 2 .y x px p x qx q= − + + − + L i gi i: 3 πM
  • 19. 19 Ta có: 2 2 2 2 ( ) ( ) .y x p p x q q= − + − + Xét i m ( , ); ( , ).M x p p N x q q− − Ta có: 2 2 ( ) ( ) .MN p q p q= − + + Vì 2 2 ( ) ( ) .OM ON MN y p q p q+ ≥ ⇔ ≥ − + + Min⇒ 2 2 ( ) ( ) .y p q p q= − + + Khi , ,M N O th ng hàng ( ) ( ) 0 . p q q p q x p q x q x p q + ⇔ − + − = ⇔ = + Ví d 3: Tìm giá tr nh nh t c a: 2 2 cos 2.cos 5 cos 4.cos 8.y x x x x= − + + + + L i gi i: Trong m t ph ng Oxy , xét i m (2;1 cos ); (4,3)M x N− Ta có: (2,2 cos )MN x= + như v y .y OM MN= + Do 0 1 cos 2x≤ − ≤ nên [ ]M AB∈ v i (2,0)A và (2,2)B . Ta có: 2 2 4 3 5.OM MN ON+ ≥ = + = ng th c x y ra khi , ,O M N th ng hàng 6 4.(1 cos ) 0x⇔ − − = 1 2 cos 2 . 2 3 x x k π π⇔ = − ⇔ = ± + V y Min 5y = khi 2 2 . 3 x k π π= ± + Ví d 4: Cho 3 s th c a, b, c tho mãn h sau 2 2 2 1 2 ( ) 6 a c b b a c  + =  + + = ( ) ( ) 1 2 Tìm giá tr nh nh t c a ( ).M b c a= − L i gi i: T gi thi t ta có: 2 2 2 2 2 2 2 8a c b ab bc+ + + + = 2 2 ( ) ( ) 4 2 2 b b a c⇔ + + + = Do ( ) 2 2 (1) 2 ( 2 ) 4c a⇔ + − = Xét ( ; ); (2 ; 2 ) 2 2 b b x a c y c a+ + −
  • 20. 20 Ta có: 2x = , 2y = , . .( ).x y b c a= − Mà . .x y x y≤ cùng hư ng: 2 2 2 2 2 .( ) 2 2 2 .( ) 2.( ) 1 2 2 10 b b b a ca c b a c a c a c c c b + = −+ +  ⇔ = ⇔ + = − + ⇒ + = −  = (do (1) và (2) ) 2 2 2 2 10 2 10 1 3 1 3 1 ( , , ) ( , 10, );( , 10, ) 10 10 10 1010 2 10 1 b a c a c a b c b a c a c  =   + = −   + = ⇔ ⇒ = − − −  = −   + =   + = Max⇒ ( ) 4M b c a= − = khi ( , , )a b c như trên. III. Bài t p t gi i: 1)Tìm giá tr l n nh t và giá tr nh nh t c a hàm s 4 4 2 1 sin cos 2cos 2y x x x= + + + + 2)Tìm giá tr nh nh t c a hàm s : 2 2 1 3 1y x x x x= − + + − + 3)Tìm giá tr l n nh t và giá tr nh nh t c a hàm s : 4 4 2 2 4 4 2 2 2 1 x y x y y y x y x     = + − + −        4)Tìm giá tr nh nh t c a hàm s : ( ) ( ) ( )2 2 2 2 2 1 2 3 1 1 2 3 1 1f x x x x x x x= − + + + + + + − − + Hư ng d n gi và áp s : 1. Ta có: ( ) ( ) 2 22 2 1 1 cos 1 1 cosy x x= + − + + + x y O 1 1 2 B N 2 A M
  • 21. 21 Xét i m M(1, 1-cos2 x), N(2, 2) ta có: ( )2 1,1 cosMN x= + Ta có: y = OM + MN V i M thu c o n [AB] v i A(1, 0) và B(1, 1) Ta có miny = ON = 2 2 2 2 2 2+ = D u “=” x y ra khi O, M, N th ng hàng ( )2 2 2 1 cos 0x− − = 2 cos 0x⇔ = Và maxy = OA + AN = 1+ 2 1 2 1 5+ = + D u “=” x y ra khi M trùng v i A 2 cos 1x⇔ = 2. Ta có: 22 1 3 3 1 2 4 2 4 y x x    = − + + − +        Xét i m 1 3 , 2 2 M x   −     và 3 1 , 2 2 N x   − −     Hai i m M, N n m hai bên Ox. Ta có: y = OM + ON ≥ MN 2 2 3 1 3 1 min 2 2 2 2 3 1 min 2 2 4 4 y MN y     = = − + + +             = + =    D u “=” x y ra khi M, O, N th ng hàng: ( ) 1 1 3 3 0 2 2 2 2 3 1 2 3 1 x x x x    − − − − =        ⇔ + = ⇔ = − 3. 4 4 2 2 4 4 2 2 2 1 x y x y y y x y x     = + − + −        ch n 2 2 2 2 x y u y x = + [ )2;u ∈ +∞ hàm s y(x) tr thành f(u) = u2 – 2u – 3 phác h a th hàm f(u) trong mi n [ )2;+∞ ta thu ư c k t qu : max f(u) không t n t i min f(u) = f(2) = -3 O ( )f u u M 2 3−
  • 22. 22 C ' C M ' MA B C ' C M ' M V y: max y(x) không t n t i min y(x) = -3 t ư c khi 2 2 2 2 2 x y y x + = 2 2 1 x y ⇒ = ⇒ m i i m (x; y) thu c 2 ư ng phân giác y = x và y = -x (tr g c O(0; 0)) 4. Hàm s f(x) có th vi t l i dư i d ng: ( ) ( ) 2 22 2 22 3 1 3 1 1 2 2 2 2 f x x x x x x x        = + − + + + + + − + +                (1) Xét trên m t ph ng t a các i m ( ) 3 1 3 1 0,1 , , , , 2 2 2 2 A B C     − − −           Và i m M(x,x) n m trên ư ng phân giác th nh t. D th y ABC là tam giác u, v i tâm là g c t a . Theo công th tính kho ng cách gi a hai i m trên m t ph ng t a , ta có v ph i c a (1) chính là MA + MB + MC. B : N u ABC là tam giác u, thì v i m i i m M c a m t ph ng tam giác, ta luôn có MA + MB + MC ≥ OA + OB + OC, trong ó O là tâm tam giác u. Ch ng minh: N u M là i m trong tam giác. Xét phép quay R(A,600 ), khi ó ' ' M M B C C C A A → → → → ⇒ MC = M’C’, MA = MM’ V y MA + MB + MC = MM’ + MB + M’ C’ ≥ BC’ M t khác n u g i O là tâm tam giác u ABC thì OA + OB + OC = BC’ ⇒ MA + MB + MC ≥ OA + OB + OC
  • 23. 23 N u M ngoài tam giác, ch ng minh tương t . Theo b ta có f(x) ≥ 3 (do OA = OB = OC) V y min f(x) = 3. ▼ Dang 5 :Phương pháp s d ng th hàm s : I. Phuơng pháp gi i: Phương pháp này thư ng dùng tìm c c tr c a các hàm s sau: 2. Các hàm s qui v tam th c b c hai. 3. Các hàm s ch a d u giá tr tuy t i. 4. Các bài toán chuy n ư c thành toán hình h c b ng cách dùng công th c dài o n th ng: 2 2 ( ) ( )A B A BAB x x y y= − + − . ây là các bài toán mà trong ó ( )f x cho dư i d ng căn b c hai mà làm dư i căn bi u di n ư c thành dài m t o n th ng nào ó. ây là ưu th c a phương pháp th . 5. Các hàm s ( , )u x y v i ,x y tho mãn trư c i u ki n. II. M t s bài t p ví d : Ví d 1: Tìm giá tr l n nh t và giá tr nh nh t c a các hàm s sau: 21 1 ) ( ) ( ) 3.( ) 1. 3 1 ) ( ) .(2 sin ).( sin ). 15 x x a y x x b y x x b x + + = + + = + − L i gi i: a) t 1 1 2. x u x x x + = = + ≥ Hàm ( )y x tr thành: 2 ( ) 3 1.y u u u= + + Theo th hàm ( )y u trên [2; ).+∞ Ta ư c max ( )y u = không có. miny(u)=y(2)=11. b) t sin 1 1.u x u= ⇒ − ≤ ≤ Hàm ( )y x thành 1 ( ) .(2 ).( ). 15 y u u b u= + − D a vào th ta có k t qu : max ( ) (1) 1 7 min ( ) ( 1) 15 y u y y u y = = = − =
  • 24. 24 Khi 2 1. 1 0 1. x x x ≥ − ≥ ⇔  ≤ − Khi 2 1 0 1 1.x x− ≤ ⇔ − ≤ ≤ Khi 1x ≤ − ho c 1.x ≥ Khi 1 1.x− ≤ ≤ Khi 1x ≥ ho c 1.x ≤ − Khi 1 1.x− ≤ ≤ Khi 3x ≤ − ho c 1.x ≥ − Khi 3 1.x− ≤ ≤ − Khi 4 3.x− ≤ ≤ − Khi 4 3.x− ≤ ≤ − Ví d 2: Tìm giá tr nh nh t c a: 2 2 ( 1) .y x x= + − L i gi i: Tao có 2 1.y x x= + − G i 2 1 1y x x= + − và 2 2 1y x x= − + + thì 1 2 y y y  =   V th c a y ta th y min ( 1) 1.y y= − = − Ví d 3: Tìm giá tr l n nh t và giá tr nh nh t c a: ( )( )2 2 6 9 2 9y x x x x x= + + + + + trong 5 [ 4; ] 4 − − . L i gi i: Ta có: ( 3)( 1).y x x x= + + + Do ( 3).( 1) ( 3).( 1) x x x x + + − + + Ta ch xét nh ng giá tr c a 5 4; . 4 n   ∈ − −   Ta ư c 2 2 5 3 3 3 x x y x x  + + =  − − − V th 2 1 5 3y x x= + + 2 2 3 3y x x= − − − D a vào th : 2 1 3 3 max ( ) . 2 4 min ( 3) 3. y y y y = − = − = − = − 2 2 1 1 x x y x x  + −  ⇒ =   + − 2 2 1 1 x x y x x  + − ⇒ =  − + + Khi 4 3.x− ≤ ≤ − Khi 5 3 . 4 x− ≤ ≤ − Y2 Y1
  • 25. 25 III. Bài t p tương t : 1.Tìm giá tr l n nh t và giá tr nh nh t c a 1 3 2 2y x x x= − + − − + v i 2 4.x− ≤ ≤ 2.Tìm giá tr nh nh t c a: 2 2 4 12 13 4 28 53.y x x x x= − + + − + 3.Tìm giá tr l n nh t và nh nh t c a hàm s : 2 5u x y= − + + bi t ,x y tho : 2 2 36 16 9.x y+ = Hương d n và áp s : 1. V i 2 1x− ≤ ≤ − thì (1 ) (3 ) (2 2) 6.y x x x= − + − − − = V i 1 1x− ≤ ≤ thì 4 2.y x= − + V i 1 3x≤ ≤ thì 2 .y x= − V i 3 4x≤ ≤ thì 6.y = − Ta v th c a hàm s 1 3 2 2y x x x= − + − − + v i 2 4.x− ≤ ≤ T th max 6 2 1.y x= ⇔ − ≤ ≤ − min 6 3 4.y x= − ⇔ ≤ ≤ 2. Ta có: 2 2 2 2 (2 3) (0 2) (2 7) (0 2) .y x x= − + − + − + − Trên m t ph ng to Oxy , xét (2 ,0)M x , (3,2)A , (7,2)B như v y .y MA MB= + ,A B n m cùng phía so v i Ox nên l y 'A i x ng A qua .Ox 'A B c t Ox t i H ta có: '(3, 2)A − 2 2 ' ' (7 3) (2 2) 4 2.y MA MB MA MB A B= + = + ≥ = − + + = ng th c x y ra khi 5 2 5 . 2 M H x x≡ ⇔ = ⇔ = V y min 4 2y = khi 5 . 2 x =
  • 26. 26 3. T i u ki n 2 2 2 2 2 36 16 9 (6 ) (4 ) 3 .x y x x+ = ⇔ + = t 1 6 6 4 1 4 x X x X y Y y Y  ==  ⇔  =  =  Ta có 2 2 2 3 .X Y+ = (1) (1) là phương trình ư ng tròn ( )» trong h tr c to Oxy có tâm O bán kính 3.R = Hàm 1 1 2 5 5. 3 4 u x y u X Y= − + + ⇔ = − + + 4 4( 5). 3 Y X u⇔ = + − ta g i phương trình này là phương trình ư ng th ng d ư ng th ng luôn song song v i ư ng th ng 4 3 Y X= và c t Oy t i (0;4( 5)).P u − -Ta v hai ư ng th ng 1 2,Y Y song song v i ư ng th ng 4 3 Y X= và ti p xúc ( )» . - 1 2,Y Y c t Oy l n lư t t i N và M khi ó maxu là giá tr xác nh khi P N≡ hay 4( 5)m naxu= − trong ó (0; ).M m Min u xác nh khi P N≡ t c là 4(min 5)n u= − trong ó (0; )N m do M i x ng N qua O nên .m u= − -K 1OH Y⊥ l y 1OH Y⊥ (3;0), (3;4),A B OAB OHM∆ = ∆ 5.m OM OB⇒ = = = Khi ó 25 max . 5 4(max 5) 4 5 4(min 5) 15 min . 4 u u u u  == −  ⇔  − = −  = 
  • 27. 27 Ph n 2: C C TR TRONG HÌNH H C PH NG ▼ D ng 1: V n d ng quan h gi a ư ng xiên và ư ng vuông góc, quan h gi a ư ng xiên và hình chi u. I. Ki n th c c n nh : II. M t s bài t p ví d : Ví d 1: Cho tam giác ( 90 ).o ABC A = M là i m chuy n ng trên c nh BC . V , ( , ).MD AB ME AC D AB E AC⊥ ⊥ ∈ ∈ Xác nh v trí c a i m M o n th ng DE có dài nh nh t. L i gi i: V ( )AH BC H BC⊥ ∈ H c nh và AH không i. T giác AEMD có 90o A E D= = = nên AEMD là hình ch nh t. Suy ra DE AM= mà AM AH≥ (không i) D u " "= x y ra .M H⇔ ≡ Ví d 2: Cho tam giác .ABC Qua nh A c a tam giác hãy d ng ư ng th ng d c t c nh BC sao cho t ng các kho ng cách t B và t C n d có giá tr nh nh t. L i gi i: G i M là giao i m c a d và c nh .BC V , ( ; )BH d CK D H K d⊥ ⊥ ∈ MAB MAC ABCS S S+ = . . 2 2 ABC BH AM CK AM S+ = 2 ABCS BH CK AM + = BH CK+ nh nh t 2 ABCS AM ⇔ nh nh t AM⇔ nh nh t Gi s AB AC≤ thì trong hai ư ng xiên ,AM AC ư ng xiên AC có hình chi u không nh hơn, do ó AM AC≤ (h ng s ) D u " "= x y ra .M C⇔ ≡ Ví d 3: Ta có AH d⊥ , A d∉ , B d∈ ,C d∈ , H d∈ . a) .AB AH≥ D u " "= x y ra khi .B H⇔ ≡ b) .AB AC BH HC≤ ⇒ ≤
  • 28. 28 Cho hình bình hành ABCD .Qua A v ư ng th ng d không c t hình bình hành. G i ', ', 'B C D l n lư t là hình chi u vuông góc c a các i m , ,B C D trên ư ng th ng d . Xác nh v trí c a ư ng th ng d t ng ' ' 'BB CC DD+ + có giá tr l n nh t. L i gi i: G i O là giao i m c a AC và BD . 'O là hình chi u vuông góc c a O trên d . ' , 'DD d BB d⊥ ⊥ ' 'DD BB⇒ ' 'DD BB⇒ là hình thang. Mà ' , 'OO d DD d⊥ ⊥ ' 'OO DD⇒ và O là trung i m BD ( ABCD là hình bình hành). Do ó 'OO là ư ng trung bình c a hình thang ' 'DD B B ' ' ' ' ' 2. ' 2 BB DD OO BB DD OO + ⇒ = ⇒ + = . ' , ' ' 'OO d CC d OO CC⊥ ⊥ ⇒ và O là trung i m AC .( ABCD là hình bình hành). Do ó 'OO là ư ng trung bình c a 'ACC ' ' ' 2. ' 2 CC OO CC OO⇒ = ⇒ = A d∈ và 'OO d⊥ nên 'OO OA≤ Do ó ' ' ' 4. ' 4.BB CC DD OO OA+ + = ≤ (không i) D u " "= x y ra "O A d⇔ ≡ ⇔ vuông góc AC t i A . Ví d 4: Cho n a ư ng tròn ( ; )O R ư ng kính .AB M là i m trên n a ư ng tròn. Xác nh v trí M : a) Di n tích tam giác MAB l n nh t. b) Chu vi tam giác MAB l n nh t. L i gi i: V , .MH AB H AB⊥ ∈ a) . 2 MAB MH AB S = .MH R= Ta có , .MH AB O AB⊥ ∈ Do ó MH OM R≤ = Nên 2 MABS R≤ (không i) D u " "= x y ra H O M⇔ ≡ ⇔ là trung i m AB b) 90o AMB = ( AMB là góc n i ti p ch n n a ư ng tròn) MAB vuông t i M có . .MH AB MH AB MA MB⊥ ⇒ = MAB vuông t i M theo nh lí Pitago có: 2 2 2 2 4 .MA MB AB R+ = = ,MABP MA MB AB= + + AB không i 2 2 2 ( ) 2 .MA MB MA MB MA MB+ = + +
  • 29. 29 Do ó MABP l n nh t MA MB⇔ + l n nh t 2 ( )MA MB⇔ + l n nh t .MA MB⇔ l n nh t MABS⇔ l n nh t M⇔ là trung i m AB (câu a) Ví d 5: Cho n a ư ng tròn ( )O ư ng kính 2 .AB R= K hai ti p tuy n Ax, By c a n a ư ng tròn ( )O và ti p xúc v i ( )O t i i m M c t Ax t i D c t By t i E. Xác nh v trí c a M trên n a ư ng tròn ( )O sao cho: a) AD BE+ t giá tr nh nh t. b) .OD OE t giá tr nh nh t. L i gi i: a) V , .DH By H By⊥ ∈ T giác ADHB có 90O A B H= = = nên ADHB là hình ch nh t 2DH AB R⇒ = = Ta có ,AD MD BE ME= = (tính ch t hai ti p tuy n c a ( )○ c t nhau t i m t i m). Do ó AD BE MD ME DE+ = + = mà DE DH≥ (vì ,DH By E By⊥ ∈ ) Do v y 2AD BE R+ ≥ (không i) D u " "= x y ra E H DE AB⇔ ≡ ⇔ OM AB M⇔ ⊥ ⇔ là trung i m AB . b) DA và DM là ti p tuy n c a( )○ OD⇒ là phân giác AOM . Tương t OE là phân giác MOB . AOM và MOB k bù. Do ó 90o EOD = ODE vuông t i O , OM DE⊥ nên . .OD OE OM DE= . .OD OE R DE= .OD OE nh nh t DE⇔ nh nh t M⇔ là trung i m AB (câu a). ▼ Dang 2: V n d ng các b t ng th c trong tam giác và quy t c các i m : I. Ki n th c c n n m: • Tam giác ABC có a) .AB AC BC AB AC− < < + b) .ABC ACB AC AB≤ ⇔ ≤ • Tam giác ABC và tam giác ' ' 'A B C có ' ', ' 'AB A B AC A C= = thì: ' ' '.BC B C A A≤ ⇔ ≤ • Quy t c ba i m , ,A B C . a) .BC AB AC≤ + D u" "= x y ra [ ]A BC⇔ ∈ b) .BC AB AC≥ − D u" "= x y ra , ,A B C⇔ th ng hàng.
  • 30. 30 Quy t c n i m 1 2; ;...; nA A A Ta có 1 1 2 2 3 3 4 1...n n nA A A A A A A A A A−≤ + + + + D u " "= x y ra 1 2 1; ;...; ;n nA A A A−⇔ th ng hàng và s p x p theo th t ó. II. M t s bài t p ví d : Ví d 1: Cho hai i m A và B n m trong n a m t ph ng b là ư ng th ng d ,hai i m ,M N thu c d và dài MN không i. Xác nh v trí hai i m ,M N ư ng g p khúc AMNB t giá tr nh nh t. L i gi i D ng hình bình hành 'BNMB (hình bên) 'BB MN a⇒ = = (không i); ', 'NB MB B= c nh. G i 'A là i m i x ng c a A qua ư ng th ng d . Ta có 'AM A M= , 'A c nh. Xét ba i m ', , 'A M B ta có ' ' ' 'A M MB A B+ ≥ Do ó ' 'AM MN NB A M MN MB+ + = + + ( ' ')A M MB MN= + + ' 'A B a≥ = không i D u " "= x y ra [ ' '].M A B⇔ ∈ Ví d 2: Cho góc nh n xOy . A là i m n m trong góc ó. Hãy tìm trên hai tiaOx và Oy l n lư t hai i m B vàC sao cho chu vi tam giác ABC nh nh t. L i gi i: G i 1A và 2A l n lư t là i m i x ng c a A qua hai tiaOx vàOy . A c nh, xOy c nh nên 1A và 2A c nh. Theo tính ch t i x ng tr c ta có: 1 ;AB A B= 2 .AC A C= ABCP AB BC AC= + + 1 2A B BC A C= + + Xét các i m 1 2, , ,A B C A ta có 1 2 1 2A B BC A C A A+ + ≥ Do ó 1 2ABCP A A≥ (không i) D u " "= x y ra 1 2, , ,A B C A⇔ th ng hàng và s p x p theo th t ó. Ví d 3: Cho hình vuông ABCD . , , ,M N P Q là nh c a t giác MNPQ l n lư t thu c các c nh , , ,AB BC CD DA ( MNPQ g i là t giác n i ti p hình vuông). Tìm i u ki n t giác MNPQ có chu vi nh nh t. L i gi i: G i , ,E F G l n lư t là trunh i m c a các o n th ng , , .MQ MP NP AMQ vuông góc t i A có AE là trung i m nên 1 2 AE MQ= 2 .MQ AE⇒ =
  • 31. 31 Tương t 2NP GC= M t khác ,EF FG l n lư t là ư ng trung bình c a các tam giác MPQ và NPM nên 1 2 EF PQ= và 1 2 FG MN= Suy ra 2PQ EF= và 2 .MN FG= Do ó MNPQP MN NP PQ QM= + + + 2 2 2 2FG GC EF AE= + + + 2( )AE EF FG GC AC= + + + ≥ (không i ) (Xét các i m , , , ,A E F G C ) D u " "= x y ra , , , ,A E F G C⇔ th ng hàng. MN AC PQ⇔ và .MQ BD NP Khi ó MNPQ là hình ch nh t. Ví d 4: Cho ư ng tròn( ; )O R ư ng kính AB c nh,C là m t i m c nh n m gi a A và .O M di ng trên ư ng tròn( ; ).O R Tìm v trí c a M trên ( ; )O R tương ng lúc dài CM l n nh t, nh nh t. L i gi i: Xét ba i m , ,C O M ta có OM CO CM CO OM− ≤ ≤ + OA OM OB R= = = Do ó CA CM CB≤ ≤ CM CB≤ (không i) D u " "= x y ra M B⇔ ≡ V y khi M B≡ thì o n th ngCM có dài l n nh t. M t khácCM CA≥ (không i) D u " "= x y ra M A⇔ ≡ V y khi M A≡ thì o n th ng CM có dài nh nh t. Ví d 5: Cho hai ư ng tròn ngoài nhau( ; )O R và( '; ').O R A n m trên ư ng tròn( )O , B n m trên ư ng tròn( ').O Xác nh v trí các i m ,A B o n th ng AB có dài l n nh t, nh nh t. L i gi i: ( ')OO c t( )O t i ,C D và c t( ')O t i , .E F Xét ba i m , ',A O B , ta có ' ' ' 'O A O B AB O A O B− ≤ ≤ + Xét ba i m , , 'O A O , ta có ' ' 'O O OA O B OA OO− ≤ ≤ + Mà OA OC OD R= = = và ' ' ' 'O B O E O F R= = = Do ó ' ' ' 'OO OD O E AB OC OO O F− − ≤ ≤ + + DE AB EF⇒ ≤ ≤ * AB EF≤ (không i) D u " "= x y ra ,A C⇔ ≡ B F≡
  • 32. 32 V y AB có dài l n nh t khi A C≡ và B F≡ * AB DE≥ (không i) D u " "= x y ra A D⇔ ≡ và B E≡ V y AB có dài nh nh t khi A D≡ và B E≡ . ▼ Dang 3: V n d ng b t ng th c trong ư ng tròn. I. Ki n th c c n nh : - ư ng kính dây cung l n nh t c a ư ng tròn. - Trong ư ng tròn ( )O : AB và CD là hai dây cung, H và K l n lư t là hình chi u vuông góc trên AB và CD . Ta có OH OK AB CD≥ ⇔ ≤ AB CD AOB COD⇔ ≤ ⇔ ≤ Ví d 1: Cho ư ng tròn ( ; );O R AC là ư ng kính.BD là dây cung c a ( ; )O R và BD vuông góc v i AC . Xác nh v trí c a dây BD di n tích t giác ABCD l n nh t. L i gi i AB CD⊥ (gt) Nên 1 . . 2 ABCDS AC BD R BD= = Mà BD là dây cung c a ( ; )O R do ó 2BD R≤ V y 2 2ABCDS R≤ . D u " "= x y ra BD là ưòng kính c a ( )O . Ví d 2: Cho n a ư ng tròn ( ; )O R ư ng kính AB . M là i m di ng trên n a ư ng tròn. Qua M v ti p tuy n v i ư ng tròn, g i D,C l n lư t là hình chi u, c a A; B trên ti p tuy n y. Xác nh v trí c a i m M di n tích cùa t giác ABCD có giá tr l n nh t. L i gi i Ta có AD DC⊥ (gt) BC DC⊥ (gt) AD BC⇒ ABCD⇒ là hình thang mà 90o D = nên ABCD là hình thang vuông. OM DC⊥ nên OM AD và O là trung i m AB Nên OM là ư ng trung bình c a hình thang ABCD 2 AD BC OM + ⇒ = Do ó . . 2 ABCD AD BC S DC OM DC + = = V AE BC⊥ . T giác ADCE là hình ch nh t ( 90 )O ADC DCE AEC= = = DC AE⇒ = 90O AEC = E⇒ thu c ư ng tròn ư ng kính AB.
  • 33. 33 AE⇒ là dây cung c a ư ng tròn ( )O . 2DC R⇒ ≤ (trong ư ng tròn ư ng kính là dây cung l n nh t) Do ó 2 .2 2ABCDS R R R≤ = D u " "= x y ra AE⇔ là ư ng kính c a ( )O OM AB M⇔ ⊥ ⇔ là trung i m AB . Ví d 3: Cho tam giác u ABC n i ti p trong ư ng tròn ( ; )O R . M là i m di ng trên trên ( )O . Xác nh các v trí c a i m M t ng MA MB MC+ + t giá tr l n nh t. L i gi i Xét M thu c cung BC. Trên dây MA l y i m D sao cho MD MB MBD= ⇒ cân. 60o BMA BCA= = (hai góc n i ti p cùng ch n AB ) Do dó MBD u. ,BD MB⇒ = 60o DBM = 60o ABD ABC DBC DBC= − = − 60o MBC MBD DBC DBC= − = − Suy ra ABD MBC= . Xét MBC và DBA có MB BD= , MBC ABD= , BC AB= ( ABC u) Do ó MBC = DBA(c.g.c) Suy ra MC DA= Ta có MA MD DA MB MC= + = + 2.MA MB MC MA⇒ + + = . MA là dây cung c a ( ; )O R 2MA R⇒ ≤ ( ư ng kính là dây cung l n nh t c a ư ng tròn) Do ó 4MA MB MC R+ + ≤ (không i) D u " "= x y ra MA⇔ là ư ng kính c a ( )O M⇔ là trung i m cung BC. L p lu n tương t ta có ba v trí MA MB MC+ + t giá tr l n nh là trung i m các cung BC; AC; AB. Ví d 4: Cho ư ng tròn ( ; )O R ; BC là dây cung c nh ( 2BC R≠ ). A là i m chuy n ng trên cung l n BC. Xác nh v trí c a A chu vi tam giác ABC l n nh t. L i gi i .ABCP AB AC BC= + + BC không i. Trên tia i tia AB l y i m D sao cho AD AC= ADC cân t i A 2BAC ADC⇒ = BAC không i ADC⇒ không i. BDC không i, BC c nh
  • 34. 34 D⇒ thu c cung ch a góc có s o 1 4 s BC c a ( )O d ng trên o n th ng BC. ABCP l n nh t ( )max ( )maxAB AC AB CD⇔ + ⇔ + maxBD⇔ ⇔ BD là ư ng kính c a cung ch a góc nói trên. Khi ó 90o BDC = . Mà 90o ABC BDC ACB ACD+ = + = BDC ACD= ( )AC AD= Do ó ABC ACB AB AC= ⇔ = ⇔ A là trung i m cung l n BC. Ví d 5 : Cho ư ng tròn ( ; )O R .A i m c nh trong ư ng tròn ( A O≠ ). Xác nh v trí c a di m B trên ư ng tròn ( )O sao cho OBA l n nh t. L i gi i V dây BC c a ư ng tròn ( )O qua A. OBC cân ( )OB OC= 180 2 o BOC OBC − = v OH BC⊥ ( )H BC∈ A BC∈ nên OH OA≤ (không i) D u " "= x y ra H A⇔ ≡ AB OA⇔ ⊥ t i A. Ta có OBAl n nh t BOC⇔ nh nh t BC⇔ nh nh t ⇔ dây BC nh nh t ⇔ OH l n nh t H A⇔ ≡ AB OA⇔ ⊥ t i A. ▼ D ng 4:V n d ng b t ng th c i s I. Ki n th c c n n m: ● B t ng th c côsi cho 2 s dương: Cho 2 s dương a và b ta có: 2 a b ab + ≥ D u “=” x y ra khi và ch khi a=b . ● B t ng th c Bunhiacopxki Sraxo (B.C.S): Cho 4 s th c a,b,x,y ta có: ( ) ( )( )2 2 2 2 2 ax by a b x y+ ≤ + + D u “=” x y ra khi và ch khi ax=by. II. M t s bài t p ví d : Ví d 1: Cho o n th ng AB=a. C là i m trên o n th ng AB. V các hình vuông ABCD và CBFG. Xác nh v trí di m C ACDE CBFGS S+ t giá tr nh nh t. L i gi i: t AC = x
  • 35. 35 Ta có CB a x= − (0 x a≤ ≤ ) 2 ACDES x= , 2 ( )CBFGS a x= − 2 2 ( )ACDE CBFGS S x a x+ = + − 2 2 2 2x a ax x= + − + 2 2 2 2( ) 4 2 a a x ax= − + + 2 2 2 2 2 2 2 a a a x   = − + ≥    (không i) D u " "= x y ra 0 2 2 a a x x⇔ − = ⇔ = Ví d 2: Cho o n th ng BC c nh. A là i m di ng sao cho tam giac ABC nh n. AA’ là ư ng cao và H là tr c tâm c a tam giác ABC. Xác nh v trí i m A '. 'AA HA t giá tr l n nh t. L i gi i: Xét 'A BH và 'A AC có ( )' ' 90 , ' 'o BA H AA C A BH A AC= = = (hai góc nh n có c nh tương ng vuông góc) Do ó ' ' ' ' '. ' ' . ' . ' ' HA A B A BH A AC AA HA A B A C A C AA ⇒ = ⇒ =∼ Ta có 2 ' . ' ' ( ' ) ' . 'A B A C A B BC A B A B BC A B= − = − 2 2 ( ' . ' ) 4 2 BC BC A B BC A B= − − + 22 2 ' . 4 2 4 BC BC BC A B   = − − ≤    V y 2 AA'.HA' . 4 BC ≤ (không i) D u " "= x y ra ' 2 BC A B⇔ = 'A⇔ là trung i m BC A⇔ thu c trung tr c BC. V y ABC nh n nên A n m ngoài ư ng tròn ư ng kính BC. Ví d 3: Trong các t giác n i ti p hình ch nh t cho trư c. Tìm t giác có t ng bình phương các c nh nh nh t. L i gi i: AMQ có 90o A = theo nh lí Pitago ta có 2 2 2 QM AM AQ= + Tương t 2 2 2 MN BM BN= + , 2 2 2 NP CN CP= + , 2 2 2 PQ DP DQ= + Do ó 2 2 2 2 2 2 2 2 2 MN NP PQ BM BN CN CP DP DQ+ + = + + + + + Ta có 2 2 2 2 2 2( ) ( ) ( ) 1 2 2 2 AM BM AM BM AM BM AM BM AB + + − + + = ≥ =
  • 36. 36 Ch ng minh tương t ta có 2 2 21 2 CP DP CD+ ≥ 2 2 21 2 DQ AQ AD+ ≥ Do ó ( )2 2 2 2 2 2 2 21 2 MN NP PQ QM AB BC CD DA+ + + ≥ + + + (không i) D u " "= x y ra AM BM BN CN CP DP DQ AQ =  = ⇔ ⇔ =  = MNPQ là hình thoi. Ví d 4: Cho i m A c nh n m ngoài ư ng tròn ( );O R . Qua A v ư ng th ng d c t ư ng tròn ( )O t i hai i m B;C.Xác nh vi trí c a d t ng AB AC+ t giá tr nh nh t. L i gi i: V cát tuy n ADE qua O Xét ABE và ACD có A (chung); AEB ACD= (hai góc n i ti p cùng ch n cung BD) Do ó . . AB AE ABE ACD AB AC AE AD AD AC ⇒ = ⇒ =∼ Mà ( )( ) 2 2 2 2 .AE AD OA OE OA OE OA OE OA R= + − = − = − Ta có ( 2 . )AB AC AB AC AB AC+ = + − 2 ( ) 2 . 2 .AB AC AB AC AB AC= − + ≥ 2 2 2 OA R= − (không i) D u " "= x y ra d⇔ là ti p tuy n c a ( );O R . Ví d 5: Cho n a ư ng tròn ( );O R ư ng kính AB. M là i m chuy n ng trên n a ư ng tròn. Xác nh v trí i m M 3MA MB+ t giá tr l n nh t. L i gi i: 90o AMB = (góc n i ti p ch n n a ư ng tròn) MAB có 90o M = nên theo nh lí Pitago ta có 2 2 2 2 4MA MB AB R+ = = Áp d ng B T 2 2 2 2 ( )( )ax by a b x y= ≤ + + Ta có: ( )2 2 3 3 (1 3)MA MB MA MB MA MB+ = + ≤ + + 2 2 21 2 BN CN BC+ ≥
  • 37. 37 2 4.4 4 .R R= = 3 4MA MB R= ≤ (không i) D u " "= x y ra 3.MA MB⇔ = MAB⇔ là n a tam giác u ⇔ s 60o MA = .
  • 38. 38 Ph n 3: C C TR TRONG LƯ NG GIÁC ▼ D ng 1: S d ng b t ng th c c a hàm Sinx va Cosx. I. Phương pháp gi i: Thông thư ng gi i m t bài c c tr ta s d ng các b t ã ư c ch ng minh. Tương t , lư ng giác v n có nh ng b t riêng bi t. i v i hàm s ơn gi n ch có sin và cos. Ta s d ng: 1 1 1 1 Sinx Cosx − ≤ ≤  − ≤ ≤ II. M t s bài t p ví d : Ví d 1: Tìm GTLN, GTNN c a hàm s : a) y = 1 - 2 sin3x b) y = 1 - 1 sin x− L i gi i: a) Vì 0≤ sin3x ≤1 nên 1≥1 - 2 sin3x ≥1-2 = -1. D u b ng x y ra khi : sin3x = 0⇔ sin3x = 0⇔ 3x = kπ ⇔ x = 3 kπ Vâ GTLN c a hàm s là 1 và GTNN c a hàm s là -1t i x = 3 kπ Vì -1≤ sinx≤1 nên 2 ≥1- sinx ≥0 ⇔ 2 ≥ 1 sin x− ≥0 ⇔ 1+ 2 ≥1+ 1 sin x− ≥1. 1+ 1 sin x− = 1 + 2 khi sinx = -1 ⇔ x = 2 2 k π π− + 1+ 1 sin x− = 1 khi sinx = 1 ⇔ x = 2 2 k π π+ Vâ GTLN c a hàm s là 1+ 2 t i x = x = 2 2 k π π− + và GTNN c a hàm s là 1 t i x = 2 2 k π π+ . V n có th s d ng m t s kĩ năng cơ b n tìm c c tr : Ví d 2: Tìm GTLN c a sin12 x + cos12 x L i gi i: Cách 1: Vì -1≤sinx≤1 và -1≤cosx⇒ 1 nên ta có : sin12 x ≤ sin2 x và cos12 x ≤ cos2 x ⇒ sin12 x + cos12 x ≤ sin2 x + cos2 x = 1 Cách 2:
  • 39. 39 Ta sin12 x + cos12 x = 1 – 2sin6 x.cos6 x ≤1. V y GTLN cu sin12 x + cos12 x là 1. Ví d 3: Tìm GTLN, GTNN cu : sinx + sin 2 3 x π  +    ( Bài t p c n qua bư c bi n i) L i gi i: Ta có sinx + sin 2 3 x π  +    = 2sin 3 x π  +    .cos 3 π = sin 3 x π  +    . Mà -1≤ sin 3 x π  +    ≤1 nên GTNN c a sinx + sin 2 3 x π  +    là -1 và GTLN là 1. Ví d 4: Tìm GTLN , GTNN c a bi u th c : 2 2 ( )(1 ) (1 )(1 ) a b ab a b + − + + ∀ a,b L i gi i: ( i v i bài t p này, ban u không ph i là d ng lư ng, ta ph i ưa v lư ng giác qua các phép bi n i tìm c c tr ). t a= tan x b= tan y Ta có: 2 2 ( )(1 ) (1 )(1 ) a b ab a b + − + + = 2 2 (tan tan )(1 tan .tan ) (1 tan )(1 tan ) x y x y x y + − + + = 2 2 sin( ) c ( ) . cos .cos cos .cos 1 1 . cos cos x y os x y x y x y x y + + = 1 2 sin 2( )x y+ ≤ 1 2 ( vì sin 2( )x y+ ≤1) ⇒ - 1 2 ≤ 2 2 ( )(1 ) (1 )(1 ) a b ab a b + − + + ≤ 1 2 V y GTNN c a bi u th c là - 1 2 và GTLN là 1 2 . ▼ D ng 2: Hình thành bình phương I. Phương pháp gi i: D a trên s chuy n i qua l i gi a sin và cos, s d ng các công th c lư ng giác. II. M t s bài t p ví d : Ví d 1: Tìm GTLN c a bi u th c A= cosA.cosB.cosC
  • 40. 40 L i gi i : Ta có: cosA.cosB.cosC = [ ] 1 ( ) ( ) cos 2 cos A B cos A B C+ + − = [ ] 1 cos ( ) cos 2 C cos A B C− + − = 2 21 1 cos . ( ) ( ) 2 4 cos C C cos A B cos A B   − − − + −   + 21 1 sin ( ) 8 A B − −  = 1 8 2 2 21 1 1 cos ( ) sin ( ) 2 2 8 C cos A B A B   − − − − −   ≤ 1 8 D u b ng x y ra khi cosC = 1 ( ) 2 cos A B− cosC = 1 2 ⇔ sin( )A B− = 0 A B= ⇔ A B= =C = 60o . V y GTLN c a bi u th c A là 1 8 khi ∆ ABC u ▼ Dang 3: S d ng các b t ng th c lư ng giác trong tam giác. I. Phương pháp gi i: Trong tam giác ABC nh n: tan tan tan tan tan tan 1 2 2 2 2 2 2 cot .cot cot cot cot cot 1 tan tan tan tan .tan .tan A B B C C A A B B C C A A B C A B C ∗ + + = ∗ + + = ∗ + + = a) tanA+ tanB+ tanC 3 3≥ b) tan tan tan 3 2 2 2 A B C + + ≥ c) 1 tan .tan .tan 2 2 2 3 3 A B C ≤ d) 3 cos cos cos 2 A B C+ + ≤ ► Ch ng minh: a) Áp d ng b t Côsi cho 3 s dương: 3tanA tanB tanC 3 tan .tan .tan 3tanA tanB tanC 3 tanA tanB tanC 3 2(tanA tanB tanC) 27(tanA tanB tanC) (tanA tanB tanC) 27 tanA tanB tanC 3 3 A B C+ + ≥ ⇔ + + ≥ + + ⇔ + + ≥ + + ⇔ + + ≥ ⇔ + + ≥ ( pcm)
  • 41. 41 b) Áp d ng b t ng th c: ( ) ( ) 2 3a b c ab bc ac+ + ≥ + + ta coù: 2 tan tan tan 3 tan tan tan tan tan tan 2 2 2 2 2 2 2 2 2 A B C A B B C C A    + + ≥ + +        = 3 ⇔ tan tan tan 2 2 2 A B C + + ≥ 3 ( pcm) c) Áp d ng b t Côsi 2 2 23tan tan tan tan tan tan 3 tan tan tan 2 2 2 2 2 2 2 2 2 A B B C C A A B C + + ≥ ⇔ 3 2 2 2 1 tan tan tan 2 2 2 3 A B B   ≤     ⇔ 1 tan tan tan 2 2 2 3 3 A B B ≤ ( pcm) D u b ng x y ra khi ∆ ABC u. Cách 1: Xét cos cos cos 60A B C cos+ + + = 2 2 2 A B A B cos cos + −            + 60 60 2 2 2 C C cos cos    + −         60 2 2 2 A B C cos cos   + +  ≤ + =         60 60 4 . 4 4 A B C A B C cos cos + + + + − − = 4 60cos≤ 1 cos cos cos 2 2 A B C⇔ + + + ≤ 3 cos cos cos 2 A B C⇔ + + ≤ ( pcm) Cách 2: ( ng d ng tích vô hư ng ch ng minh) L y các vectơ 1e , 2e , 3e như hình v và có dài là 1: 1 2 3e e e= = =1. Hi n nhiên ta có: ( ) ( ) ( ) ( ) 2 1 2 3 1 2 2 3 3 1 0 3 2 , 2 , 2 , 0 e e e cos e e cos e e cos e e + + ≥ ⇔ + + + ≥ 3 cos cos cos 2 A B C⇔ + + ≤ ⇒ pcm D u b ng x y ra khi 1 2 3e e e+ + = 0 ABC⇔ ∆ ABC⇔ ∆ u Có th s d ng các b t th c trên ho c khai thác thêm các b t sau trong tam giác (ph i ch ng minh trư c khi áp d ng): ) tan tan tan 3 3n n n n e A B C+ + ≥ 3 3 )sin sin sin 2 f A C B+ + ≤ A B C
  • 42. 42 2 2 2 )cot cot cot 3 3 ) 4 g A B C h cos A cos B cos C + + ≥ + + ≥ 2 2 2 3 )sin sin sin 2 2 2 2 3 ) 2 2 2 2 3 )sin sin sin 2 2 2 4 A B C i A B C j cos cos cos A B C k + + ≤ + + ≤ + + ≥ 2 2 2 )tan tan tan 9m A B C+ + ≥ (∆ nh n) 2 2 2 ) tan tan tan 1 2 2 2 3 3 )sin .sin .sin 8 1 )sin .sin .sin 2 2 2 8 3 3 ) os . os . os 2 2 2 8 A B C n o A B C A B C p A B C q c c c + + ≥ ≤ ≤ ≤ 2 2 2 9 ) os os os 2 2 2 4 A B C l c c c+ + ≤ II. M t s bài t p ví d : Ví d 1: Tìm GTNN c a bi u th c: A= 2 2 2 sin sin sin 2 2 2 A B C + + sin .sin .sin 2 2 2 A B C
  • 43. 43 L i gi i: Ta có A= 1 cos 1 cos 1 cos sin .sin .sin 2 2 2 2 2 2 A B C A B C− − − + + + = 3 1 1 4sin .sin .sin sin .sin .sin 2 2 2 2 2 2 2 2 A B C A B C  − + +    (vì cos cos cos 1 4sin .sin .sin 2 2 2 A B C A B C+ + = + ) =1− sin .sin .sin 2 2 2 A B C 1 1 8 ≥ − = 7 8 V y giá tr nh nh t c a bi u th c là : 7 8 . Ví d 2: Tìm GTNN c a bi u th c B= 6 6 6 tan tan tan 2 2 2 A B C + + L i gi i: Không s d ng gián ti p các b t th c cm trên, ta s d ng các b t toán h c quen thu c. Ta có: ( )6 6 6 3 3 3 3 3 3 tan tan tan tan tan .tan tan .tan tan 1 2 2 2 2 2 2 2 2 2 A B C A B B C C A + + ≥ . N u x = tan tan 2 2 A B , y = tan tan 2 2 B C , z = tan tan 2 2 C A , thì x+y+z = 1. Áp d ng BCS v i hai dãy , ,x y z vaø 3 3 3 , ,x y z ta có: ( )( ) ( ) 23 3 3 2 2 2 x y z x y z x y z+ + + + ≥ + + ( ) 22 3 x y z + + ≥      = 1 9 ( )3 3 3 1 2 9 x y z⇔ + + ≥ T (1) và (2) 6 6 6 1 tan tan tan 2 2 2 9 A B C ⇒ + + ≥ . D u b ng x y ra khi x = y = z ⇔ A=B=C . V y GTNN c a B là 1 9 . 1. Tìm GTLN c a bi u th c D= tan .tan .tan 4 4 4 A B C
  • 44. 44 2. Cho , , 0 . . a b c a Sinx b Siny c ≥  + = Tìm GTLN c a bi u th c E= 2 2 os osc x c y a b + . ▼ D ng 4: Bi u th c ch a các hàm s lư ng giác. I. Phương pháp gi i: Gi s các góc A, B, C tho mãn hai i u ki n: 1) ) ( ) ( ) 2 2 A B f A f B f +  + ≥     ho c 2 ( ). ( ) 2 A B f A f B f +  ≥     ng th c x y ra khi A=B; 2) 3( ) 2 3 2 C f C f f π π   +   + ≥          ho c 2 3( ). 3 2 C f C f f π π   +   ≥          ng th c x y ra khi C= 3 π . Khi c ng ho c nhân (1), (2) ta s có b t ( ) ( ) ( ) 3 3 f A f B f C f π  + + ≥     (3) ho c 3 ( ). ( ). ( ) 3 f A f B f C f π  ≥     (4) ng th c x y ra khi A=B=C. Tương t ta cũng có b t v i chi u ngư c l i Xét các VD sau: Ví d 1: Trong tam giác ABC, tìm GTNN c a bi u th c 1 1 1 1 sin 1 sin 1 sinA B C + + + + + L i gi i: Ta có: 1 1 1 sin 1 sinA B + + + 4 2 sin sinA B ≥ + + (áp d ng 1 1 4 x y x y + ≥ + ) ( ) 4 4 2 2 sin sin 2 2 sin . 2 2 A B A BA B cos ≥ = + −+ + + (áp d ng BCS) 2 1 sin 2 A B ≥ + + ⇒ ≥ 1 1 2 + (5) 1+ sinA 1+ sinB A + B 1+ sin 2 (có d ng (1))
  • 45. 45 Tương t sin 60 60 ≥ 1 1 2 + (6) 1+ sinC 1+ C+ 1+ sin 2 .C ng (5) và (6) ta có: 1 1 1 1 1 sin 1 sin 1 sin 1 sin 60A B C + + + + + + + 1 1 2 60      ≥ +       A + B C+1+ sin 1+ sin 2 2 4 1 sin 60 ≥ + ( Cũng làm tương t các bư c (5), (6)) Suy ra 1 1 1 1 sin 1 sin 1 sinA B C + + + + + 3 1 sin 60 ≥ + 4 3 2 2 3 = + V y GTNN c a bi u th c là 4 3 2 2 3+ . D u b ng x y ra khi ABC∆ u. Ví d 2: Trong tam giác ABC, tìm GTNN c a bi u th c 1 1 1 1 . 1 . 1 sin sin sinA B C       + + +            L i gi i: Ta có: 1 1 1 . 1 sin sinA B     + +        1 1 1 1 sin sin sin .sinA B A B = + + + 2 2 1 1 sin .sin sin .sinA B A B   ≥ + +     2 1 1 sin .sinA B   = +    ( ) ( ) 2 2 1 cos A B cos A B    = +  − − +  ( ) 2 2 2 1 1 1 1 sin 2 A Bcos A B        ≥ + = + + − +      2 1 1 60 sin 2 C     ≥ +  +     (có d ng (1))
  • 46. 46 Tương t 2 1 1 1 1 . 1 1 60sin sin 60 sin 2 CC        + + ≥ +     +         (8) Nhân (7) và (8) ta ư c 1 1 1 1 1 . 1 . 1 . 1 sin sin sin sin 60A B C         + + + +                2 1 1 1 . 1 60sin sin 2 2 A B C         ≥ + +   + +           4 1 1 sin 60   ≥ +    Suy ra 1 1 1 1 . 1 . 1 sin sin sinA B C       + + +            3 1 1 sin 60   ≥ +    3 2 1 3   = +    V y GTNN c a bi u th c là 3 2 1 3   +    khi ABC∆ u. ▼ D ng 5: S dung o hàm I . Ki n th c c n n m: gi i các d ng bài toán này c n s d ng t i m t s cong th c tính o hàm sau ây: ( ) ( ) ( ) ( ) ( ) ( ) ( ) ( ) ( ) 2 2 ' ' '. ' os ' '. 1 tan ' ' tan ' Sinx Cosx Sinu u Sinx Cosx Sinx C u u Sinx x Cos x u u Cos u = = = − = − = = II. M t s bài t p ví d : .Ví d 1: Tìm giá tr l n nh t và nh nh t c a hàm s ( ) 3cos3 2cos2 9cos 2y f x x x x= = + + + L i gi i: L i gi i: TX : D=R Ta có ( ) ( ) ( )3 2 3 4cos 3cos 2 2cos 1 9cos 2y f x x x x x= = − + − + + 3 2 12cos 4cosx x= +
  • 47. 47 10 243 16 8− 0 t t = cos x , 1 1t− ≤ ≤ Ta có ( ) 3 2 12 4y g t t t= = + ( )' ' 2 ' 2 36 8 0 36 8 0 y g t t t y t t = = + = ⇔ + = ( )4 9 2 0 0 2 9 t t t t ⇔ + = = ⇔  = −  B ng bi n thiên t -1 2 9 − 0 1 ( )' g t + 0 - 0 + ( )g t Căn c vào b ng bi n thiên ta ư c: max f(x) = max g(t) = 16 min f(x) = min g(t) = -8 Ví d 2: Tìm giá tr l n nh t và giá tr nh nh t c a hàm s ( ) 2 2 cos 2cos 5 cos 4cos 8y f x x x x x= = − + + + + L i gi i: TX : D=R t t = cos x, 1 1t− ≤ ≤ Ta có ( ) 2 2 2 5 4 8y g t t t t t= = − + + + + Dg(x)= [-1,1] ( )' ' 2 2 1 2 2 5 4 8 t t y g t t t t t − + = = + − + + + ( ) ( ) ( )( ) 2 2 2 2 1 4 8 2 2 5 2 5 4 8 t t t t t t t t t t − + + + + − + = − + + +
  • 48. 48 2 2 5+ 2 13+ 5 ' 0y = ( ) ( )2 2 1 4 8 2 2 5 0t t t t t t⇔ − + + + + − + = ( ) ( )2 2 2 2 5 1 4 8 0t t t t t t⇔ + − + = − + + = (do 1 1t− ≤ ≤ ) ( ) ( ) ( ) ( ) ( )( ) 2 22 2 2 2 5 1 4 8 2 1 0 24 12 0 2 1 1 1 2 2 2 1 t t t t t t t t t t t t t  + − + = − + + ⇔  + − ≥ + = ⇔  − ≤ ≤  = − ⇔ ⇔ = − − ≤ ≤ B ng bi n thiên t -1 1 2 − 1 ( )' g t + 0 - ( )g t Căn c vào b ng bi n thiên ta có max f(x) = max g(x) = 2 13+ min f(x) = min g(x) = 5 Ví d 3: Cho cos 2 cos 2 1, ,x y x y R+ = ∀ ∈ .Tìm giá tr nh nh t c a hàm s : 2 2 tan tanA x y= + L i gi i: Ta có: 2 2 tan tanA x y= + ( ) ( )2 2 tan 1 tan 1 2x y= + + + − 2 2 2 2 1 1 2 2 2 2 cos cos 1 cos2 1 cos2 2cos 2 2cos2 2 cos 2 cos2 2 x y x y x x x x = + − = + − + + − + = − + + t cos2t x= v i 1 1t− ≤ ≤ , ta có :
  • 49. 49 ( ) ( ) ( ) 2 2 22 2 1 ( ) , 1 1 2 6 2 1 1 '( ) 0 22 t t A f t t t t t f t t t t − + = = − ≤ ≤ − + + − ⇒ = = ⇔ = − + + t 1− 1 2 1 '( )f t − 0 + ( )f t 2 3 V y min A 2 3 = khi , 6 x π π= ± + ∈»k k ( )g x nh nh t 2 1 sin 3 x⇔ = ⇒ min ( )g x 2 1 1 5 5 3 3 3 3 3   = − + =    Do ó : 1 3 4 8 1 1 3 5 3 5 y y+ ≤ ≤ + ⇔ ≤ ≤ V y max 8 5 y = ; min 4 3 y = ► . M t s bài t p d ng tương t : 1.Tìm giá tr l n nh t c a hàm s : 2 21 1 1 cos 5 2sin 2 2 y x x= + + + 2.Tìm giá tr nh nh t c a: 2 2 2 2 2 2 1 1 sin cos sin cos y x x x x     = + + +        3.Tìm giá tr l n nh t và giá tr nh nh t c a hàm s : 1
  • 50. 50 2 2 ( ) 2sin 3sin cos 5cosy f x x x x x= = + + 4.Tim giá tr l n nh t và giá tr nh nh t c a hàm s : 2 4sin 2 sin 2 4 y x x π  = + +    5.Tìm giá tr nh nh t c a hàm s : 1 1 ,0 sin cos 2 y x x x π = + < < 6.Tìm giá tr l n nh t và giá tr nh nh t c a hàm s : ( )( )( ) 2sin cos 2cos sinf x x x x x= + − 7. Tìm giá tr l n nh t và giá tr nh nh t c a hàm s : 4 2 4 2 3cos 4sin 3sin 2cos x x y x x + = + 8.Cho tam giác ABC tìm giá tr l n nh t c a: ( )3 cos 3 cos cosP B A C= + + 9.Tìm giá tr nh nh t c a bi u th c : 4 4 2 2 cot cot 2tan .tan 2P a b a b= + + + 10.Cho , ,α β δ tho mãn i u ki n : 2 2 2 cos cos cos 1α β δ+ + = Tìm giá tr l n nh t c a: 2 2 2 1 cos 1 cos 1 cosy α β δ= + + + + + 11.Tìm giá tr nh nh t c a hàm s ( ) 3 2 2 1 cos sin cos sin y x x x x = + + 12.Tìm giá tr nh nh t c a hàm s 10 10 cos siny x x= + 13. Cho ∆ ABC. Tìm giá tr l n nh t c a bi u th c M= cos2A + cos2B – cos2C 14. Tìm GTLN và GTNN c a hàm s 2 cos sin cos 2 x y x x + = + − 15. Tìm GTLN và GTNN c a hàm s 2 2 cos sin cos sin 1 x x x y x + = + (1) 16. nh m hàm s ( )4 4 2 sin cos sin cos cos2y x x m x x x= + + (1)
  • 51. 51 có giá tr l n nh t không l n hơn 2 17. Tìm GTLN và GTNN c a hàm s cos siny x x= + 18. Tìm giá tr l n nh t c a hàm s ( )2 2 4 3sin 1 4sin cos x x y x − = v i 0 6 x π < < 19. Cho ABC∆ có 3 góc nh n, tìm giá tr nh nh t c a bi u th c P = tanA.tanB.tanC 20. Tìm giá tr l n nh t c a bi u th c 1 tan tan 1 tan tan 1 tan tanM A B B C C A= + + + + + v i A, B, C >0 và A + B + C = 2 π 21. Trong m i tam giác ABC,nh ng tam giác nào làm cho bi u th c sau t giá tr l n nh t: Hư ng d n và áp s : 1. Ta có: 2 21 5 1 1 cos sin 2 4 2 y x x= + + + Áp d ng B T Bunhiacopski cho 4 s , ta có: 2 21 5 1 1 cos sin 2 4 2 x x+ + + 2 2 2 21 5 1 1 1 . 1 cos sin 2 4 2 x x≤ + + + + 9 1 22 2 4 2 2 ≤ + = V y max 22 2 y = D u “=” x y ra khi : 2 21 5 1 cos sin 2 4 x x+ = 2. Ta có: ( ) 2 2 2 2 2 2 2 2 2 2 2 2 2 1 1 1 1 sin cos 1 1 sin cos sin cos cos cos x x x x x x x x           + + + ≤ + + + +                      2 2 2 2 2 2 2 2 2 2 2 1 1 1 1 1 sin cos sin cos sin cos 2 sin cos x x x x x x x x       ⇒ + + + ≥ + + +            3 3 3 3 3 3 sin sin sin cos cos cos A B C M A B C + + = + +
  • 52. 52 2 2 2 1 1 1 2 sin .cosx x   ≥ +    ( ) 2 2 2 1 4 1 25 1 1 4 2 sin 2 2 2x   ≥ + ≥ + =    V y min 25 2 y = D u “=” x y ra khi: 2 2 sin cos 4sin 2 1 x x x x π π  = ⇔ = ± + = k 3.Ta có : 2 2 2sin 3sin cos 5cosy x x x x= + + ( ) 3 5 1 cos2 sin 2 1 cos2 2 2 x x x= − + + + ( ) 7 3 sin 2 +cos2x 2 2 x= + 7 3 2 cos 2 2 2 4 x π  = + −    Ta có: ( ) ( ) 3 2 3 2 3 2 1 cos 2 1 cos 2 4 2 2 4 2 1 7 3 2 1 7 3 2 cos 2 7 3 2 2 2 2 4 2 x x x π π π     − ≤ − ≤ ⇔ − ≤ − ≤          ⇔ − ≤ + − ≤ +    V y max ( )1 7 3 2 2 y = + ,min ( )1 7 3 2 2 y = − 4. Ta có: 2 4sin 2 sin 2 4 y x x π  = + +    ( )2 1 2cos sin 2 +cos2x =2+sin 2x - cos2x =2+ 2 sin 2 4 x x x π = − +   −    V i 1 sin 2 1 2 2 2 2 4 x y π  − ≤ − ≤ ⇔ − ≤ ≤ +    V y max 2 2y = + , min 2 2y = − 5. V i 0 2 x π < < cos 0x⇒ > và sin 0x >
  • 53. 53 Áp d ng B T Cauchy, ta có: 1 1 sin cos y x x = + 2 2 2 2 2 sin cos sin 2x x x ≥ = ≥ D u “=” x y ra khi: sin 2 1 1 1 cos sin 2 x x x =   = tan 1 0, 4 2 x x π π  ⇔ = ⇔ = ∈    6.Ta có: ( )( )( ) 2sin cos 2cos sinf x x x x x= + − 2 2 4sin cos 2sin 2cos sin cosx x x x x x= − + − ( )2 2 3sin cos 2 cos sinx x x x= + − 3 sin 2 2cos2 2 x x= + ( ) 1 4cos2 3sin 2 2 x x= + 5 4 3 cos2 sin 2 2 5 5 x x   = +    t 4 3 cos ,sin 5 5 α α= = v i 0 2 π α< < Ta có : ( ) ( ) 5 5 ( ) cos2 cos sin 2 sin cos 2 2 2 f x x x xα α α= + = − V i ( ) ( ) 5 5 5 1 cos 2 1 cos 2 2 2 2 x xα α− ≤ − ≤ ⇔ − ≤ − ≤ 5 5 ( ) 2 2 f x⇔ − ≤ ≤ V y max 5 ( ) ; 2 f x = min 5 ( ) 2 f x = − 7. Ta có: ( ) ( ) 22 24 2 4 2 4 2 3 1 sin 4sin3cos 4sin 3sin 2cos 3sin 2 1 sin xx x y x x x x − ++ = = + + − 4 2 4 2 4 2 3sin 2sin 3 1 1 3sin 2sin 2 3sin 2sin 2 x x x x x x − + = = + − + − + t 2 4 2 2 1 5 ( ) 3sin 2sin 2 3 sin 3 3 g x x x x   = − + = − +    ( )g x l n nh t 2 sin 1x⇔ = ⇒ max 2 1 5 ( ) 3 1 3 3 3 g x   = − + =   
  • 54. 54 8. Ta có: 3 cos 6cos cos 2 2 A C A C P B + − = + 2 2 2 3 cos 6sin cos 2 2 3 1 2sin 6sin 2 2 2 3 sin 6sin 3 2 2 3 5 3 5 3 2 3 sin 2 2 2 2 B A C B B B B B B − = +   ≤ − +    ≤ − + +   ≤ − − + ≤     Suy ra : max 5 3 2 P = khi cos 1 302 3 120 sin 2 2 A C A C B B − =  = =  ⇔  = =  9. Ta có: ( ) 2 4 4 2 2 2 2 cot cot cot cot 2cot .cota b a b a b+ = − + ( ) 2 2 2 2 2 2 2 cot cot 2cot .cot 2tan .tan 2P a b a b a b⇒ = − + + + ( ) ( ) 22 2 2 2 2 2 cot cot 2 cot .cot tan .tan 2cot .cot .tan .tan 4cot .cot .tan .tan 2 a b a b a b a b a b a b a b = − + + − + + ( ) ( ) 2 22 2 cot cot 2 cot .cot tan .tan 4 2 6a b a b a b= − + − + + ≥ D u “=” x y ra khi và ch khi : cot cot 0 cot .cot tan .tan 0 cot 1 4 a b a b a b a b a b a π− = =  ⇔ ⇔ = =  − = =  V y min 6P = 10. Áp d ng B T Bunhiacopski cho 6 s , ta có: 2 2 2 1 cos 1 cos 1 cosα β δ+ + + + + 2 2 2 2 2 2 1 1 1 . 1 cos 1 cos 1 cosα β δ≤ + + + + + + + 2 2 2 3. 3 cos cos cosα β δ≤ + + + vì 2 2 2 cos cos cos 1α β δ+ + = nên ta có : 2 2 2 1 cos 1 cos 1 cosα β δ+ + + + + 3. 4≤ 2 3= V y max 2 3y =
  • 55. 55 D u “=” x y ra khi: 2 2 2 1 cos 1 cos 1 cosα β δ+ = + = + 2 2 2 2 2 2 1 cos 1 cos 1 cos 1 cos cos cos 3 1 cos cos cos 3 α β δ α β δ α β δ ⇔ + = + = + ⇒ = = = ⇔ = = = ± 11.Ta có ( ) 3 2 2 1 cos sin cos sin y x x x x = + + 3 2 1 2 cos 4 1 sin 2 2 x x π   = − +            3 2 4 2 2 cos 4 sin 2 x x π  = − +    vì 3 cos 1 cos 1 4 4 x x π π    − ≥ − ⇒ − ≥ −        3 2 2 cos 2 2 4 x π  ⇔ − ≥ −    và 2 2 4 0 sin 2 1 4 sin 2 x x ≤ ≤ ⇒ ≥ suy ra 3 2 4 2 2 cos 4 2 2 4 sin 2 y x x π  = − + ≥ −    D u “=” x y ra 2 cos 1 4 sin 2 1 x x π   − = −   ⇔    = 5 4 x π ⇔ = V y min 4 2 2y = − 12. Ta có: 10 10 cos siny x x= + 5 5 1 cos2 1 cos2 2 2 x x+ −    = +        ( ) ( ) 5 5 5 1 1 cos2 1 cos2 2 x x = + + −  
  • 56. 56 ( 2 3 4 5 5 1 1 5cos2 10cos 2 10cos 2 5cos 2 cos 2 2 x x x x x= + + + + + )2 3 4 5 1 5cos2 10cos 2 10cos 2 5cos 2 cos 2x x x x x+ − + − + − ( )2 41 2 20cos 2 10cos 2 32 x x= + + ( )2 41 1 10cos 2 5cos 2 16 x x= + + ( ) 221 5 cos 2 1 4 16 x = + −    M t khác 2 2 0 cos 2 1 1 1 cos 2 2x x≤ ≤ ⇔ ≤ + ≤ ( ) 22 1 1 cos 2 4x⇔ ≤ + ≤ ( ) 22 5 5 1 cos 2 20x⇔ ≤ + ≤ ( ) ( ) ( ) 221 1 1 5 4 5 1 cos 2 4 20 4 16 16 16 x ⇔ − ≤ + − ≤ −    1 1 16 y⇔ ≤ ≤ V y max y =1, d u “=” x y ra khi x=0 min y = 1 16 , d u “=” x y ra khi x= 4 π 13. Ta có M= cos2A + cos2B – cos2C = 2cos(A+B)cos(A-B) + 1 – 2cos2 C = -2cosC cos(A-B) + 1 – 2cos2 C = -2[cos2 C + cos(A-B) cosC + 1 4 cos2 (A-B)] + 1 2 [cos2 (A-B)] +1 = -2[cosC + 1 2 cos(A-B)]2 + 1 2 [1 - sin2 (A-B)] +1 = 3 2 -2[cosC + 1 2 cos(A-B)]2 - 1 2 sin2 (A-B)] 3 2 ≤ D u “=” x y ra ( ) ( ) 1 cos cos 0 2 sin 0 C A B A B  + − = ⇔   − =
  • 57. 57 1 cos 0 2 0 C A B  + = ⇔   − = 1 cos 6 2 2 3 A B C A B C π π  = = = −  ⇔ ⇔   = =  V y max M = 3 2 ng v i ∆ ABC có A = B = 6 π và C = 2 3 π 14. Vì sin cos 2 cos 4 x x x π  + = −    sin cos 2 2 sin cos 2 0 x x x x ⇒ + ≤ < ⇔ + − < hay sin cos 2 0x x+ − ≠ x R∀ ∈ Do ó 2 cos sin cos 2 x y x x + = + − (1) ( )sin cos 2 2 cosy x x x⇔ + − = + ( )sin 1 cos 2 2y x y x y⇔ + − = + (2) (1) có nghi m i v i x ⇔ (2) có nghi m i v i x ( ) ( ) 2 22 2 2 2 1 2 2 2 2 1 4 8 4 2 10 3 0 5 19 5 19 2 2 y y y y y y y y y y ⇔ + − ≥ + ⇔ − + ≥ + + ⇔ + + ≤ − − − + ⇔ ≤ ≤ V y min y = 5 19 2 − − và max y = 5 19 2 − + 15. Ta có : ( ) 24 4 2 2 2 2 sin cos sin cos 2sin cosx x x x x x+ = + − 2 21 1 1 2 sin 2 1 sin 2 2 2 1 1 cos 4 3 1 1 cos4 2 2 4 4 x x x x   = − = −    −  = − = +   
  • 58. 58 Và 1 1 sin cos cos2 sin 2 cos2 sin4 2 4 x x x x x x= = Nên ( ) 3 1 1 2 cos4 sin4 4 4 4 4 6 2cos4 sin4 2cos4 sin4 4 6 m y x x y x m x x m x y   ⇔ = + +    ⇔ = + + ⇔ + = − PT trên có nghi m i v i x ( ) 22 2 2 2 2 2 2 4 6 16 48 32 0 6 4 6 4 4 4 m y y y m m m y ⇔ + ≥ − ⇔ − + − ≤ − + + + ⇔ ≤ ≤ Do ó 2 6 4 max 4 m y + + = Ta có 2 6 4 max 2 2 4 m y + + ≤ ⇔ ≤ 2 2 4 2 4 4 0 m m m ⇔ + ≤ ⇔ + ≤ ⇔ = 16. Ta có (1) ( )2 2 sin 1 cos sin cosy x x x x⇔ + = + (do sin2 x +1≠ 0 ) 1 cos2 1 cos2 1 1 sin2 2 2 2 cos2 2y 1 cos 2 sin2 x x y x y y x x x − +  ⇔ + = +    ⇔ − + = + + ( )1 cos2 sin2 3 1y x x y⇔ + + = − (2) (1) có nghi m i v i x ⇔ (2) có nghi m i v i x ( ) ( ) 2 22 1 1 3 1y y⇔ + + ≥ − 2 2 2 2 2 9 6 1 8 8 1 0 y y y y y y ⇔ + + ≥ − + ⇔ − − ≤ 2 6 2 6 4 4 − + ⇔ ≤ v y 2 6 max 4 y + = và 2 6 min 4 y − =
  • 59. 59 17. Tìm GTLN và GTNN c a hàm s cos siny x x= + L i gi i: Ta có cos siny x x= + ( )( )2 2 1 cos 1 sin 1 1 cos sinx x x x= + ≤ + + (B T Bunhiacopski) ( )2 cos siny x x⇒ ≤ + m t khác cos sin 2 cos 2 4 x x x π  + = − ≤    suy ra 2 2y ≤ D u “=” x y ra cos sin 4cos 1 4 x x x x π π  =  ⇔ ⇔ =   − =    V y max 2 2y = khi 4 x π = Ta có 0 cos 1x≤ ≤ ( K y xác nh) và 0 sin 1x≤ ≤ 2 2 2 2 cos cos cos sin sin sin 1 cos sin cos sin x x x x x x x x x x y  ≤ ≤ ⇒  ≤ ≤ ⇒ = + ≤ + = nên 1y ≥ , d u “=” x y ra khi x = 0 V y min y = 1 khi x = 0 18. Tìm giá tr l n nh t c a hàm s ( )2 2 4 3sin 1 4sin cos x x y x − = v i 0 6 x π < < L i gi i: Vì 1 0 0 sin 6 2 x x π < < ⇒ < < 2 21 0 sin 1 4sin 0 4 x x⇒ < < ⇒ − > Áp d ng B T cô-si cho 2 s 2 3sin x và 2 1 4sin x− ta ư c
  • 60. 60 ( ) ( ) ( ) ( ) 2 2 2 2 22 2 2 4 2 2 3sin 1 4sin 3sin 1 4sin 2 1 sin 3sin 1 4sin 2 cos 3sin 1 4sin (1) 4 x x x x x x x x x x + − ≥ −  − ⇔ ≥ −    ⇒ ≥ − Chia 2 v c a (1) cho 4 cos x ( vì 4 0 cos 0 6 x x π < < ⇒ > ) Ta ư c ( )2 2 4 3sin 1 4sin 1 4cos x x y x − = ≤ d u “=” x y ra 2 2 2 1 3sin 1 4sin sin 7 x x x⇔ = − ⇔ = ta tìm ư c 0 0, 6 x π  ∈    thì 2 1 sin 7 x = V y 1 max 4 y = 19. Ta có 2 2 1 2cos 1 3siny x x= + + + 2 21 1 3 6cos 2 6sin 3 2 x x= + + + Áp d ng B T Bunhiacopski ta có : ( ) ( ) 2 21 1 3 6cos 2 6sin 3 2 5 55 5 6 6 6 y x x y   ≤ + + + +    ⇒ ≤ + = D u “=” x y ra 2 2 2 2 2 2 2 2 3. 3 6cos 2. 2 6sin 3.(3 6cos ) 2.(2 6sin ) 9 18cos 4 12(1 cos ) 7 30cos 7 cos 30 x x x x x x x x ⇔ + = + ⇔ + = + ⇔ + = + − ⇔ = ⇔ = V y 55 max 6 y = 20. Ta có A + B + C = π
  • 61. 61 tan( ) tan( ) A B C A B C π π ⇔ + = − ⇒ + = − tan tan tan 1 tan .tan A B C A B + ⇒ = − − tan tan tan (1 tan .tan )A B C A B⇔ + = − − tan tan tan tan .tan .tanA B C A B C⇔ + + = (1) Vì ABC∆ có 3 góc nh n tan ,tan ,tan 0A B C⇒ > Áp d ng B T cô-si cho 3 s tgA, tgB, tgC 3 tan tan tan 3 tan tan tanA B C A B C+ + ≥ (2) t (1) và (2) ta ư c 3 tan .tan .tan 3 tan tan tanA B C A B C≥ 3 2 (tan .tan .tan ) 27 tan .tan .tan (tan .tan .tan ) 27 tan .tan .tan 3 3 A B C A B C A B C A B C ⇔ ≥ ⇔ ≥ ⇔ ≥ D u “=” x y ra khi tanA = tanB = tanC A B C⇔ = = hay ABC∆ u 21. Vì A + B + C = 2 π ( ) 2 tan tan 2 A B C A B C π π ⇔ + = −   ⇒ + = −    ( ) tan tan 1 cot 1 tan tan tan tan tan tan 1 tan tan tan tan tan tan tan tan 1 A B C A B C A B C A B A B B C C A + ⇔ = = − ⇔ + = − ⇔ + + = M t khác áp d ng B T Bunhiacôpski ta ư c ( )( )2 2 2 1 1 1 1 tan tan 1 tan tan 1 tan tanM A B B C C A≤ + + + + + + + ( )3 3 1 2 3= + = d u b ng x y ra khi tanA tanB = tanB tanC = tanC tanA tan tan tanA B C⇔ = = 6 A B C π ⇔ = = = (do A + B + C = 2 π ) Ta có: sin sin 2sin cos 2cos cos 2cos 2 2 2 2 2 A B A B C A B C A B + − − + = = ≤ (1)
  • 62. 62 Áp d ng B T : 33 3 2 2 a b a b+ +  ≥     , d u “=” x y ra khi a b= Ta có: 3 3 3 sin sin sin sin cos 2 2 2 A B A B C + + ≤ ≤     ( theo(1) ) 3 3 3 sin sin cos 2 2 A B C+ ⇔ ≤ (2) Tương t : 3 3 3 sin sin cos 2 2 B C A+ ≤ (3) 3 3 3 sin sin cos 2 2 C A B+ ≤ (4) C ng (2),(3),(4) ta có: 3 3 3 3 3 3 sin sin sin cos cos cosA B C A B C+ + ≤ + + ⇔ 3 3 3 3 3 3 sin sin sin cos cos cos A B C M A B C + + = + + 1≤ D u “=” x y ra khi sin sin sin 3cos 1 2 A B C A B CA B π = =  ⇔ = = = − = V y max 1M = ⇔ ABC là tam giác u
  • 63. 63 Ph n 5 : BÀI T P TR C NGHI M Bài 1: Cho a + b ≥ 1, giá tr nh nh t c a bi u th c a3 + b3 là A) 1 B) 1/2 C) 1/4 D) 2 Bài 2: Giá tr l n nh t c a hàm s 2 2 1 1 x x y x x + + = − + là A. 1 3 B. 3 C. 3 2 D.5 Bài 3: Giá tr nh nh t c a hàm s 2 2 2 1 2 1 x x y x x − + = + + : A. 1 5 2 + B. 1 5 2 − C. 9 4 2 7 + D. 9 4 2 7 − Bài 4: Cho a + b = 1, giá tr nh nh t c a bi u th c a4 + b4 là A) 2 B) 1 C) 1/8 D) 1/4 Bài 5: Cho a, b, c >0 tho mãn 1 1 2 a c b + = , giá tr nh nh t c a 2 2 a b c b a b c b + + + − − là A.1 B.2 C.3 D.4 Bài 6: Giá tr nh nh t c a hàm s ( ) 2 2 2 1 1 x x f x x x + − = − + Bài 7: GTNN, GTLN c a hàm s 2 2 2 4 5 1 x x y x + + = − + A. Min y = 1, max y = 6 B. Min y = -6, max y = -1 C. Min y =2, max y = 5 D. Min y = -5, max y = -2 A) 0 B) 2 C) 3 D) 4 Bài 8: Cho a, b, c >0, giá tr nh nh t c a bi u th c a b c b c c a a b + + + + + là A) 1 B) 1/2 C) 3/2 D) 2 Bài 9:
  • 64. 64 Cho a, b, c, d >0. Giá tr nh nh t c a bi u th c a b b c c d d a b c d c d a d a b a b c + + + + + + + + + + + + + + + là A) 8/3 B)1/3 C) 2/3 D) 1 Bài 10: Cho hàm s 6 5 cos siny x x= − . Giá tr l n nh t c a y là A) -1 B) 0 C) 1/2 D) 1 Bài 11: Giá tr l n nh t và giá tr nh nh t c a hàm s 2 1 1 x y x x + = + + l n lư t là A) max y = 1, min y = -1/3 B) max y = 2, min y = 1/2 C) max y = 1/2, min y = 1/3 D) max y = 3, min y = 1/3 Bài 12: Gi s x, y, z là nh ng s dương thay i th a x + y + z = 1. Giá tr l n nh t c a bi u th c 1 1 1 x y z x y z + + + + + là A) 3/4 B) 1/3 C) 1 D) 2 Bài 13: Cho các s dương x, y, z sao cho xyz = 1 và n là s nguyên dương. Giá tr nh nh t c a bi u th c 1 1 1 2 2 2 n n n x y z+ + +      + +            là A) 1 B) 2 C) 3 D) 4 Bài 14: Cho sin sin sin 0x y z+ + = . Giá tr l n nh t c a bi u th c 2 4 6 sin sin sinP x x x= + + là A) 0 B) 1/2 C) 1 D) 2 Bài 15: Giá tr nh nh t c a bi u th c: ( ) ( )2 2 2 2 2 2 4cos cos sin 4sin sin sinx y x y x y x y+ − + + − là A) 0 B) 1 C) 2 D) 4 Bài 16: Giá tr l n nh t c a bi u th c ( )( ) ( ) ( ) 2 22 2 1 1 1 x y xy x y + − + + là A) 0 B) 1/2 C) 1 D) 2 Bài 17: Cho x, y, z dương và x + y + z = 1. Giá tr l n nh t c a S = xyz(x+y)(y+z)(z+x) là A) 8/729 B) 1/729 C) 0 D) 1/2 Bài 18:
  • 65. 65 Cho x, y thay i sao cho 0 3 0 4 x y ≤ ≤  ≤ ≤ . giá tr l n nh t c a bi u th c (3-x)(4-y)(2x+3y) là A) 1 B) 6 C) 2 D) 0 Bài 19: Giá tr nh nh t c a bi u th c 2 2 2 2 2 12 37 6 6 18a b a b a b a b+ − − + + + + − + A) 2 B) 5/2 C) 3 D) 5 Bài 20: Cho x2 + y2 = 1. Giá tr l n nh t, giá tr nh nh t c a P = x + y l n lư t là A) max P = 1, min P = 0 B) max P = 0, min P = - 2 C) max P = 2 , min P = 1 D) max P = 2 , min P = - 2 Bài 21: Cho x2 + y2 = u2 + v2 = 1.Giá tr l n nh t c a P= ( ) ( )x u v y u v− + + là A) 2 B) 1 C) 0 D) - 2 Bài 22: Cho ∆ ABC giá tr l n nh t c a 2 2 2 2 2 2 sin sin sin cos cos cos A B C P A B C + + = + + là A) 0 B) 1/2 C) 2 D) 3 Bài 23: Cho x, y, z là 3 góc nh n th a x + y + z = 90o. Giá tr l n nh t c a bi u th c 5 tan tan 5 tan tan 5 tan tanP x y y z z x= + + + + + là A) 2 B) 3 C) 4 3 D) 2 2 Bài 24: Cho , 0 1 x y x y >  + = , giá tr nh nh t c a 22 1 1 P x y x y    = + + +       là A) 25/2 B) 1/2 C) 1 D) 2 Hư ng d n và áp án : 1. T gi thi t 1a b+ ≥ bi n i tương ương ta ư c 3 3 2 3 3 1a b b b+ ≥ − + mà 2 2 1 1 1 3 3 1 3 2 4 4 b b b   − + = − + ≥    2.B 3.D
  • 66. 66 4. T a + b = 1 suy ra a2 + 2ab + b2 =1 m t khác a2 – 2ab + b2 ≥ 0 t ó ta có 2 2 1 2 a b+ ≥ bình phương hai v , k t h p v i bdt 4 2 2 4 2 0a a b b− + ≥ ta ư c 4 4 1 8 a b+ ≥ . 5. T gi thi t ta có 2ac b a c = + v y : ( )2 2 2 33 3 4 2 2 2 2 2 ac a ca b c b a b c a a b c b a c ac + ++ + + + + = + = ≥ − − 6.C 7.A 8. t P = a b c b c c a a b + + + + + Ta có 2(P + 3) = ( ) ( ) ( ) 1 1 1 9a b b c c a a b b c C a   + + + + + + + ≥    + + +  (Bunhiacopski cho 3 c p s ) Suy ra P ≥ 3/2 9. A 10. D 11. A 12. Áp d ng bunhiacopski cho ba c p s tìm ư c max = ¾ 13. Ta có 1 1 2 2 n na a a a + +  ≥ ⇒ ≥    Áp d ng ta tìm ư c min = 3 14. D 15. C 16. B 17. Áp d ng côsi cho 3 s : ( ) ( ) ( ) ( )( )( ) 3 3 1 3 2 3 x y z xyz x y y z z x x y y z z x = + + ≥ = + + + + + ≥ + + + Nhân v theo v , bi n i tìm ư c max = 8/729 18. Có th vi t l i bi u th c ã cho thành: ( )( )( ) 1 6 2 12 3 2 3 6 x y x y− − + Áp d ng côsi cho ba s tìm ư c max = 36. 19. D 20. D 21. A 22. D
  • 67. 67 23. C 24. Áp d ng B.C.S cho 2 c p s (1, 1) và 1 1 ,x y x y   + +    Sau ó bi n i tương ương ta ư c 2 22 1 1 1 1 1 2 x y xy x y      + ≤ + + +          vì 2 1 2 4 x y xy +  ≤ =    2 1 4 1 1 25 xy xy ⇒ ≥   ⇒ + ≥    v y min = 25/2